You are on page 1of 128

PROBLEMA 1

FASES CEFLICA E ORAL


- A principal caracterstica da fase ceflica a ativao do trato GI em prontido para a refeio. Os
estmulos envolvidos so cognitivos e incluem a antecipao e o pensamento sobre o consumo da comida,
o estmulo olfatrio, o estmulo visual (ver e cheirar uma comida apetitosa, quando se est com fome) e
estmulos auditivos.
- Todos esses estmulos resultam em aumento do fluxo parassimptico excitatrio neural para o
intestino. Estmulos sensoriais, como o cheiro, estimulam os nervos sensoriais a ativarem o fluxo
parassimptico para o tronco cerebral.
- A resposta pode ser positiva (antecipao da comida) e negativa (ansiedade). Entretanto, a via final
comum a ativao do ncleo motor do vago, no tronco cerebral, regio de onde os corpos celulares dos
neurnios pr-ganglionares parassimpticos saem; a ativao do ncleo leva atividade aumentada nas
fibras eferentes, passando para o trato GI, pelo nervo vago.
- Por sua vez, as fibras eferentes ativam os neurnios motores ps-ganglionares.
- O fluxo parassimptico aumentado melhora a secreo salivar, a secreo de cido gstrico, a
secreo enzimtica do pncreas, a contrao da bexiga e o relaxamento do esfncter de Oddi (o esfncter
entre o ducto comum da bile e o duodeno).
- A resposta salivar mediada pelo nono nervo craniano; as respostas remanescentes so mediadas
pelo nervo vago.
- Muitas das respostas, que so iniciadas pela presena da comida na cavidade oral, so idnticas s
iniciadas na fase ceflica porque a via eferente a mesma.
- A mastigao subdivide e mistura o alimento com as enzimas amilase salivar e lipase lingual e com
a glicoprotena mucina, que lubrifica o alimento para a mastigao e deglutio.
- Como na fase ceflica, importante perceber que o estmulo da cavidade oral inicia respostas mais
distais do trato GI, incluindo a secreo de aumentada cido gstrico, a secreo aumentadas de enzimas
pancreticas, a contrao da vescula biliar e o relaxamento do esfncter de Oddi, mediado pela via eferente
vagal.

PROPRIEDADES DA SECREO
- Secrees do trato GI vm das glndulas associadas ao trato (as glndulas salivares, pncreas e
fgado), das glndulas formadas pela parede do intestino (p. ex., glndulas de Brunner, no duodeno) e pela
mucosa intestinal.
- Secrees do trato GI e das glndulas associadas incluem gua, eletrlitos, protena e agentes
humorais. A gua essencial para gerar um ambiente aquoso, para a ao eficiente das enzimas. A secreo
de eletrlitos importante para a gerao de gradientes osmticos que direcionam o movimento da gua.
As enzimas digestivas, no fluido secretado, catalisam a quebra de macronutrientes no alimento ingerido.
- A secreo provocada pela ao de substncias efetoras especficas, chamadas secretagogos,
atuando sobre as clulas secretrias. Os secretagogos trabalham por uma das trs vias (endcrina,
parcrina e neurcrina).

CONSTITUINTES DAS SECREES


- Componentes secretores inorgnicos: Eletrlitos (H+, HCO3-), especficos de regies ou de
glndulas dependendo das condies particulares requeridas nessa parte do trato GI. (Alguns so
diferentes. Ex.: HCl no estmago, p/ ativar a pepsina e iniciar a digesto de protenas; HCO3- no duodeno,
neutralizando cido gstrico e fornecendo condies p/ ao de enzimas digestivas no int. delgado).
- Componentes secretores orgnicos: enzimas (para digesto), mucina (lubrificao e proteo da
mucosa) e outros fatores como fatore de crescimento, imunoglobulinas e fatores absortivos.
SECREO SALIVAR
- Principais funes da saliva na digesto incluem
lubrificao e umidificao do material para a
deglutio, solubilizao para o paladar, incio da
digesto de carboidratos, depurao e
neutralizao do refluxo das secrees gstricas
no esfago.

ANATOMIA FISIOLGICA DAS SALIVARES


- Trs pares de glndulas salivares: partida,
submandibular e sublingual. Essas glndulas tm
a estrutura tpica tubuloalveolar das glndulas do
trato GI.

- As clulas nas partes terminais secretrias, ou cinos, so chamadas de clulas acinares e so


caracterizadas por ncleos de situao basal, retculo endoplasmtico rugoso abundante, e grnulos
secretrios localizados em seu pice que contm a enzima amilase e outras protenas secretadas. Existem
tambm clulas mucosas nos cinos; os grnulos nessas clulas so maiores e contm a glicoprotena
especializada mucina.
- Transporte das secrees dos cinos at a boca e que tambm modificam a secreo (modificam
a composio inica e a osmolaridade da saliva.: ductos intercalados > fluido acinar > ductos estriados >
ductos excretores.

COMPOSIO DA SALIVA
- A composio inorgnica inteiramente dependente do estmulo e da intensidade do fluxo salivar.
Nos humanos, a secreo salivar sempre hipotnica. Os principais componentes so: Na+, K+, HCO3,
Ca++, Mg++ e Cl.
- Fluoretos podem ser secretados na saliva, e a secreo de fluoreto forma a base do tratamento
oral com fluoreto para a preveno de cries dentais.
- A concentrao de ons varia com a intensidade da secreo, que estimulada durante o perodo
ps-prandial.
- A secreo primria produzida pelas clulas acinares nas partes secretrias finais (cinos) e
modificada pelas clulas do ducto, quando a saliva passa por eles. A secreo primria isotnica, e a
concentrao dos ons principais similar do plasma.
- A secreo impulsionada de modo predominante pela sinalizao dependente de Ca++, que abre
os canais apicais de Cl, nas clulas acinares. Por conseguinte, o Cl flui para fora do lmen e estabelece o
gradiente osmtico e eltrico. Como o epitlio dos cinos relativamente permevel, Na+ e gua, ento,
passam atravs do epitlio, via junes celulares (transporte paracelular).
- As clulas do ducto excreto e as clulas do ducto estriado modificam a secreo primria, para
produzir a secreo secundria. As clulas do ducto reabsorvem Na+ e Cl e secretam K+ e HCO3 no
lmen. No repouso, a secreo salivar final hipotnica e levemente alcalina.
- A alcalinidade da saliva , provavelmente, importante para a restrio do crescimento da
microbiota na boca, bem como na neutralizao do refluxo de cido gstrico, quando a saliva deglutida.
- Os constituintes orgnicos da saliva, protenas e glicoprotenas, so sintetizados, armazenados e
secretados pelas clulas acinares. Os principais produtos so amilase (uma enzima que inicia a digesto do
amido), lipase (importante para a digesto lipdica), glicoprotena (mucina que forma muco quando
hidratada) e lisozima (ataca as paredes de clulas bacterianas, para limitar a colonizao bacteriana na
boca). Amilase salivar: no necessria em adultos saudveis (excesso de amilase pancretica).
REGULAO DA SECREO SALIVAR
- O controle da secreo salivar exclusivamente neural.
- A secreo salivar estimulada pelas duas subdivises, simptica e parassimptica, do sistema
nervoso autnomo.
- As fibras simpticas para as glndulas salivares se ramificam do gnglio cervical superior. As clulas
acinares e ductos so supridos com terminaes nervosas parassimpticas.
- A estimulao parassimptica aumenta a sntese e a secreo de amilase salivar e de mucina,
melhora as atividades de transporte do epitlio ductular, aumenta muito o fluxo sanguneo para as
glndulas e estimula o metabolismo glandular e seu crescimento.

DEGLUTIO

- O reflexo da deglutio
sequncia rigidamente ordenada de
eventos, que levam o alimento da boca
para a faringe e de l para o estmago.
Esse reflexo tambm inibe a respirao
e impede a entrada do alimento na
traqueia durante a deglutio.
- A via aferente do reflexo da
deglutio comea quando os
receptores de estiramento, mais
notadamente os prximos abertura
da faringe, so estimulados. Impulsos
sensoriais desses receptores so
transmitidos para uma rea no bulbo e
na ponte inferior, chamada centro da
deglutio. Os impulsos motores
passam do centro da deglutio para a
musculatura da faringe e do esfago superior, via vrios nervos cranianos e para o restante do esfago por
neurnios motores vagais.

FASE ESOFGICA
- O esfago, o EES
(superior) e o esfncter
esofgico inferior (EEI)
executam duas funes
principais (Fig. 27-6).
Primeira, impulsionam
o alimento da boca para
o estmago. Segunda,
os esfncteres
protegem as vias
areas, durante a
deglutio, e protegem
o esfago das secrees
gstricas cidas.
- Os estmulos que iniciam as variaes da atividade do msculo liso que resultam nessas funes
propulsoras e protetoras so mecnicos e consistem em estmulo faringeano, durante a deglutio, e em
distenso da parede esofgica. As vias so exclusivamente neurais e envolvem reflexos extrnsecos e
intrnsecos.

ESTMAGO

- O alimento que chega ao estmago, vindo do


esfago, produz estimulao mecnica da
parede gstrica, pela distenso e pelo
estiramento do msculo liso. Nutrientes
predominantemente oligopeptdeos e
aminocidos tambm provocam estimulao
qumica quando presentes, no lmen gstrico.
A regulao da funo do estmago, durante a
fase gstrica, dependente de componentes
endcrinos, parcrinos e neurais. Esses
componentes so ativados por estmulos
mecnicos e qumicos, que resultam em vias reflexas neurais intrnsecas e extrnsecas, importantes para a
regulao da funo gstrica. Neurnios aferentes que se dirigem do trato GI para o sistema nervoso central
(e, numa menor extenso, para a medula espinal) via nervo vago respondem a esses estmulos mecnicos
e qumicos e ativam a eferncia parassimptica.
- O revestimento interno do estmago recoberto por epitlio colunar dobrado, para formar em as
criptas gstricas; cada cripta (ou fosseta) a abertura de ducto, no qual uma ou mais glndulas gstricas
lanam suas secrees.
- Alm de clulas secretoras de muco que revestem toda a superfcie do estmago, h outras duas
glndulas tubulares: oxnticas (parietais) e pilricas.
- Glndulas oxnticas (parietais, ou gstricas)
- Localizadas nas superfcies internas do corpo e do fundo do estmago, constituindo 80%
do estmago proximal.
- Clulas mucosas do clon: secretam, basicamente, muco.
- Clulas ppticas (principais): secretam grandes quantidades de pepsinognio.
- Clulas parietais (oxnticas): secretam cido clordrico e fator intrnseco.
SECREO DO CIDO CLORDRICO
- Secreo pelas clulas parietais de cerca de 160mmol/L de HCl (ph = 0,8). Ao mesmo tempo que
esses ons de hidrognio so secretados, os ons bicarbonato se difundem para o sangue, para que o sangue
venoso gstrico tenha um pH mais alto que o sangue arterial, quando o estmago est secretando cido.
- Principal fora motriz: bomba de hidrognio-potssio (H+ - K+ - ATPase).

- gua na cl parietal -> H+ + OH- no


citoplasma, catalisado pela bomba
(ativo).
- ons K+ tendem a vazar para o
lmen, mas so reciclados, de volta
para a clula, pela bomba.
- A Na+ - K + - ATPase basolateral
produz baixa no Na+ intracelular, o
que contribui para a reabsoro de
Na+ do lmen dos canalculos.

- Assim, a maior parte do K+ e Na+, nos canalculos reabsorvida para o citoplasma celular, e os ons
hidrognio tomam seus lugares nos canalculos.
- Bombeamento de H+, para fora da clula, pela bomba -> OH- se acumula e forma HCO3- com o
CO2 (do metabolismo da clula ou vindo do sangue), catalisado pela anidrase carbnica.
- HCO3- transportado atravs da membrana basolateral, para o fluido extracelular, em troca de ons
cloreto que entram na clula e so secretados pelos canais de cloreto para os canalculos, resultando em
soluo concentrada de Hcl nos canalculos. Hcl ento excretado para fora pela extremidade aberta do
canalculo no lmen da glndula.
- gua passa para os canalculos por osmose devido aos ons extras secretados nos canalculos. A
secreo final tem Hcl concentrado, Kcl e pequena quantidade de NaCl.

- Barreira gstrica: devido alta concentrao de H necessria, deve ocorrer o menor vazamento
possvel, de volta para a mucosa do cido secretado. Assim, h formao de muco alcalino e junes
estreitas, entre as cls epiteliais. Caso seja danificada (excesso de lcool, aspirina, etc), o cido secretado
vaza para a mucosa, de acordo com seu gradiente qumico, lesando a mucosa gstrica.
- Estimulao parassimptica -> acetilcolina -> excita a secreo de pepsinognio pelas cls ppticas,
de Hcl pelas cls parietais e de muco pelas cls da mucosa.
- Gastrina e Histamina -> estimula fortemente a secreo de cido pelas cls parietais, mas tem
pouco efeito sobre as outras clulas.

SECREO E ATIVAO DO PEPSINOGNIO


- Quando secretado, no tem atividade digestiva.
- Assim que entra em contato com HCl -> clivagem -> pepsina ativa (peso molecular 42500 > 35000).
- Pepsina: enzima proteoltica, ativa em meio muito cido (pH entre 1,8 e 3,5), mas em pH acima de
5 no tem quase nenhuma propriedade proteoltica e completamente inativada em pouco tempo.
GLNDULAS PILRICAS SECREO DE MUCO E GASTRINA
- Estruturalmente semelhantes s oxnticas, mas contm poucas cls ppticas e quase nenhuma cl
parietal.
- Contm, essencialmente, cls mucosas idnticas s cls mucosas do colo das glndulas oxnticas.
- Secretam pequena quantidade de pepsinognio e quantidade particularmente grande de muco,
que auxilia na lubrificao e proteo da parede gstrica da digesto pelas enzimas gstricas.
- Tambm liberam gastrina, que tem papel crucial no controle da secreo gstrica.

CLULAS MUCOSAS SUPERFICIAIS


- Toda a superfcie da mucosa, entre as glndulas, apresenta camada contnua de tipo especial de
cls mucosas (cls mucosas superficiais).
- Secretam grande quantidade de muco viscoso, que recobre a mucosa gstrica com camada
gelatinosa de muco, proporcionando barreira de proteo para a parede gstrica, bem como contribuindo
para a lubrificao do transporte de alimento.
- Alm disso: alcalinidade. A parede gstrica subjacente normal no exposta secreo proteoltica
muito cida do estmago. Menor contato com alimentos ou qualquer irritao da mucosa estimula
diretamente as cls mucosas superficiais a secretar quantidades adicionais desse muco.

ESTIMULAO DA SECREO DE CIDO PELO ESTMAGO


- As cls parietais das glndulas oxnticas so as nicas cls que secretam HCl. Secreo controlada
por sinais endcrinos e nervosos. Alm disso, as cls parietais so controladas pelas clulas semelhantes s
enterocromafins (ECL).
- ECL: funo primria secretar histamina no espao adjacente s cls parietais das
glndulas. A intensidade da secreo de HCl (pelas parietais) diretamente proporcional quantidade de
histamina secretada pelas cls ECL.
- So estimuladas pela gastrina, formado na poro antral da mucosa gstrica, em resposta
s protenas nos alimentos que esto sendo digeridos.
- Podem ser estimuladas tambm por substncias hormonais secretadas pelo sistema
nervoso entrico da parede gstrica.

ESTIMULAO DA SECREO DE CIDO PELA GASTRINA


- Secretado pelas cls da gastrina (clulas G), localizadas glndulas pilricas no estmago distal.
- Peptdeo secretado em duas formas: G34 (34AA) e G17 (17AA, mais abundante).
- Alimentos proteicos -> regio antral do estmago -> efeito estimulador das cls da gastrina nas
glndulas pilricas -> liberao de gastrina no sangue -> cls ECL -> liberao de histamina -> ao nas
glndulas oxnticas profundas -> secreo de HCl gstrico.

REGULAO DA SECREO DE PEPSINOGNIO


- Estimulao das cls ppticas por acetilcolina, liberada pelo plexo mioentrico.
- Estimulao da secreo das cls ppticas, pelo cido no estmago. O cido no atua diretamente,
mas provoca outros reflexos nervosos entricos que amplificam os sinais nervosos para as cls ppticas.
- Pessoas que no produzem a quantidade adequada de cido -> secreo de pepsinognio menor.
SECREO PANCRETICA
- cinos pancreticos secretam as enzimas digestivas pancreticas.
- Grandes volumes de soluo de bicarbonato de sdio so secretados pelos ductos pequenos e
maiores que comeam nos cinos.
- Suco pancretico secretado de forma mais abundante em resposta presena de quimo nas
pores superiores do intestino delgado e as caractersticas so determinadas, at certo ponto, pelos tipos
de alimento no quimo.
* Insulina: secretada para o sangue pelas ilhotas de Langherans.

ENZIMAS DIGESTIVAS PANCRETICAS


- Mltiplas enzimas e grande quantidade de ons bicarbonato (neutralizar acidez do quimo
transportado do estmago para o duodeno).
- Protenas: tripsina (mais abundante), quimotripsina e carboxipolipeptidase.
- Tripsina e quimotripsina: hidrolisam protenas a peptdeos de tamanhos variados, sem
liberar AA individuais.
- Carboxipolipeptidase: pode clivar alguns peptdeos a AA individuais, completando a
digesto de algumas protenas.
- Carboidratos:
- Amilase pancretica: hidrolisa amidos, glicognio e outros carboidratos (exceto celulose),
para formar principalmente dissacardeos e alguns trissacardeos.
- Gorduras:
- Lipase pancretica: hidrolisa gorduras neutras a cidos graxos e monoglicerdeos.
- Colesterol esterase: hidrolisa steres de colesterol.
- Fosfolipase: cliva os cidos graxos dos fosfolipdios.
- As enzimas proteolticas s so ativadas aps serem secretadas no trato intestinal.
- Tripsinognio -> enterocinase (secretada pela mucosa intestinal, quando o quimo entra
em contato com a mucosa) -> Tripsina. Pode ser ativado autocataliticamente pela prpria tripsina formada.
- Quimotripsinognio ativado pela tripsina -> quimotripsina
- Procarboxipolipeptidade ativado pela tripsina -> carboxipolipeptidase.
SECREO DE INIBIDOR DA TRIPSINA EVITA A DIGESTO DO PRPRIO PNCREAS
- As cls que secretam enzimas proteolticas, no cino do pncreas, secretam, simultaneamente,
inibidor de tripsina.
- Formada no citoplasma das cls glandulares e inativa a tripsina ainda nas cls secretoras, nos
cinos e nos ductos do pncreas. Sendo a tripsina que ativa outras enzimas proteolticas, a cascata de
inativao fica completa.
* Pncreas lesado gravemente ou quando ocorre bloqueio do ducto -> acmulo de secreo
pancretica na rea comprometida -> efeito inibidor insuficiente -> digesto do pncreas -> pancreatite
aguda. Letal ou leva insuficincia pancretica crnica.

SECREO DE ONS BICARBONATO


- So secretados, junto com gua, pelas cls epiteliais dos ductos que se originam nos cinos.
- CO2 -> difuso para as cls a partir do sangue -> anidrase
carbnica -> combina com gua e forma H2CO3 -> H2CO3 se
converte em ons bicarbonato e hidrognio -> on bicarbonato,
por transporte ativo, associado a Na+, transportado da
membrana luminal da cl para o lmen do ducto.
- on hidrognio trocado por Na+ na membrana sangunea da
clula por processo de transporte ativo secundrio -> supre os
ons sdio transportados atravs da borda do lmen para
dentro do lmen do ducto pancretico para fornecer
neutralidade eltrica para os ons bicarbonato secretados.
- Movimento global de ons sdio e bicarbonato do sangue para
o lmen do ducto cria gradiente de presso osmtica que causa
fluxo de gua tambm para o ducto pancretico, formando
soluo de bicarbonato quase isosmtica.

REGULAO DA SECREO PANCRETICA


- Acetilcolina: liberada pelas terminaes do nervo vago parassimptico e por outros nervos
colinrgicos para o sistema nervoso entrico. (1)
- Colecistocinina: secretada pela mucosa duodenal e do jejuno superior, quando o alimento entra
no intestino delgado. (2)
- Secretina: tambm secretada pelas mucosas duodenal e jejunal, quando alimentos muito cidos
entram no intestino delgado. (3)
- 1 e 2 estimulam as cls acinares do pncreas, levando produo de grande quantidade de
enzimas digestivas pancreticas e quantidades pequenas de gua e eletrlitos vo com as enzimas.
- 3 estimula a secreo de grandes volumes de soluo aquosa de bicarbonato de sdio pelo epitlio
do ducto pancretico.
* A ao de todos os diferentes estmulos ao mesmo tempo gera um produto maior que a soma das
aes de maneira individual -> os diversos estmulos potencializam uns aos outros.
- Secretina: polipeptdeo com 27 AA.
- Presente em forma inativa (pr-secretina) nas clulas S, na mucosa do duodeno e do jejuno.
- Quimo cido no duodeno -> ativao e liberao de secretina pela mucosa duodenal para
o sangue. O constituinte verdadeiramente potente do quimo o HCl.
- Secretina -> pncreas -> secreo de soluo concentrada de bicarbonato e pequena
concentrao de ons cloreto.
- Comea a ser liberada pela mucosa do intestino delgado, quando o pH do contedo
duodenal cai abaixo de 4,5 a 5, e sua liberao aumenta bastante quando aumenta para 3.
- HCl + NaHCO3 -> NaCl + H2O + CO2
- CO2 transferido para o sangue e expirado pelos pulmes -> soluo neutra de cloreto de
sdio no duodeno.
- Neutralizao do cido -> bloqueio da atividade digestiva peptdica
- Colecistocinina (CCK): polipeptdeo com 33 AA.
- Liberado pelas cls I mucosa do duodeno e do jejuno superior, estimulado pela presena
de proteoses e peptonas (produtos da digesto parcial de protenas) e cidos graxos de cadeira longa, no
quimo que vem do estmago.
- Chega ao pncreas pela circulao sangunea.
- Provoca, principalmente, a secreo de ainda mais enzimas digestivas pancreticas pelas
cls acinares (respondendo por 70% a 80% da secreo total de enzimas digestivas pancreticas).

BILE
- Secretada pelo fgado (600~1000mL/dia).
- Importante papel na digesto de gorduras: no possui enzimas digestivas de gordura, mas ajuda a
emulsificar as grandes partculas, formando partculas diminutas cujas superfcies sero atacadas pelas
lipases do suco pancretico. Alm disso, ajuda na absoro dos produtos finais da digesto de gorduras
atravs da membrana mucosa intestinal.
- Serve como meio de excreo para diversos produtos do sangue: bilirrubina, produto final da
degradao de hemoglobina e o colesterol em excesso.
SECREO BILIAR
- Soluo inicial (grande quantidade
de cidos biliares, colesterol e outros
prod. orgnicos) secretada pelos
hepatcitos -> canalculos biliares
(originam-se entre as cls hepticas)
-> septos interlobulares -> ductos
biliares terminais -> ducto heptico e
ducto biliar comum -> bile flui
diretamente para o duodeno ou
armazenada na vescula biliar, onde
chega pelo ducto cstico.

- No percurso pelos ductos biliares,


uma 2 poro da secreo heptica
acrescentada, estimulada,
especialmente, pela secretina.
Contm ons sdio e bicarbonato
(secretados pelas cls epiteliais que
revestem os canalculos e ductos)
que iro suplementar a secreo
pancretica p/ neutralizar a acidez.

- Armazenamento e concentrao da bile na vescula biliar: Volume mximo da vescula: 30~60ml.


Entretanto, at 12hs de secreo da bile (~450ml) podem ser armazenadas porque gua e outros eletrlitos
so continuamente absorvidos pela mucosa da vescula biliar, concentrando os outros constituintes. Essa
absoro se d pelo transporte ativo de sdio atravs do epitlio da vescula, seguido pela absoro
secundria de ons cloreto, gua, etc.
- Composio da bile:
- Sais biliares -> substs mais abundantes.
- Devido absoro de gua e eletrlitos e no
reabsoro dos outros constituintes (sais, colesterol),
ocorre a concentrao da bile na vescula biliar.
- Esvaziamento da vescula: Estmulo pelo incio da
digesto no trato GI superior e pela entrada de
alimentos gordurosos no duodeno. D-se por
contraes rtmicas da parede da vescula, com o
relaxamento simultneo do esfncter de Oddi, que
controla a entrada do ducto biliar comum no duodeno.
- Estmulo mais potente: CCK. Presena de alimentos
gordurosos no duodeno causa sua liberao no
sangue, pela mucosa duodenal.
- Estmulo menor: fibras nervosas secretoras de
acetilcolina, tanto no nervo vago como no sistema
nervoso entrico, os mesmos que promovem motilidade e secreo em outras partes do trato GI superior.
- Quando h presena significativa de gorduras, se esvazia de forma completa (~1h). Caso no, esvazia
lentamente.
- Funo dos sais biliares na digesto/absoro de gordura:
- Colesterol (da dieta ou sintetizado nas cls hepticas) -> convertido em cido clico ou cido
quenodesoxiclico (qtdes aprox. iguais) -> cidos se combinam com glicina (+ parte) e taurina (- parte) ->
formao de cidos biliares glico ou tauroconjugados -> sais desses cidos (principalmente de sdio) so
secretados para a bile.
- Ao detergente: diminuio da tenso superficial das gotas de gordura, permitindo que a agitao
no trato intestinal as quebre em partculas menores (funo emulsificante).
- Ajudam na absoro de cidos graxos, monoglicerdeos, colesterol e outros lipdios: formao de
complexos fsicos com os lipdios (micelas), semissolveis no quimo devido s cargas eltricas dos sais
biliares, que ento carregam os lipdios para a mucosa intestinal, de onde so absorvidos pelo sangue.
- Sem a presena de sais biliares no trato intestinal, at 40% das gorduras ingeridas so perdidas nas
fezes, resultando em dficit metablico.

- Infeco crnica de baixo grau -> pode causar


absoro excessiva de gua e sais biliares, mas
no de colesterol, aumentando sua concentrao
e causando precipitao (1 formando pequenos
cristais na superfcie da mucosa inflamada, depois
crescem para formar os grandes clculos biliares).

SECREES DO INTESTINO DELGADO


- Glndulas de Brunner: glndulas mucosas compostas, localizadas na parede dos primeiros
centmetros de duodeno. Secretam grande quantidade de muco alcalino em resposta a:
- Estmulos tteis ou irritativos na mucosa duodenal.
- Estimulao vagal, que causa maior secreo, concomitante ao aumento da secreo gstri.
- Hormnios gastrointestinais, em especial a secretina.
- Funo do muco: proteger a parede duodenal da digesto pelo suco gstrico. Contm tambm
ons bicarbonato, que se juntam aos ons da secreo pancretica e da bile heptica, neutralizando o HCl
que entra no duodeno vindo do estmago.
- So inibidas por estimulao simptica. Assim, pessoas tensas fazem com que o bulbo duodenal
fique desprotegido, muitas vezes resultando em lceras ppticas nessa rea do trato GI.
- Secreo de sucos digestivos intestinais pelas criptas de Lieberkuhn
- Criptas entre as vilosidades intestinais, formando
depresses na superfcie do intestino delgado.
- Superfcies das criptas e vilosidades so cobertas
por epitlio composto por:
- cls caliciformes: secretam muco que lubrifica
e protege as superfcies intestinais.
- entercitos: nas criptas, secretam gua e
eletrlitos. Sobre as superfcies das vilosidades
adjacentes, absorvem gua, eletrlitos e produtos
finais da digesto.
- Secrees intestinais so formadas pelos
entercitos das criptas (~1800mL/dia). So
semelhantes ao lquido extracelular e tem pH
ligeiramente alcalino (7,5 a 8,0).
- Esse fluxo criptas -> vilosidades proporciona
veculo aquoso para absoro de substncias do
quimo, em contato com as vilosidades. Ou seja, a funo primria do intestino delgado a de absorver
nutrientes e seus produtos digestivos para o sangue.
- Mecanismo de secreo do lquido aquoso: secreo ativa de ons cloreto e bicarbonato nas
criptas, gerando diferena de potencial eltrico de ons sdio, atravs da membrana e para o lquido
secretado. Em conjunto, causam o fluxo osmtico da gua.

- Enzimas digestivas na secreo do intestino delgado


- Entercitos da mucosa (os que recobrem as vilosidades): possuem enzimas digestivas que digerem
alimentos especficos enquanto eles so absorvidos atravs do epitlio.
- Diversas peptidases para hidrlise de pequenos peptdeos a AA.
- sucrase, maltase, isomaltase e lactase para hidrlise de dissacardeos a monossacardeos.
- Pequenas qtdes de lipase intestinal para clivagem das gorduras neutras em glicerol e cidos
graxos.
- Cls epiteliais mais profundas nas criptas -> mitose contnua -> migram da base para a ponta das
vilosidades para reconstituir o epitlio e formar novas enzimas. Tempo de vida ~5 dias.

- Regulao da secreo do intestino delgado:


- Reflexos nervosos entricos locais, em especial reflexos desencadeados por estmulos tteis ou
irritantes do quimo sobre os intestinos.

SECREO DE MUCO PELO INTESTINO GROSSO


- A mucosa do intestino grosso possui muitas criptas de Lieberkuhn, mas no possuem vilos. As cls
epiteliais quase no secretam enzimas. Assim, so cls mucosas que secretam apenas muco.
- Muco: Quantidade moderada de ons bicarbonato.
- Protege a parede intestinal contra escoriaes e a intensa atividade bacteriana que ocorre
nas fezes. / Proporciona meio adesivo para material fecal. / Seu pH alcalino constitui barreira para impedir
que os cidos formados nas fezes ataquem a parede intestinal.
- Infeco bacteriana: alm do muco normal, h secreo intensa de gua e eletrlitos, que ajudam
a diluir os fatores irritantes e causam movimento rpido das fezes, eliminando os fatores. (diarreia)
HIDRLISE DE CARBOIDRATOS
- Reintroduo dos ons hidrognio e hidroxila, obtidos na gua, nos polissacardeos, separando os
monossacardeos.

HIDRLISE DE GORDURAS
- Maior parte das gorduras ingeridas consistem em triglicerdeos (gorduras neutras), que so 3
molcs de cido graxo condensadas em uma molc de glicerol. Remoo de 3 molcs de gua.
- Digesto: reinsero de gua na molc do triglicerdeo, separando cs graxos do glicerol.

HIDRLISE DE PROTENAS
- Protenas so formadas por ligao peptdica. Em cada ligao, hidroxila foi removida de um AA e
hidrognio do outro AA. Assim, na digesto, as enzimas quebram essa ligao e reinserem hidroxila e H.

DIGESTO DOS CARBOIDRATOS


3

- 3 fontes principais: sacarose (acar de


cana), lactose (dissacardeo do leite) e
amidos (amilose + amilopectina, grandes
polissacardeos de reserva de plantas,
principalmente).
- Outros: amilose, glicognio, lcool, cido
lctico, pectinas, dextrinas, etc.
- Celulose: polmero de glicose conectado por ligaes beta-1,4 que no podem ser digeridas pelas
enzimas dos mamferos. No entanto, enzimas provenientes da ao de bactrias no clon podem degradar
essas fibras (pouco significativo).
- Hidrlise intraluminal do amido em oligossacardeos pelas enzimas salivares e pancreticas:
- Alimento na boca -> ptialina (alfa-amilase), secretada principalmente pelas partidas ->
Hidrolisa o amido em maltose e outros pequenos polmeros de glicose.
- Pelo pouco tempo que permanece na boca, cerca de 5% do amido hidrolisado.
- Devido continuao do processo, a atividade da amilase bloqueada pelo cido das
secrees gstricas (inativa em pH abaixo de 4). Mas at ocorrer a completa mistura, at 30% a 40% do
amido ter sido hidrolisado para formar maltose.
- Amilase pancretica recomea a digesto no duodeno.
- Alfa amilase no cliva ligaes a-1,4 terminais, ligaes a-1,6 ou ligaes a-1,4 adjacentes
s ligaes a-,16. Assim, a glicose no um produto da degradao do amido.
- Entercitos que revestem as vilosidades: lactase, sacarase, maltase e a-dextrinase (ou
isomaltase), que clivam lactose, sacarose e maltose, mais outros pequenos polmeros de glicose, nos seus
monossacardeos constituintes. Essas enzimas ficam localizadas nos entercitos que forram a borda em
escova (membrana da borda em escova, MBE) das microvilosidades intestinais, de maneira que os
dissacardeos so digeridos quando em contato.
- Sacarase + isomaltase = nica que consegue separar as ramificaes que apresentam a ligao a-
1,6 das dextrinas a-limitante.
- O pico de distribuio e atividade ocorre na regio proximal do jejuno. Uma atividade
consideravelmente menor existe no duodeno e na regio distal do leo, e nenhuma atividade enzimtica
encontrada no intestino grosso.
- Na dieta comum, com muito mais amidos que todos os outros carboidratos, a glicose representa
80% dos produtos finais da digesto de carboidratos, enquanto a frao de frutose ou galactose no passa
de 10%.
DIGESTO DE PROTENAS
- A digesto de protenas influenciada pela
composio dos aminocidos da protena, pela fonte
proteica e pelo tipo de processamento a que o
alimento foi submetido. Assim, protenas ricas em
prolina e hidroxiprolina sofrem digesto menos
completa. O cozimento, o armazenamento e a
desidratao reduzem a plenitude da digesto. Em
geral, protenas de origem animal so mais bem
digeridas que protenas de origem vegetal.

- Estmago: pepsina. Atividade mxima com pH entre 1,8 e 3,5. Estmago responsvel por 10% a
20% da digesto total das protenas. Em especial, digere o colgeno, que pouco afetado por outras
enzimas. O colgeno constituinte importante do tecido conjuntivo celular das carnes, ou seja, para que
outras enzimas digiram outras protenas da carne, necessrio degradar o colgeno primeiro.
- Grande parte da digesto das protenas ocorre no intestino delgado superior, duodeno e jejuno,
sob influncia das enzimas proteolticas da secreo pancretica.
- A tripsina, a quimiotripsina e a elastase so endopeptidases com afinidade a ligaes peptdicas
adjacentes a determinados aminocidos no interior da cadeia, formando, assim, oligopeptdeos compostos
de dois a seis aminocidos.
- As exopeptidases carboxipeptidases A e B hidrolisam ligaes peptdicas adjacentes regio
carboxiterminal, resultando, assim, na liberao individual de cada aminocido.
- A ao coordenada dessas proteases pancreticas converte aproximadamente 70% do nitrognio
amnico em oligopeptdeos e aproximadamente 30% em aminocidos livres.
- A maioria digerida at dipeptdeos e tripeptdeos.
- No lmen intestinal, o ltimo estgio da digesto feito pelo entercitos que revestem as
microvilosidades do intestino delgado.
- Nas membranas de cada microvilosidade -> mltiplas peptidases.
- Aminopolipeptidase e diversas dipeptidases: mais importantes. Continuam a hidrlise da maioria
dos polipeptdeos remanescentes em tripeptdeos e dipeptdeos e de uns poucos AA.
- Existem tambm peptidases no citosol dos entercitos. Di/tripeptdeos entram nos entercitos,
sofrem hidrlise e os AA resultantes vo para o sangue.

DIGESTO DE GORDURAS
- Gorduras mais abundantes: triglicerdeos (gorduras neutras).
- Lecitina: importante para emulsificao das gorduras.
- Enzimas lipases so hidrossolveis e podem atacar os glbulos de gordura apenas em suas
superfcies, da a importncia da secreo biliar e emulsificao.
- Lipase pancretica: digere os triglicerdeos, resultando em cidos graxos e 2-monoglicerdeos.
- Micelas: cada molcula de sal biliar composta por ncleo esterol, muito lipossolvel e grupo polar
muito hidrossolvel. O ncleo esterol envolve os produtos da digesto das gorduras, formando pequeno
glbulo de gordura, no meio da micela resultante, com os grupos polares dos sais biliares se projetando
para fora, para cobrir a superfcie da micela. Como esses grupos polares tm carga negativa, eles permitem
que todo o glbulo de micela se dissolva na gua dos lquidos digestivos e permanea em soluo estvel
at a absoro de gordura.
- As micelas de sais biliares tambm so meios de transporte, carreando monoglicerdeos e cidos
graxos, que sero posteriormente absorvidos pelo sangue.
PROBLEMA 2.1
Regulao do metabolismo
1) Sinais intracelulares: a velocidade de uma via pode ser influenciada pela disponibilidade de
substratos, inibio ocasionada pelos produtos, alteraes nos nveis de ativadores/inibidores.
2) Sinais intercelulares: comunicao pode ser mediada pelo contato entre suas superfcies, pela
formao de junes comunicantes, sinalizao qumica por hormnios/neurotransmissores (via mais
importante no metabolismo energtico).
3) Sistemas de segundos mensageiros: Molculas que intervm entre o mensageiro original
(neurotransmissor/hormnio) e o efeito final dentro da clula.
- Sistema da adenilato-ciclase:
- Adenilato-ciclase: enzima ligada membrana que converte ATP em AMPc.
- A ligao de neurotransmissor/hormnio a um receptor (como os adrenrgicos) faz
com que esse interaja com protenas reguladoras dependentes de GTP. O receptor ativado faz com que ocorra
a ativao das protenas G. Ocorre troca de GDP por GTP. Aps isso, a protena G se dissocia, e sua subunidade
alfa se move para o receptor na adenilato-ciclase, causando sua ativao, que converte ATP em AMPc. Aps
converso, ocorre o caminho inverso.
- O AMPc formado ativa a protena-cinase A ligando-se s suas duas subunidades
reguladoras, liberando as subunidades catalticas ativas. Essas subunidades ativas catalisam a transferncia de
fosfato do ATP em protenas que so substrato dessa enzima. As protenas fosforiladas podem atuam
diretamente sobre canais inicos da clula ou podem se tornar enzimas ativadas ou inibidas.
- Enzimas do tipo protena-fosfatase clivam hidroliticamente steres de fosfato, fazendo
com que o fosfato adicionado s protenas seja removido, garantindo que mudanas enzimticas induzidas
pela fosforilao no sejam permanentes.
- A enzima fosfodiesterase rapidamente hidrolisa o AMPc a 5-AMP. Como o 5-AMP no
uma molcula de sinalizao intracelular, a remoo do sinal extracelular (neurotransmissor/hormnio)
determina a parada da sinalizao na clula.

GLICLISE VISO GERAL


- Quebra da glicose com o objetivo de fornecer energia (ATP) e intermedirios para outras vias
metablicas. A glicose o centro do metabolismo de carboidratos.
- O piruvato o produto final da gliclise nas clulas que possuem mitocndrias e um fornecimento
adequado de oxignio.
- O oxignio necessrio para a reoxidao de NADH formado durante a oxidao do gliceraldedo-3-
fosfato.
- Prepara as condies necessrias para a descarboxilao oxidativa do piruvato a acetil-CoA.
- Sem a presena de oxignio, a glicose convertida em piruvato, que reduzido pelo NADH para
formar lactato. [Tecidos que no apresentam mitocndrias (ex.: eritrcitos) ou em clulas que estejam com
suprimento pouco efetivo de oxignio].

TRANSPORTE DE GLICOSE PARA DENTRO DAS CLULAS


- Transporte por difuso facilitada, independente de Na+:
- Mediado por uma famlia de pelo menos 14 transportadores de glicose (GLUT-1 a GLUT-14),
que ocorrem nas membranas celulares em dois estados conformacionais. Glicose liga ao transportador, que
sofre alterao em sua conformao e transporta a glicose atravs da membrana.
- Especificidade tecidual da expresso gnica dos GLUTs: padro de expresso com
especificidade tecidual. Ex.: GLUT-3 o principal transportador nos neurnios / GLUT-1 abundante nos
eritrcitos e no encfalo / GLUT-4 abundante no tecido adiposo e no msculo esqueltico.
- Funes especializadas das isoformas de GLUT: o movimento da glicose segue um gradiente
de concentrao (+ para -). GLUT-1-3-4 envolvidos principalmente na captao a partir do sangue / GLUT-2
(fgado, rins e clulas beta do pncreas) pode transportar tanto para dentro das cls (quando glicose est alta
no sangue) quanto para o sangue (quando glicose est baixa no sangue) / GLUT-5 principal transportador para
a frutose no int. delgado e testculos. / GLUT-7 medeia fluxo de glicose atravs da membrana do Ret end.
- Sistema de co-transporte monossacardeo-Na+:
- Processo ativo que transporta glicose contra o gradiente de concentrao.
- Movimento da glicose est acoplado ao gradiente de concentrao de Na+, o qual
transportado concomitantemente glicose para o interior da clula. Ocorre nas cls epiteliais do intestino,
tbulos renais e plexo coroide.

REAES DA GLICLISE
- 10 etapas: as 5 primeiras (fase preparatria) e 5 ltimas (fase de pagamento)
- Fase preparatria: Nessas reaes, a glicose inicialmente fosforilada no grupo hidroxil ligado
ao C-6 (etapa 1). A D-glicose-6-fosfato assim formada convertida a D-frutose-6-fosfato (etapa 2), a qual
novamente fosforilada, desta vez em C-1, para formar D-frutose-1,6-bifosfato (etapa 3). Nas duas reaes de
fosforilao, o ATP o doador de grupos fosforil. A frutose-1,6-bifosfato dividida em duas molculas de trs
carbonos, a di-hidroxiacetona-fosfato e o gliceraldedo-3-fosfato (etapa 4); essa a etapa de lise que d
nome via. A di-hidroxiacetona-fosfato isomerizada a uma segunda molcula de gliceraldedo-3-fosfato
(etapa 5), finalizando a primeira fase da gliclise.
- Duas molculas de ATP so consumidas antes da clivagem da glicose em duas partes de trs
carbonos; haver depois um bom retorno para esse investimento.
- Resumindo: na fase preparatria da gliclise, a energia do ATP consumida, aumentando o
contedo de energia livre dos intermedirios, e as cadeias de carbono de todas as hexoses metabolizadas so
convertidas a um produto comum, o gliceraldedo-3-fosfato.
- Fase de pagamento: O ganho de energia provm da fase de pagamento da gliclise. Cada
molcula de gliceradedo-3-fosfato oxidada e fosforilada por fosfato inorgnico (no por ATP) para formar
1,3-bifosfoglicerato (etapa 6). Ocorre liberao de energia quando as duas molculas de 1,3-bifosfoglicerato
so convertidas a duas molculas de piruvato (etapas 7 a 10).
- Grande parte dessa energia conservada pela fosforilao acoplada de quatro molculas de
ADP a ATP. O rendimento lquido so duas molculas de ATP por molcula de glicose utilizada, j que duas
molculas de ATP foram consumidas na fase preparatria. A energia tambm conservada na fase de
pagamento com a formao de duas molculas do transportador de eltrons NADH por molcula de glicose.
- Nas reaes seguintes da gliclise, trs tipos de transformaes qumicas so particularmente
notveis: (1) a degradao do esqueleto carbnico da glicose para produzir piruvato; (2) a fosforilao de ADP
a ATP pelos compostos com alto potencial de transferncia de grupos fosforil, formados durante a gliclise; e
(3) a transferncia de um on hidreto para o NAD+, formando NADH.

1) FOSFORILAO DA GLICOSE
- Molculas fosfoliradas de glicdeos no atravessam
facilmente as membranas celulares (no h
carreadores especficos na membrana para elas), e
so muito polares para difundir atravs da membrana.
- A fosforilao irreversvel da glicose retm
efetivamente o glicdeo na forma glicose-6-fosfato
citoslica, assegurando seu posterior metabolismo na
clula.
- Cinases: enzimas que catalisam a transferncia do grupo fosforil terminal do ATP a um aceptor nucleoflico.
- A hexocinase, como muitas outras cinases, requer Mg2+ para sua atividade. O Mg2+ protege as cargas
negativas do grupo fosforil do ATP, tornando o tomo de fsforo terminal um alvo mais fcil para o ataque
nucleoflico por um grupo -OH da glicose. Fosforila a glicose na maior parte dos tecidos.
- Apresenta ampla especificidade quanto ao substrato (fosforila diversas hexoses alm da glicose).
- inibida pelo produto da reao, glicose-6-fosfato, que se acumula quando a metabolizao dessa
hexose-fosfato est reduzida.
- Apresenta baixo Km (Km numericamente igual concentrao do substrato na qual a velocidade
da reao igual metade da Vmx). Ou seja, h uma alta afinidade para a glicose, pois uma baixa
concentrao de substrato necessria para atingir a metade da saturao da enzima, permitindo fosforilao
eficiente e o metabolismo subsequente da glicose ainda que em baixas concentraes teciduais.
- Apresenta baixa Vmx para a glicose, portanto, no pode reter fosfato celular na forma de hexoses
fosforiladas, ou fosforilar maior quantidade de glicdeos que a clula pode utilizar.
- Glicocinase: isoenzimas da hexocinase (hexocinase D, ou do tipo IV). Clulas do parnquima heptico e nas
clulas das ilhotas no pncreas.
- Nas clulas beta: funciona como sensor de glicose, determinando limiar para secreo de insulina.
- No fgado: facilita a fosforilao da glicose durante uma hiperglicemia.
- Cintica:
- Apresenta Km muito maior: funciona apenas quando a concentrao intracelular de glicose
no hepatcito est elevada (ps-prandial de uma refeio rica em carboidratos, quando muita glicose levada
ao fgado via veia porta).
- Alta Vmx: fgado remove eficientemente o excesso de glicose fornecido pela circulao porta.
Impede que muita glicose chegue circulao sistmica, minimizando a hiperglicemia no p. absortivo. (Papel
da GLUT-2 no equilbrio entre os dois lados da membrana do hepatcito).
- Regulao por frutose-6-fosfato e glicose:
- Inibida indiretamente pela frutose-6-fosfato (que est em equilbrio com glicose-6-fosfato) e
estimulada diretamente pela glicose.
- Presena no ncleo do hepatcito de protena reguladora da glicocinase.
- Presena de frutose-6-fosfato >> glicocinase translocada para o ncleo >> se liga fortemente
protena reguladora >> inativao. (a protena ancora a enzima dentro do ncleo, onde ela fica segregada
das outras enzimas da gliclise no citosol)
- Aumento de glicose no sangue (e tambm no hepatcito por conta da GLUT-2) >> induz
liberao da glicocinase de sua protena >> retorna ao citosol >> fosforila glicose dando glicose-6-fosfato.
- Diminuindo glicose, frutose-6-fosfato reinicia o processo.
- Regulao pela insulina:
- Aumento de glicose no sangue >> clulas beta liberam insulina na circulao porta.
- Metade da insulina recm-secretada se liga ao fgado na 1 passagem sobre ele.
- Insulina promove transcrio do gene da glicocinase, aumentando a protena e sua atividade.
2) ISOMERIZAO DA GLICOSE-6-FOSFATO
- Enzima fosfo-hexose-isomerase (fosfoglicose-
isomerase) catalisa a isomerizao reversvel
(facilmente) da glicose-6-fosfato (aldose) a frutose-6-
fosfato (cetose).
- O prton inicialmente em C-2 torna-se mais
facilmente removvel pela retirada do eltron do
grupo carbonil adjacente e pelos grupos hidroxilas
vizinhos. Aps sua
transferncia do C-2 para o
resduo de Glu do stio ativo (c
fraco), o prton livremente
trocado com a soluo ao
redor. (O prton removido em
C-2 no necessariamente o
mesmo adicionado ao C-1 na
etapa 3).
3) FOSFORILAO DA FRUTOSE-6-FOSFATO
- A enzima fosfofrutocinase (PFK-1) catalisa a transferncia
de um grupo fosforil do ATP para a frutose-6-fosfato,
formando frutose-1,6-bifosfato. Degradao ocorre pela
frutose-1,6-bifosfatase (FBPase-1).
*** A reao com PFK-1 essencialmente irreversvel em
condies celulares, e essa a primeira etapa
comprometida da via glicoltica (a glicose-6-fosfato e a
frutose-6-fosfato tm outros destinos possveis, mas a
frutose-1,6-bifosfato direcionada para a gliclise).
*** O mais importante ponto de controle e passo limitante
da velocidade da gliclise.
- PFK-1 controlada pela concentrao dos substratos (ATP,
Frutose-6-fosfato) e por substncias reguladoras.
- Regulao pelos nveis energticos dentro da clula: PFK-1 inibida alostericamente por nveis altos de ATP
(indica abundncia de compostos energticos) e de citrato (ciclo dos cidos tricarboxlicos). Por outro lado,
ativada por nveis altos de AMP (sinaliza depleo das reservas energticas celulares).
- Regulao pela frutose-2,6-bifosfato: o mais potente ativador da PFK-1. Tambm atua como inibidor da
FBPase-1, da gliconeognese.
- Suas aes recprocas sobre gliclise e gliconeognese asseguram que as vias no estejam
completamente ativas ao mesmo tempo (caso contrrio, a gliclise formaria glicose->piruvato, e logo aps a
gliconeognese formaria piruvato->glicose).
- Quando a frutose-2,6-bifosfato se liga ao seu stio alostrico na PFK-1, ela aumenta a afinidade dessa
enzima pelo seu substrato, frutose-6-fosfato, e reduz a afinidade pelos inibidores alostricos ATP e citrato.
- Em concentraes fisiolgicas de seus substratos, a PFK-1 est praticamente inativa na ausncia da
frutose-2,6-bifosfato, que tem efeito oposto sobre a FBPase-1: ela reduz a afinidade pelo seu substrato,
reduzindo a gliconeognese.
- Ela se forma pela fosforilao da frutose-6-fosfato, catalisada pela fosfofrutocinase-2 (PFK-2) e
degradada pela frutose-2,6-bifosfatase (FBPase-2).
- PFK-2 e FBPase-2 so duas atividades enzimticas separadas de uma nica protena bifuncional.
- Durante estado alimentado: insulina e glucagon causa aumento da frutose-2,6-bifosfato e da
velocidade da gliclise no fgado (sinalizador intracelular da abundncia de glicose).
- Durante o jejum: glucagon e insulina diminuem a concentrao de frutose-2,6-bifosfato,
diminuindo a gliclise e aumentando a gliconeognese.
4) CLIVAGEM DA FRUTOSE-1,6-BIFOSFATO
- A enzima frutose-1,6-bifosfato-aldolase, muitas vezes
chamada simplesmente de aldolase A, catalisa uma
condensao aldlica reversvel. A frutose-1,6-bifosfato
clivada para a formao de duas trioses-fosfato
diferentes, a aldose gliceraldedo-3-fosfato e a cetose di-
hidroxiacetona-fosfato.
- Embora a reao da aldolase tenha uma variao da
energia livre padro fortemente positiva no sentido de
clivar a frutose-1,6-bifosfato, nas baixas concentraes
dos reagentes presentes na clula a variao real da
energia livre pequena, e a reao da aldolase
prontamente reversvel, alm de no regulada.
- Aldolase B, no fgado e no rim, tambm cliva a F16BP e funciona no metabolismo da frutose da dieta.

5) ISOMERIZAO (INTERCONVERSO) DA DI-HIDROXIACETONA-FOSFATO


- Apenas uma das duas trioses-fosfato formada pela
aldolase, o gliceraldedo-3-fosfato, pode ser
diretamente degradada nas etapas subsequentes da
gliclise (cetona no oxida). O outro produto, a di-
hidroxiacetona-fosfato, rpida e reversivelmente
convertida a gliceraldedo-3-fosfato pela quinta enzima
da sequncia
glicoltica, a triose-fosfato-isomerase.
- Essa isomerizao resulta na produo lquida de duas molculas de gliceraldedo-3-fosfato pelos produtos
da clivagem da F16BP.
- O mecanismo de reao similar ao da reao promovida pela fosfo-hexose-isomerase na etapa 2 da gliclise
- Essa reao completa a fase preparatria da gliclise. A molcula de hexose foi fosforilada em C-1 e C-6 e
ento clivada para formar duas molculas de gliceraldedo-3-fosfato.

** A fase de pagamento da gliclise inclui as etapas de fosforilao que conservam energia, nas quais parte
da energia qumica da molcula da glicose conservada na forma de ATP e NADH. Lembre-se de que uma
molcula de glicose rende duas molculas de gliceraldedo-3-fosfato, e as duas metades da molcula de glicose
seguem a mesma via na segunda fase da gliclise. A converso das duas molculas de gliceraldedo-3-fosfato
a duas molculas de piruvato acompanhada pela formao de quatro molculas de ATP a partir de ADP. No
entanto, o rendimento lquido de ATP por molcula de glicose consumida de apenas dois, j que dois ATP
foram consumidos na fase preparatria da gliclise para fosforilar as duas extremidades da molcula da
hexose.

6) OXIDAO DO GLICERALDEDO-3-FOSFATO
- A primeira etapa da fase de pagamento a oxidao do
gliceraldedo-3-fosfato a 1,3-bifosfoglicerato, catalisada
pela enzima gliceraldedo-3-fosfato-desidrogenase.
- Como o NAD+ limitado na clula, o NADH produzido
deve ser reoxidado a NAD+ para que a gliclise continue.
- Oxidao do NADH: converso ligada ao NADH de
piruvato em lactato ou oxidao do NADH via cadeira
respiratria.
- Sntese do 1,3-bifosfoglicerato: Oxidao do grupo aldedo do gliceraldedo-3-fosfato a um grupo carboxila
acoplada ligao de fosfato inorgnico a esse grupo carboxila. O grupo fosfato de alta energia no carbono 1
do 1,3-BPG conserva boa parte da energia livre produzida pela oxidao. Essa energia fora/impulsiona a
sntese de ATP na prxima etapa.
- Envenenamento por arsnico (uma das vias): Impede a produo lquida de ATP e NADH durante a gliclise,
sem a inibio da via em si, ao competir com o fosfato inorgnico como substrato da gliceraldedo-3-fosfato-
desidrogenase, formando um complexo que espontaneamente hidrolisa, para produzir 3-fosfoglicerato. Esse
pulo da sntese e da desfosforilao do 1,3-BPG priva a clula da energia normalmente obtida.
- Sntese de 2,3-BPG nos eritrcitos: Parte do 1,3-BPG convertida em 2,3-BPG por ao da bifosfoglicerato-
mutase. pouco encontrado na maioria das clulas, mas abundante em eritrcitos. 2,3-BPG hidrolisado por
uma fosfatase, dando 3-fosfoglicerato (tambm intermedirio da gliclise)
- Estabiliza a conformao tencionada da desoxiemoglobina.
- Diminui a afinidade da hemoglobina pelo oxignio, fazendo com que o O2 seja eficientemente
liberado nas presses parciais encontradas nos tecidos.
- Em grandes altitudes, alm do aumento do nmero de eritrcitos, h tambm aumento da
concentrao de 2,3-BPG nos indivduos como mecanismo de adaptao.

7) SNTESE DE 3-FOSFOGLICERATO, COM PRODUO DE ATP


- A enzima fosfoglicerato-cinase transfere o grupo fosforil de alta
energia do grupo carboxil do 1,3-bifosfoglicerato para o ADP, formando
ATP e 3-fosfoglicerato.
- A fosfoglicerato-cinase tem esse nome devido reao inversa, na
qual ela transfere um grupo fosforil do ATP para o 3-fosfoglicerato
(reao na gliconeognese).
- Uma vez que duas molculas de 1,3-BPG so produzidas por molcula
de glicose, essa reao repe os 2 ATPs consumidos na formao da
glicose-6-fosfato (reao 1) e frutose-1,6-bifosfato (reao 3).
- As etapas 6 e 7 da gliclise constituem um processo de acoplamento
de energia em que 1,3-bifosfoglicerato um intermedirio comum; ele
formado na primeira reao e seu grupo acil-fosfato transferido ao
ADP na segunda reao.
- O resultado do acoplamento dessas reaes, ambas reversveis em
condies celulares, que a energia liberada da oxidao de um aldedo
a um grupo carboxilato conservada pela formao acoplada de ATP a
partir de ADP e Pi.
- A formao de ATP pela transferncia do grupo fosforil de um
substrato, como o 1,3-bifosfoglicerato, chamada de fosforilao no
nvel do substrato, para distinguir esse mecanismo daquele da fosforilao ligada respirao.
- As fosforilaes no nvel do substrato envolvem enzimas solveis e intermedirios qumicos (nesse
caso, 1,3-bifosfoglicerato).
- As fosforilaes ligadas respirao, por outro lado, envolvem enzimas ligadas membrana e
gradientes transmembrana de prtons.

8) CONVERSO DE 3-FOSFOGLICERATO PARA 2-FOSFOGLICERATO


- A enzima fosfoglicerato-mutase catalisa o deslocamento
reversvel do grupo fosforil entre C-2 e C-3 do glicerato; Mg2+
essencialpara essa reao.
9) DESIDRATAO DO 2-FOSFOGLICERATO
- Na segunda reao glicoltica que gera um composto com alto
potencial de transferncia de grupamento fosforil (a primeira foi a
etapa 6), a enolase promove a remoo reversvel de uma
molcula de gua do 2-fosfoglicerato para gerar fosfoenolpiruvato
(PEP), que contm um enol fosfato de alta energia.

- O mecanismo da reao da enolase envolve um intermedirio enlico estabilizado por Mg2+. A reao
converte um composto com relativamente baixo potencial de transferncia de grupo fosforil para um com
alto potencial de transferncia de grupo fosforil:
- (O G para a hidrlise de 2-fosfoglicerato -17,6 kJ/mol).
- (O G para a hidrlise de PEP -61,9 kJ/mol).

10) FORMAO DO PIRUVATO, COM PRODUO DE ATP


-

- A ltima etapa na gliclise a transferncia (irreversvel) do grupo


fosforil do fosfoenolpiruvato ao ADP, catalisada pela piruvato-cinase,
que exige K+ e Mg2+ ou Mn2+.
- Nesta fosforilao no nvel do substrato, o piruvato resultante
aparece inicialmente em sua forma enlica (muito instvel), depois
tautomeriza de modo rpido e no enzimtico sua forma cetnica,
que predomina em pH 7,0.
- Regulao por proao: No fgado, a piruvato-cinase ativada pela
frutose-1,6-bifosfato (que produto da reao da fosfofrutocinase).
Tem o efeito de unir as atividades das duas cinases: atividade da
fosfofrutocinase resulta em nveis de frutose-1,6-bifosfato,
ativando a piruvato-cinase.
- Modulao covalente da piruvato-cinase: Fosforilao por
protena-cinase dependente de AMPc -> inativao da piruvato-
cinase.
- glicose no sangue -> glucagon -> induo para dos nveis
intracelulares de AMPc -> fosforilao -> inativao da piruvato-
cinase.
- Assim, o fosfoenolpiruvato no prossegue na via glicoltica e
entra na via da gliconeognese, explicando a inibio da gliclise e a
estimulao da gliconeognese observadas em resposta ao glucagon.
- Desfosforilao por fosfoprotena-fosfatase -> ativao enzima.
- Deficincia de piruvato-cinase: Eritrcito normal no possui mitocndrias
-> completamente dependente da gliclise.
- ATP: satisfazer as necessidades energticas e tambm alimentar as
bombas necessrias para a manuteno da forma bicncava e flexvel
(importante para forar o caminho por capilares muito estreitos).
- A anemia observada na deficincia de enzimas glicolticas uma
consequncia da reduo da velocidade da gliclise, diminuindo prod. ATP.
- Alteraes na membrana -> mudanas no formato da clula ->
fagocitose por clulas do sistema reticuloendotelial (principalmente
macrfagos do bao) -> anemia hemoltica.
- Defeitos genticos: 95% (piruvato-cinase) / 4% (fosfoglicose-isomerase).
- As duas molculas de NADH formadas pela gliclise no citosol so, em condies aerbias, reoxidadas a NAD+
pela transferncia de seus eltrons para a cadeia de transporte de eltrons, que em clulas eucariticas est
localizada na mitocndria. A cadeia de transporte de eltrons conduz esses eltrons para o seu destino final,
o O2.
- A transferncia de eltrons do NADH para o O2 na mitocndria fornece a energia para a sntese de ATP pela
fosforilao ligada respirao.
- No processo glicoltico em geral, uma molcula de glicose convertida a duas molculas de piruvato (a via
do carbono). Duas molculas de ADP e duas de Pi so convertidas a duas molculas de ATP (a via dos grupos
fosforil). Quatro eltrons, na forma de ons hidreto, so transferidos de duas molculas de gliceraldedo-3-
fosfato para duas de NAD+ (a via dos eltrons).

FERMENTAO LCTICA
- Em condies aerbias, o piruvato formado na etapa final da gliclise oxidado a acetato (acetil-
CoA), que entra no ciclo do cido ctrico e oxidado a CO2 e H2O. O NADH formado pela desidrogenao do
gliceraldedo-3-fosfato finalmente reoxidado a NAD+ pela transferncia de seus eltrons ao O2 na respirao
mitocondrial.
- Em condies de hipoxia (pouco oxignio) assim como no msculo esqueltico muito ativo, nos
tecidos vegetais submersos, nos tumores slidos ou nas bactrias lcticas o NADH gerado pela gliclise no
pode ser reoxidado pelo O2.
- A falha na regenerao de NAD+ deixaria a clula carente de aceptor de eltrons para a oxidao de
gliceraldedo-3-fosfato, e as reaes geradoras de energia da gliclise cessariam. Portanto, NAD+ deve ser
regenerado de outra forma.
- Quando tecidos animais no podem ser supridos com oxignio suficiente para realizar a oxidao
aerbia do piruvato e do NADH produzidos na gliclise, NAD+ regenerado a partir de NADH pela reduo do
piruvato a lactato.
- A reduo do piruvato por essa via catalisada pela lactato-
desidrogenase, que forma o ismero L do lactato em pH 7:
- Na gliclise, a desidrogenao de duas molculas de
gliceraldedo-3-fosfato derivado de cada molcula de glicose
converte duas molculas de NAD+ a duas de NADH. Como a reduo
de duas molculas de piruvato em duas de lactato regenera duas
molculas de NAD+, no ocorre variao lquida de NAD+ ou NADH.
- Ou seja, fermentao o processo que extrai energia (como
ATP) mas no consome oxignio nem varia as concentraes de NAD+
ou NADH.
- Embora a converso de glicose em lactato compreenda duas
etapas de oxidao-reduo, no ocorre variao lquida no estado de
oxidao do carbono; na glicose (C6H12O6) e no cido lctico
(C3H6O3), a relao H:C a mesma. Todavia, parte da energia da molcula da glicose extrada pela sua
converso em lactato o suficiente para dar um rendimento lquido de duas molculas de ATP para cada
molcula de glicose consumida.
- Formao de lactato no msculo: Em exerccio, a produo de NADH excede a capacidade oxidativa
da cadeia respiratria, aumentando a razo NADH/NAD+ e favorecendo a reduo de piruvato e lactato. O
lactato se acumula no msculo, diminuindo pH intracelular e causando cibras.
- O lactato liberado pelas clulas realizando gliclise anaerbia captado por outros tecidos (fgado,
corao) e oxidado de volta a piruvato. No fgado, o piruvato utilizado para sintetizar glicose
(gliconeognese), a qual devolvida para o sangue. Circulao de lactato e glicose entre tec. e fgado -> Ciclo
de Cori.
- Tecidos dependentes: em geral possuem baixa demanda de ATP, altos nveis de enzimas glicolticas e
pouco capilares (de tal modo que o O2 deve difundir por uma distncia maior para atingir as cls-alvo), ou
ainda, algum aspecto da funo celular:
- Eritrcitos no tm mitocndrias, pois o metabolismo oxidativo pode interferir na sua funo
de transporte de O2 ligado hemoglobina.
- Um pouco do cido lctico produzido pela gliclise anaerbia na pele secretado no suor,
onde atua como agente antibacteriano.
- Acidose lctica: Colapso do sistema circulatrio (infarto do miocrdio, embolia pulmonar, hemorragia
no-controlada, indivduo em choque) -> falha em levar quantidades adequadas de oxignio aos tecidos ->
prejuzo na fosforilao oxidativa e sntese de ATP.
- Para sobreviver, as clulas utilizam a gliclise anaerbia, elevando a concentrao de lactato
no plasma.
- Dbito de oxignio: aumento de oxignio necessrio para a recuperao aps um perodo em
que a sua disponibilidade foi inadequada. A medida dos nveis sanguneos de cido lctico fornece uma
deteco rpida do dbito de oxignio.

Gliclise anaerbia: 2 molc de ATP geradas p/ cada glicose convertida em 2 lactatos. No h produo ou
consumo lquido de NADH. (Medula renal, eritrcitos maduros, leuccitos, clulas do cristalino, da crnea, dos
testculos).
Gliclise aerbia: 2 molc de ATP geradas p/ cada glicose convertida em 2 piruvatos. 2 molc de NADH
produzidas p/ cada glicose. Maior parte desse NADH oxidada na cadeia de transporte de eltrons produzindo
aproximadamente 3 molc de ATP.

METABOLISMO DA FRUTOSE
- A frutose metabolizada pela converso
a gliceraldedo-3-P e diidroxiacetona-
fosfato, que so intermedirios da gliclise.
As etapas so paralelas quelas da gliclise.
- A hexocinase possui baixa afinidade pela
frutose (alto Km). Assim, quando a
concentrao de frutose intracelular
muuuuito alta, a hexocinase capaz de
fosforila-la a frutose-6-P, que entra na via
glicoltica.
- A primeira etapa do metabolismo da
frutose, assim como no da glicose, a
fosforilao. A frutocinase, a principal
cinase envolvida, fosforila frutose na
posio 1. A frutocinase tem uma Vmx alta
e rapidamente fosforila frutose quando ela
entra na clula.
- A frutose-1-fosfato formada no um intermedirio da gliclise, mas sim quebrada pela aldolase B
(frutose-1-fosfato-aldolase) a diidroxiacetona-fosfato (intermedirio da gliclise) e gliceraldedo. O
gliceraldedo , ento, fosforilado a gliceraldedo-3-P pela triose-cinase.
- Diidroxiacetona-Fosfato e gliceraldedo-3-P so intermedirios da rota glicoltica e podem atravs
dela a piruvato. Tambm podem ser convertidos a glicose por gliconeognese.
- Embora todas as aldolases (A, B, C e fetal) possam quebrar frutose-1,6-bifosfato, apenas aldolase B
tambm pode quebrar frutose-1-fosfato.
- Aldolase B a enzima limitante da velocidade do metabolismo da frutose (afinidade muito menor por
frutose-1-fosfato do que por frutose-1,6-bifosfato). Aps alta ingesto de frutose, pessoas normais acumulam
frutose-1-fosfato no fgado enquanto ela convertida lentamente a intermedirios glicolticos.
- Fosforilao da frutose:
- Muita frutose: frutose -> hexocinase -> frutose-6-fosfato.
- Normal: frutose -> frutocinase -> frutose-1-fosfato.
- Clivagem da frutose-1-fosfato:
- frutose-1-fosfato -> aldolase B ->
1 diidroxiacetona-fosfato -> triose-fosfato-isomerase
+
1 gliceraldedo -> triose-cinase e ATP

2 gliceraldedo-3-fosfatos
- Cintica do metabolismo da frutose:
- Metabolismo da frutose mais rpido que a glicose porque as
trioses formadas a partir da frutose-1-fosfato desviam da reao da
fosfofrutocinase (PFK-1) etapa marca-passo mais importante no
controle da velocidade da gliclise.
- Deficincias no metabolismo da frutose:
- Condio benigna: deficincia de frutocinase.
- Condio grave: deficincia de aldolase B (intolerncia
hereditria frutose).
- Frutose-1-fosfato se acumula -> nveis de ATP e P inorgnico -
> adenina convertida em cido rico, causando hiperuricemia -> afeta a
gliconeognese (causando hipoglicemia com vmitos) e a sntese de
protenas ( dos fatores de coagulao sangunea e outros) -> falncia
heptica e morte.
- Inibio da gliconeognese e glicogenlise.
- Converso da manose em frutose-6-fosfato:
- Manose um epmero (tipo de ismero) em C-2 da glicose
(diferena na posio do grupo hidroxila no carbono 2). Importante
componente de glicoprotenas.
- Manose -> hexocinase manose-6-fosfato fosfomanose-
isomerase frutose-6-fosfato.
- H pouca manose nos carboidratos da dieta.
- Converso de frutose em glicose, via sorbitol: (rota poliol)
- Aldose-redutase: cristalino, retina, clulas de Schwann de nervos
perifricos, fgado, rins, placenta, eritrcitos e cls dos ovrios e da
vescula seminal.
- Sorbitol-desidrogenase: cls do fgado, ovrio, espermatozoides
e vesculas seminais.
- Glicose aldose-redutase (reduo) + NADPH sorbitol
sorbitol-desidrogenase (oxidao em carbono 2) + NADPH frutose
- Espermatozoides usam frutose como principal substrato
energtico no lquido seminal e trocam para glicose no trato reprodutor
feminino. A utilizao de frutose parece prevenir a quebra acrossomal da
membrana plasmtica (e consequente ativao) enquanto os
espermatozoides ainda esto no lquido seminal.
- Via do sorbitol no fgado: qualquer sorbitol disponvel
convertido em um substrato que pode entrar na gliclise ou na
gliconeognese.
- [glicose intracelular] + NADPH aldose-redutase sintetize
muito sorbitol acmulo de sorbitol na clula (acentuado quando
sorbitol-desidrogenase deficiente/ausente).
- O alto nvel de sorbitol gera efeitos osmticos muito fortes, causando inchao pela reteno
de gua. Em diabetes, pode ser responsvel pela formao de catarata, neuropatia perifrica e problemas
vasculares que levam formao de nefropatia e retinopatia.

METABOLISMO DA GALACTOSE

- Fosforilao da galactose, formando galactose-1-fosfato.


- Galactose-1-fosfato + UDP-glicose -> UDP-galactose + glicose-1-fosfato.
- UDP-galactose deve ser convertida a UDP-glicose pela UDP-glicose-epimerase reversvel (a
configurao do grupo hidroxila no carbono 4 revertida nessa reao).
- Galactosemia clssica: Deficincia de galactose-1-P-uridiltransferase. Leva ao acmulo de galactose
(acumula no sangue e urina) e galactose-1-fosfato (acumula nos tecidos). Esses compostos acumulados levam
produo de galactitol pela rota poliol (aldose-redutase). Causam leso heptica, retardo mental grave e
catarata.
- Galactosemia no clssica: Deficincia de Galactocinase, gerando acmulo de galactose. Pode causar
acmulo de galactitol se a galactose estiver presenta na dieta. Forma mais rara e mais branda.
DESCARBOXILAO OXIDATIVA DO PIRUVATO
- Processo de oxidao irreversvel no qual
o grupo carboxil removido do piruvato na forma
de uma molcula de CO2, e os dois carbonos
remanescentes so convertidos ao grupo acetil da
acetil-CoA.
- O NADH formado nessa reao doa um on
hidreto para a cadeia respiratria, que transferir
os dois eltrons ao oxignio ou, em microrganismos anaerbios, a um aceptor de eltrons alternativo, como
nitrato ou sulfato.
- A transferncia de eltrons do NADH ao oxignio gera, ao final, 2,5 molculas de ATP por par de
eltrons.
- Enzimas componentes do complexo:
- Piruvato-desidrogenase (ou piruvato-decarboxilase) (E1), di-hidrolipoil-transacetilase (E2) e
di-hidrolipoil-desidrogenase (E3), cada uma presente em mltiplas cpias.
- Sua associao fsica une as reaes na sequncia apropriada, sem a liberao dos
intermedirios.
- Coenzimas:
- A E1 requer tiamina-pirofosfato (TPP). A E2 requer cido lipico e coenzima A (CoA).
A E3 requer dinucleotdeo de flavina-adenina (FAD) e dinucleotdeo de nicotinamida-adenina (NAD).
- Quatro vitaminas diferentes essenciais nutrio humana so componentes vitais desse
sistema: tiamina (no TPP), riboflavina (no FAD), niacina (no NAD) e pantotenato (na CoA).
- A coenzima A tem um grupo tiol reativo (-SH) que crucial para a funo da CoA como
transportador de acilas em diferentes reaes metablicas.
- O lipoato tem dois grupos tiol que podem ser reversivelmente oxidados por uma ligao
dissulfeto. Devido sua capacidade de participar de reaes de oxidao e reduo, atua como transportador
de eltrons (hidrognio) e como transportador de acilas.
- Regulao do complexo da piruvato-desidrogenase
- A E1 inibida por protena-cinase independente de AMPc (por
fosforilao) e ativada por fosfoprotena-fosfatase.
- Cinase ativada por sinais ricos em energia (ATP, Acetil-CoA, NADH),
diminuindo a ao do complexo.
- Cinase inativada por sinais de falta de energia (NAD+, CoA), alm de
piruvato, acionando o complexo.
- A liberao de Ca++ durante a contrao muscular estimula o
complexo e a produo de energia.

- Deficincia na piruvato-desidrogenase:
- a causa bioqumica mais comum de acidose lctica congnita.
- Faz com que o piruvato seja desviado para a reao de formao
de cido lctico, via lactato-desidrogenase.
- Gera problema principalmente para o encfalo, que depende do
ciclo do cido ctrico para a produo da maior parte de sua energia e
muito sensvel acidose. Pode gerar profundo retardo psicomotor,
leses no crtex cerebral, levando morte.
- O defeito na E1 ligado ao X dominante.
- No h tratamento. Uma dieta cetognica (- carboidrato e +
gorduras) parece melhorar o quadro, fornecendo suprimento de
combustvel alternativo na forma de corpos cetnicos.
- Etapa 1: o C-1 do piruvato liberado como CO2, e o C-2, que no piruvato est no estado de oxidao
de um aldedo, unido ao TPP como um grupo hidroxietil. (a etapa mais lenta)
- Etapa 2: transferncia de dois eltrons e do grupo acetil a partir do TPP para a forma oxidada do grupo
lipoil-lisina do centro do complexo, formando o acetil-tioster do grupo lipoil reduzido.
- Etapa 3: transesterificao na qual o grupo -SH da CoA substitui o grupo -SH de E2, produzindo acetil-
CoA e a forma completamente reduzida (ditiol) do grupo lipoil.
- Etapa 4: E3 promove a transferncia de dois tomos de hidrognio dos grupos lipoil reduzidos de E2
ao grupo prosttico FAD de E3, restaurando a forma oxidada do grupo lipoil-lisina de E2, formando FADH2.
- Etapa 5: o FADH2 reduzido de E3 transfere um on hidreto ao NAD1, formando NADH.
PROBLEMA 2.2
CICLO DO CIDO CTRICO
- Para iniciar uma rodada do ciclo, a acetil-CoA doa seu grupo acetil ao composto de 4 carbonos
oxaloacetato, formando o composto de 6 carbonos citrato. O citrato , em seguida, transformado a isocitrato,
tambm uma molcula com 6 carbonos, o qual desidrogenado com a perda de CO2 para produzir o composto
de 5 carbonos -cetoglutarato (tambm chamado de oxoglutarato). O -cetoglutarato perde uma segunda
molcula de CO2, originando ao final o composto de 4 carbonos succinato. O succcinato , ento, convertido
por quatro etapas enzimticas ao composto de 4 carbonos oxaloacetato que est, assim, pronto para reagir
com outra molcula de acetil-CoA.
- Em cada rodada do ciclo entra um grupo acetil (2 carbonos) na forma de acetil-CoA, e so removidas
duas molculas de CO2; uma molcula de oxaloacetato utilizada para a formao do citrato e uma molcula
de oxaloacetato regenerada.
- 4 das 8 etapas deste processo so oxidaes, nas quais a energia da oxidao conservada de
maneira muito eficiente na forma das coenzimas reduzidas NADH e FADH2.
- Sua funo no est limitada conservao energtica: intermedirios do ciclo com 4 e 5 carbonos
servem como precursores para uma ampla variedade de produtos.
- A acetil-CoA produzida deve ser completamente oxidada a CO2 para que o mximo da energia
potencial possa ser extrado destes combustveis. No entanto, a oxidao direta do acetato (ou da acetil-CoA)
a CO2 no bioquimicamente possvel (formaria tambm CH4), menos reativo que o metileno formado.
- A oxidao do grupo metileno (oxidao do succinato) forma um grupo carbonil (oxaloacetato), que
quimicamente mais reativo do que metano/grupo metileno.
- Para a acetil-CoA ser oxidada de maneira eficiente, o seu grupo metil deve estar ligado a alguma coisa.
A primeira etapa do ciclo do cido ctrico resolve o problema do grupo metil pouco reativo por meio da
condensao da acetil-CoA com o oxaloacetato. O carbonil do oxaloacetato atua como centro eletroflico, que
atacado pelo carbono do grupo metil da acetil-CoA para a formao do citrato. O grupo metil do acetato
convertido a metileno no cido ctrico. Esse cido tricarboxlico, ento, prontamente passa por uma srie de
oxidaes que eliminam dois carbonos na forma de CO2.
- Cada etapa envolve ou uma oxidao que conserve energia ou ela um preldio necessrio para a
oxidao, colocando grupos funcionais em posies que facilitem a oxidao ou a descarboxilao oxidativa.

1) FORMAO DO CITRATO
- Condensao aldlica de acetil-CoA e
oxaloacetato para a formao do citrato, catalisada
pela citrato-sintase. (Condensao de Claisen)
- Apresenta equilbrio bastante deslocado para
direita.
- Citrato-sintase ativada alostericamente por
Ca2+ e ADP e inibida por ATP, NADH, succinil-CoA e
derivados acil-CoA graxos.
- No entanto, a principal forma de regulao pela
disponibilidade de seus substratos.
- O carbono do metil do grupo acetil unido ao grupo carbonil (C-2) do oxaloacetato.
- Acetil-CoA + oxaloacetato Citroil-CoA citrato + CoA livre.
- A hidrlise do intermedirio tio-ster transitoriamente formado de alta energia torna a reao direta
altamente exergnica.
- A grande e negativa variao de energia livre padro da reao da citrato-sintase fundamental para
o funcionamento do ciclo (j que a concentrao de oxaloacetato normalmente muito baixa). A CoA liberada
nessa reao reciclada para participar da descarboxilao oxidativa de outra molcula de piruvato pelo
complexo PDH.
- Citrato inibe fosfofrutocinase (que determina a veloc. da gliclise) e ativa acetil-CoA-carboxilase
(limitante da veloc. de sntese de cidos graxos).
2) ISOMERIZAO DO CITRATO
- A enzima aconitase (mais formalmente,
aconitato-hidratase) catalisa a transformao
reversvel do citrato a isocitrato, pela formao
intermediria do cido tricarboxlico cis-aconitato,
o qual normalmente no se dissocia do stio ativo.
- A aconitase pode promover a adio reversvel de
H2O ligao dupla do cis-aconitato ligado
enzima de duas maneiras diferentes, uma levando
a citrato e a outra a isocitrato.
- Embora a concentrao de isocitrato seja baixa no
equilbrio (menos de 10%, a 25C e pH 7,4), ele
rapidamente consumido na prxima etapa do ciclo,
diminuindo sua concentrao no estado estacionrio e deslocando a reao para a direita.
- A aconitase contm um centro de ferro-enxofre, que atua tanto na ligao do substrato ao stio ativo
quanto na adio ou na remoo cataltica de H2O. Em clulas exauridas de ferro, a aconitase perde o centro
de ferro-enxofre e adquire uma nova funo na regulao da homeostase do ferro. (Realiza mais de uma funo)
- A aconitase inibida por fluoracetato, utilizado como raticida. O fluoracetato convertido em
fluoracetil-CoA, que condensa com o oxalacetato para formar fluorcitrato (potente inibidor da aconitase),
resultando em acmulo de citrato.

3) OXIDAO E DESCARBOXILAO DO ISOCITRATO

- A isocitrato-desidrogenase catalisa a descarboxilao oxidativa irreversvel do isocitrato para formar


-cetoglutarato. O Mn2+ presente no stio ativo interage com o grupo carbonil do oxalosuccinato
intermedirio, mas s deixa o stio ativo quando a descarboxilao o converte em -cetoglutarato. O Mn2+
tambm estabiliza o enol formado transitoriamente por descarboxilao.
- Origina a primeira das 3 molculas de NADH produzidas pelo ciclo e a primeira liberao de CO2.
- Em todas as clulas, existem duas formas diferentes de isocitrato-desidrogenase, uma que exige
NAD+ como aceptor de eltrons e outra que exige NADP+. Em clulas eucariticas, a enzima dependente de
NAD encontra-se na matriz mitocondrial e participa do ciclo do cido ctrico. A principal funo da enzima
dependente de NADP, encontrada na matriz mitocondrial e no citosol, possivelmente seja a produo de
NADPH, essencial para as reaes redutoras anablicas.
- um dos passos limitantes da velocidade do ciclo do cido ctrico.
- A isocitrato-desidrogenase ativada alostericamente por ADP (sinaliza quantidade de energia na
clula) e por Ca++ e inibida por ATP e NADH (sinaliza quantidade de energia na clula).
4) DESCARBOLIXAO OXIDATIVA DO -CETOGLUTARATO
- O -cetoglutarato convertido a succinil-CoA
e CO2 pela ao do complexo da
a-cetoglutarato-desidrogenase.
- NAD+ o aceptor de eltrons e CoA o
transportador do grupo succinil.
- A energia da oxidao do -cetoglutarato
conservada pela formao da ligao tioster da
succinil-CoA.
- Essa reao essencialmente idntica reao da piruvato-desidrogenase. O complexo da
-cetoglutarato-desidrogenase bastante semelhante ao complexo da PDH em estrutura e funo; incorpora
trs enzimas homlogas s E1, E2 e E3 do complexo da PDH, e tambm contm TPP e lipoato ligado enzima,
FAD, NAD e coenzima A. O equilbrio da reao encontra-se deslocado no sentido da formao de succinil-CoA
um tio-ster de alta energia, semelhante acetil-CoA.
- A reao libera o segundo CO2 e produz o segundo NADH do ciclo.
- O complexo da -cetoglutarato-desidrogenase inibido por ATP, GTP, NADH e succinil-CoA, alm de
ser ativado por Ca++. No entanto, ele no regulado por reaes de fosforilao/desfosforilao, como
descrito para o complexo da piruvato-desidrogenase.

5) CLIVAGEM DA SUCCINIL-CoA
- A succinil-CoA, como a acetil-CoA, tem uma ligao
tioster com uma energia livre padro de hidrlise
grande e negativa. A energia liberada pelo
rompimento dessa ligao utilizada para impelir a
sntese de uma ligao fosfoanidrido no GTP ou ATP,
com formao de succinato.
- A enzima que catalisa essa reao reversvel
chamada de succinil-CoA-sintetase ou succinato-
tiocinase.
- Essa reao que poupa energia envolve uma etapa intermediria, na qual a prpria molcula da
enzima fosforilada em um resduo de His no stio ativo. Esse grupo fosfato, que tem alto potencial de
transferncia de grupo, transferido ao ADP (ou GDP) para a formao de ATP (ou GTP).
- A formao de ATP (ou GTP) custa da energia liberada pela descarboxilao oxidativa do
-cetoglutarato uma fosforilao ao nvel do substrato. O GTP formado pela succinil-CoA-sintetase pode
doar o grupo fosfato terminal ao ADP para formar ATP, em uma reao reversvel catalisada pela nucleosdeo-
difosfato-cinase.
- O resultado lquido da atividade de cada isoenzima da succinil-CoA-sintetase a conservao de
energia como ATP. No h variao de energia livre na reao da nucleosdeo-difosfato-cinase; ATP e GTP so
energeticamente equivalentes.

6) OXIDAO DO SUCCINATO
- O succinato formado a partir da succinil-CoA oxidado
a fumarato pela flavoprotena succinato-desidrogenase.
- A enzima contm trs grupos ferro-enxofre diferentes
e uma molcula de FAD covalentemente ligada. Os
eltrons do succinato passam pelo FAD e pelos centros
de ferro-enxofre antes de entrarem na cadeia de
transportadores de eltrons da membrana mitocondrial interna. O fluxo dos eltrons do succinato ao longo
desses transportadores acoplado sntese de ~ 1,5 molc de ATP por par de eltrons.
- Malonato, anlogo do succinato normalmente ausente nas clulas, um forte inibidor competitivo da
succinato-desidrogenase. Oxalacetato tambm inibe a succinato-desidrogenase.
- O FAD o aceptor de eltrons porque o poder redutor do succinato no suficiente para reduzir o NAD+.
7) HIDRATAO DO FUMARATO
- A hidratao reversvel do fumarato a L-malato
catalisada pela fumarase (fumarato-hidratase).
- O fumarato tambm produzido pelo ciclo da ureia, na
sntese de purinas e durante o catabolismo dos AA
fenilalanina e tirosina.
- Essa enzima altamente estereoespecfica; ela catalisa a
hidratao da ligao dupla trans do fumarato, mas no a
da ligao dupla cis do maleato (o ismero cis do fumarato).
Na direo inversa (de L-malato para fumarato), a fumarase
igualmente estereoespecfica: D-malato no um
substrato.

8) OXIDAO DO MALATO
- A L-malato-desidrogenase ligada ao NAD catalisa a
oxidao de L-malato a oxaloacetato, produzindo o
terceiro e ltimo NADH do ciclo.
- O equilbrio dessa reao muito deslocado para a
esquerda sob as condies termodinmicas padro,
porm, nas clulas intactas, o oxaloacetato
continuamente removido pela reao altamente exergnica da citrato-sintase. Isso mantm a concentrao
celular de oxaloacetato extremamente baixa, deslocando a reao da malato-desidrogenase no sentido da
formao de oxaloacetato.
- O oxaloacetato tambm produzido por transaminao, a partir do AA cido asprtico.

- Embora o ciclo do cido ctrico gere diretamente somente um ATP por rodada (na converso de succinil-CoA
a succinato), as quatro etapas de oxidao do ciclo abastecem a cadeia respiratria, via NADH e FADH2, com
um grande fluxo de eltrons e, assim, levam formao de um grande nmero de molculas de ATP durante
a fosforilao oxidativa.
- Um grupo acetil com dois carbonos entra no ciclo combinando-se com o oxaloacetato. Dois tomos de
carbono saem do ciclo na forma de CO2 pela oxidao do isocitrato e do a-cetoglutarato. A energia liberada
por estas oxidaes foi conservada pela reduo de trs NAD+ e um FAD e pela produo de um ATP ou GTP.
No final do ciclo, uma molcula de oxaloacetato foi regenerada. Os dois tomos de carbono que emergem
como CO2 no so os mesmos dois carbonos que entraram na forma de grupo acetil.
- A oxidao de um NADH pela cadeia transportadora de eltrons gera aproximadamente 2,5 ATP, enquanto
a oxidao do FADH2 fornece 1,5 ATP.

REAES ANAPLERTICAS
- Conforme os intermedirios do ciclo do cido ctrico so removidos para servirem como precursores
na biossntese, eles so repostos por reaes anaplerticas.
- A reao anaplertica mais importante no fgado e nos rins de mamferos a carboxilao reversvel
do piruvato pelo CO2 para a formao de oxaloacetato, catalisada pela piruvato-carboxilase. Quando o ciclo
do cido ctrico est deficiente em oxaloacetato ou qualquer outro intermedirio, o piruvato carboxilado
para produzir mais oxaloacetato. A adio enzimtica de um grupo carboxil ao piruvato requer energia, que
suprida pelo ATP a energia livre necessria para unir um grupo carboxil ao piruvato aproximadamente igual
energia livre disponibilizada pelo ATP.
- Sempre que a acetil-CoA, o combustvel do ciclo do cido ctrico, est presente em excesso, ela
estimula a reao da piruvato-carboxilase para a produo de mais oxaloacetato, permitindo que o ciclo utilize
mais acetil-CoA na reao da citrato-sintase.

- Alm da acetil-CoA, qualquer composto que origine um intermedirio do ciclo do cido ctrico com quatro
ou cinco carbonos por exemplo, os produtos da degradao de muitos aminocidos podem ser oxidados
pelo ciclo.
- O ciclo do cido ctrico anfiblico, servindo ao catabolismo e ao anabolismo; os intermedirios do ciclo
podem ser desviados e utilizados como material de partida para diversos produtos da biossntese.
REGULAO DO CICLO DO CIDO CTRICO
- Trs fatores controlam a velocidade do fluxo no
ciclo: disponibilidade de substrato, inibio pelos
produtos acumulados e inibio alostrica por
retroalimentao das enzimas que catalisam as
etapas iniciais do ciclo.
- Cada uma das trs etapas fortemente exergnicas
do ciclo aquelas catalisadas por citrato-sintase
(etapa 1), isocitrato-desidrogenase (etapa 3) e
-cetoglutarato-desidrogenase (etapa 4) podem
tornar-se a etapa limitante da velocidade sob
algumas circunstncias.
- A disponibilidade dos substratos da citrato-
sintase (acetil-CoA e oxaloacetato) varia com o
estado metablico da clula e, s vezes, limita a
taxa de formao de citrato.
- O NADH, produto da oxidao do citrato e do
-cetoglutarato, acumula-se sob determinadas
condies, e em alta [NADH]/[NAD+] ambas as
reaes de desidrogenao so fortemente
inibidas pela ao das massas.
- Na clula, de maneira similar, a reao da malato-desidrogenase est essencialmente em equilbrio
(ou seja, limitada pelo substrato), e quando a [NADH]/[NAD+] est alta, a concentrao de oxaloacetato est
baixa, desacelerando a primeira etapa do ciclo.
- O acmulo de produto inibe as trs etapas limitantes do ciclo:
- A succinil-CoA inibe a -cetoglutarato-desidrogenase (e tambm a citrato-sintase);
- O citrato bloqueia a citrato-sintase;
- O produto final, ATP, inibe a citrato-sintase e a isocitrato-desidrogenase. A inibio da citrato-
sintase pelo ATP abrandada por ADP, um ativador alostrico dessa enzima.
- Nos msculos dos vertebrados, Ca2+, o sinalizador para a contrao e o aumento concomitante na
demanda de ATP, ativa a isocitrato-desidrogenase e a -cetoglutarato-desidrogenase, assim como ativa o
complexo da PDH. Dessa forma, as concentraes dos substratos e dos intermedirios do ciclo do cido ctrico
ajustam o fluxo dessa via para a velocidade que fornea as concentraes timas de ATP e NADH.
- Sob condies normais, as velocidades da gliclise e do ciclo do cido ctrico esto integradas de modo
que a quantidade de glicose metabolizada a piruvato seja exatamente a quantidade suficiente para suprir o
ciclo do cido ctrico com o seu combustvel, os grupos acetil da acetil-CoA.
- Piruvato, lactato e acetil-CoA normalmente so mantidos nas concentraes do estado estacionrio.
- A velocidade da gliclise vinculada velocidade do ciclo do cido ctrico no apenas por meio da
inibio pelos altos nveis de ATP e NADH, os quais so comuns a ambos os estgios da oxidao da glicose, a
via glicoltica e a respirao, mas tambm regulada pela concentrao de citrato. Citrato, o produto da
primeira etapa do ciclo do cido ctrico, um importante inibidor alostrico da fosfofrutocinase-1 na via
glicoltica.
CADEIA TRANSPORTADORA DE
ELTRONS / FOSFORILAO OXIDATIVA

- A fosforilao oxidativa comea com a


entrada de eltrons na cadeia de carregadores
de eltrons, chamada de cadeia respiratria. A
maioria desses eltrons surge da ao das
desidrogenases, que coletam eltrons das vias
catablicas e os canalizam para aceptores
universais de eltrons nucleotdeos de
nicotinamida (NAD+ ou NADP+) ou
nucleotdeos de flavina (FMN ou FAD).
- A energia gerada se deve transformao
da energia contida nas coenzimas reduzidas em
um gradiente de prtons e utilizar esse
gradiente para promover a sntese de ATP.
- A produo do gradiente de prtons feita
pela transferncia dos eltrons das coenzimas
atravs de passagens intermedirias por vrios
compostos, que constituem uma cadeia de
transporte de eltrons.
- Os eltrons partem da coenzima reduzida (que contm potencial de xido-reduo menor que os
compostos da cadeia de transporte), percorrem uma sequncia de transportadores com potenciais
crescentes, at atingirem o O2, que tem o maior potencial.
- O transporte facilitado pelo fato de tais compostos estarem organizados em membranas, com
posies definidas, de modo a situar cada componente exatamente entre aquele que lhe fornecer eltrons
e aquele ao qual seus eltrons sero doados.
- Ao mesmo tempo da passagem dos eltrons, estabelece-se uma concentrao de prtons diferente
de cada lado da membrana onde ocorre o transporte. A sntese de ATP possvel porque aproveita a energia
potencial contida no gradiente de prtons.
- Os transportadores de eltrons esto agrupados em quatro complexos:
- H a presena de 4 complexos, designados I, II, III e IV.
- Sem fazer parte de complexos, aparecem ainda a coenzima Q (CoQ), que conecta os
complexos I e II ao complexo III, e o citocromo c, uma protena livremente solvel que conecta o complexo III
ao complexo IV.
- Os eltrons presentes no NADH so transferidos dele para o complexo I, do complexo I para a
coenzima Q, depois para o complexo III, citocromo c, complexo IV e finalmente para o oxignio.
- Os eltrons presentes no succinato e em outros substratos tm uma entrada especial na
cadeia: so transferidos ao complexo II e deste para a coenzima Q, desse ponto em diante, seguem o caminho
comum.
- Essas transferncias so possveis porque todos os componentes do complexo, mais CoQ e o
citocromo c, podem-se apresentar nos estados reduzido e oxidado recebem um eltron do componente
anterior e se reduzem; transferem o eltron ao componente seguinte e se oxidam.
- Com exceo da coenzima Q, todos os componentes da cadeia de transporte so protenas.
As protenas esto associadas a grupos prostticos (FAD, FMN) e centros ferro-enxofre.
COMPLEXO I

- O complexo I oxida o NADH, transferindo


seus eltrons para a CoQ (ou apenas Q).
- O complexo I, tambm chamado de NADH:
ubiquinona-oxidorredutase ou NADH-
desidrogenase, uma enzima grande,
composta por 42 cadeias diferentes de
polipeptdeos, incluindo uma flavoprotena
contendo FMN e pelo menos seis centros de
ferro-enxofre. O complexo I tem formato de L,
com um brao do L na membrana e o outro se
estendendo para a matriz.
- O complexo I catalisa dois processos
simultneos e obrigatoriamente acoplados: (1)
a transferncia exergnica para a ubiquinona
de um on hidreto do NADH e de um prton da
matriz e (2) a transferncia endergnica de
quatro prtons da matriz para o espao
intermembrana.

- O complexo I um bombeador de prtons


que utiliza a energia da transferncia de
eltrons, e a reao que ele catalisa vetorial:
ela move prtons em uma direo especfica de
um local (a matriz, que se torna negativamente
carregada com a sada dos prtons) a outro (o
espao intermembrana, que se torna
positivamente carregado).
- Os Centros Fe-S no recebem prtons, so
transportadores de eltrons unicamente, recebidos pelos ons ferro (Fe3+ Fe2+).
- Transferncia de um on hidreto de NADH a FMN, de onde dois eltrons passam por uma srie de
centros de Fe-S para o centro Fe-S N-2 no brao da matriz do complexo. A transferncia de eltrons de N-2
para a ubiquinona no brao da membrana forma QH2, que se difunde na bicamada lipdica. Esta transferncia
de eltrons tambm governa a expulso da matriz de quatro prtons por par de eltrons.
- O fluxo de prtons produz um potencial eletroqumico atravs da membrana mitocondrial interna.
- Eltrons transferidos ao FMNH2 Centro Fe-S passagens intermedirias por outros centros Fe-S
deixam o complexo I entregues CoQ.
- Como no h transferncia de prtons pelos centros, sendo transferidos para o espao
intermembrana, gerando o potencial.
- Amital (frmaco do tipo barbiturato, depressor do SNC, usado como antiepilptico, sedativo e
hipntico), rotenona (produto vegetal comumente usado como inseticida) e piericidina A (antibitico) inibem
o fluxo de eltrons dos centros Fe-S do complexo I para a ubiquinona, bloqueando o processo global da
fosforilao oxidativa.
COMPLEXO II
- Este complexo tem duas subunidades
transmembrana, C e D: as extenses citoplasmticas
contm as subunidades A e B. Logo atrs de FAD na
subunidade A est o stio de ligao do succinato. A
subunidade B tem trs conjuntos de centros de Fe-S;
ubiquinona ligada subunidade B; um heme b est
localizado entre as subunidades C e D. Duas molculas
de fosfatidiletanolamina esto to fortemente ligadas
subunidade D que aparecem na estrutura cristalina. Os
eltrons se movem (setas azuis) do succinato ao FAD e,
ento, atravs de trs centros de Fe-S, para a
ubiquinona. O heme b no est na via principal da
transferncia de eltrons, mas protege contra a
formao de espcies reativas de oxignio (ERO) por
eltrons que saem da via.
- Tambm chamado succinato-CoQ redutase,
representa uma segunda porta de entrada de eltrons
na cadeia de transporte em direo ao O2.
- Faz parte desse complexo a enzima succinato
desidrogenase, a nica do ciclo de Krebs presente na
membrana e no na matriz mitocondrial.
- Essa enzima tem FAD como grupo prosttico e catalisa a oxidao de succinato a fumarato: os eltrons
e prtons do succinato so transferidos para o FAD, que se reduz a FADH2.
- Outros componentes: alguns centros Fe-S e o citocromo b560. Por eles passam os eltrons derivados
do FADH2 antes de atingirem a CoQ.
- Nem os centros nem o citocromo recebem prtons; os prtons presentes no FADH2 so devolvidos
matriz mitocondrial. Assim, o complexo II no contribui para a formao do gradiente de prtons.
- O heme b do complexo II aparentemente no est na via direta de transferncia de eltrons; em vez
disso, pode servir para reduzir a frequncia com que eltrons vazam para fora do sistema, movendo-se do
succinato ao oxignio molecular para produzir as espcies reativas de oxignio (ERO) perxido de hidrognio
(H2O2) e o radical superxido (O2-). Humanos com mutaes pontuais nas subunidades do complexo II perto
do heme b ou do stio de ligao da quinona sofrem de paraganglioma hereditrio. Essa condio hereditria
caracterizada por tumores benignos da cabea e do pescoo, comumente no corpo carotdeo, rgo que
percebe os nveis de O2 no sangue.

COENZIMA Q (CoQ; Q)

- Ubiquinona (Q) o ponto de entrada para os eltrons


derivados das reaes do citosol, a partir de reaes de
oxidao dos cidos graxos e da oxidao do succinato (no
ciclo do cido ctrico). Os eltrons do NADH passam por uma
flavoprotena com o cofator FMN e para uma srie de centros
Fe-S (no complexo I) e, ento, para Q. Os eltrons do
succinato passam por uma flavoprotena com o cofator FAD
e de vrios centros Fe-S (no complexo II) a caminho de Q. O
glicerol-3-fosfato doa eltrons a uma flavoprotena (glicerol-
3-fosfato-desidrogenase) na face externa da membrana
mitocondrial interna, de onde eles passam para Q. A acil-
CoA-desidrogenase (a primeira enzima da b-oxidao)
transfere eltrons flavoprotena transferidora de eltrons (FTE), de onde eles passam a Q por meio da
FTE:ubiquinona-oxidorredutase.
- As caractersticas hidrofbicas da CoQ permitem sua mobilidade na fase lipdica da membrana. Ela
recebe 2 prtons e 2 eltrons, passando forma reduzida QH2.
- Alm do complexo I e II, recebe eltrons de outras vias. Nessas vias, o substrato oxidado por uma
desidrogenase, que uma flavoprotena, com reduo de FAD a FADH2; os eltrons, depois de passarem por
compostos intermedirios, so entregues CoQ.
- Esses compostos, quer estejam presentes nos complexos I e II ou nas flavoprotenas, no transferem
prtons. Ao receber eltrons, a CoQ retira prtons da matriz mitocondrial, convertendo-se na forma reduzida,
QH2.
- Por outro lado, o complexo III, que recebe eltrons de QH2, no recebe prtons e estes sofrem
extruso para o espao intermembrana.

COMPLEXO III

- Tambm chamado coenzima Q citocromo c redutase, constitudo por dois citocromos b, por um
centro Fe-S e pelo citocromo c1.
- As transferncias de eltrons da CoQ para os componentes do complexo III e deste para o citocromo
c1 so acompanhadas de movimentos de prtons; os citocromos e o centro Fe-S recebem apenas eltrons, e
os prtons presentes na CoQ reduzida (QH2) so liberados no espao intermembranas.
- Ciclo Q: consta de duas etapas, nas quais os dois eltrons de QH2 so passados, um a um, para o
citocromo c, com a formao intermediria da semiquinona, QH.
- Na primeira etapa, QH2 perde 1 eltron e 1 prton, formando QH. O eltron segue a rota QH2
Fe-S c1 c e o H+ liberado no espao intermembranas. A semiquinona converte-se na forma oxidada,
Q, por transferncia do seu eltron aos citocromos b e por extruso do prton. O eltron volta para o Q, que
reage com um H+ da matriz, reconstituindo QH. A transferncia de um dos eltrons de QH2 para o citocromo
c resulta, portanto, na extruso de 2H+ e no consumo de um H+ da matriz para formar QH.
- Na segunda etapa, outra molcula de QH2 percorre a mesma sequncia de reaes da
primeira etapa, at a passagem do eltron para os citocromos b e formao de Q. Nessa etapa, esse eltron
utilizado para reduzir a semiquinona formada na etapa anterior (QH) e, custa de um H+ do interior da
mitocndria, regenerar QH2. A reduo da 2 molcula de citocromo c promove a extruso de mais 2 H+, o
consumo de um outro H+ da matriz, a regenerao da molcula de QH2 consumida e a produo da forma
oxidada, Q, que se torna disponvel para receber os eltrons dos componentes da cadeia que a antecedem.
CITOCROMO C
- Ao contrrio dos outros citocromos, que so protenas integradas, o citocromo c uma protena
perifrica e relativamente pequena. Seu tamanho e mobilidade permitem-lhe cumprir sua funo na cadeia
de transporte de eltrons, a de conectar o complexo III, do qual recebe eltrons, ao complexo IV, ao qual doa
eltrons.

COMPLEXO IV
- Tambm chamado citocromo c oxidase,
contm dois citocromos do tipo a (a e a3) e dois
ons de cobre, cada qual associado a um dos dois
citocromos. Os ons de cobre, alternando entre
os estados de oxidao Cu2+ e Cu+, fazem parte
do transporte de eltrons.
- A subunidade II mitocondrial contm dois ons
cobre complexados com os grupos -SH de dois
resduos de Cys em um centro binuclear que
lembra os centros de 2Fe-2S das protenas ferro-
enxofre. A subunidade I contm dois grupos
heme, designados a e a3, e um outro on cobre
(CuB). Heme a3 e CuB formam um segundo
centro binuclear que aceita eltrons de heme a e
os transfere ao O2 ligado ao heme a3.
- A transferncia de eltrons pelo complexo IV
d-se do citocromo c para o centro de CuA, para
o heme a, para o centro de heme a3-CuB e,
finalmente, para o O2. Para cada quatro eltrons
que passam por complexo, a enzima consome
quatro substratos H+ da matriz (lado N) na
converso de O2 a 2H2O. Ela tambm usa a
energia dessa reao redox para bombear um prton para fora em direo ao espao intermembrana (lado P)
para cada eltron que passa, contribuindo para o potencial eletroqumico produzido pelo transporte de
prtons possibilitado pela energia dessas reaes redox pelos complexos I e III.
- Esta reduo do O2 por quatro eltrons envolve centros redox
que carregam apenas um eltron por vez, e ela deve ocorrer sem
a liberao de intermedirios que no so completamente
reduzidos, como perxido de hidrognio ou radicais livres hidroxila espcies muito reativas que danificariam
os componentes celulares. Os intermedirios permanecem fortemente ligados ao complexo, at serem
completamente convertidos em gua.
FOSFORILAO OXIDATIVA

- A transferncia de eltrons ao longo da cadeia de transporte de eltrons energeticamente


favorecida, pois o NADH um forte doador de eltrons, e o O2 um vido aceptor de eltrons. O fluxo de
eltrons do NADH para o oxignio, porm, no resulta diretamente na sntese de ATP.

HIPTESE QUIMIOSMTICA
- Explica como a energia livre gerada pelo transporte de eltrons atravs da cadeia transportadora de
eltrons utilizada para produzir ATP a partir de ADP + Pi.
- Bomba de prtons: O transporte de eltrons est acoplado fosforilao do ADP pelo
transporte de prtons (H+) atravs da membrana mitocondrial interna, prtons esses que so bombeados da
matriz para o espao intermembranas. Esse processo cria, atravs da membrana mitocondrial interna, um
gradiente eltrico (com cargas + positivas no lado externo que no lado interno) e um gradiente de pH (o meio,
no lado de fora da membrana, est em um pH mais baixo do que no lado interno. A energia gerada por esse
gradiente impulsiona a sntese de ATP, funcionando como o intermedirio comum que acopla a oxidao
fosforilao.
- ATP-sintase: Complexo que sintetiza ATP, utilizando a energia do gradiente de prtons
gerado pela cadeia transportadora. Tambm chamada de ATPase, pois, isolada, catalisa a hidrlise do ATP a
ADP e Pi. A hiptese prope que, aps os prtons serem transferidos para o lado citoslico da membrana
mitocondrial interna, eles retornam matriz mitocondrial passando atravs de um canal no complexo ATP-
sintase, resultando ento na sntese de ATP a partir de ADP e Pi e, ao mesmo tempo, dissipando os gradientes
eltricos e de pH.
- Oligomicina: droga que se liga ao pednculo da ATP-sintase, fechando o canal de H+ e
impedindo o retorno dos prtons matriz mitocondrial. Uma vez que os gradientes eltrico e de pH no
podem ser dissipados na presena dessa droga, o transporte de eltrons cessa, devido dificuldade de
bombear mais prtons contra gradientes to grandes.
- Protenas desacopladoras: protenas encontradas na membrana mitocondrial que
criam um vazamento de prtons, permitindo a eles retornarem matriz mitocondrial sem que a energia
seja capturada como ATP, sendo liberada como calor.
- Desacopladores sintticos: O transporte de eltrons e a fosforilao podem ser
desacoplados por meio de compostos que aumentam a permeabilidade mitocondrial interna a prtons. Ex.:
2,4-dinitrofenol, transportador de prtons lipoflico, que difunde facilmente atravs da membrana. Faz com
que o transporte de eltrons funcione em uma alta velocidade, sem estabelecer um gradiente de prtons, de
forma semelhante ao que ocorrem com as UCPs. A energia produzida liberada como calor em vez de ser
utilizada para sintetizar ATP. Em doses altas, alguns salicilatos desacoplam a fosforilao, explicando a febre
que acompanha superdoses txicas.
- Sistema de transporte atravs da membrana:
- A membrana interna impermevel maior parte das substncias hidroflicas ou com carga.
Essa membrana contm, porm, numerosas protenas de transporte, que permitem a passagem de molculas
especficas desde o citosol at a matriz.
- Transporte de ATP-ADP: A membrana interna requer transportadores especficos para
transportar o ADP e o Pi do citosol para dentro da mitocndria. Um carreador de nucleotdeos da adenina
transporta uma molcula de ADP do citosol para a mitocndria, enquanto exporta um ATP da matriz para o
citosol. Ele fortemente inibido pela toxina vegetal atractilosdeo, resultando na depleo do conjunto de
ADP intramitocondrial e na inibio da produo de ATP.
- Transporte de equivalentes redutores: A membrana interna no possui uma protena
transportadora de NADH, de modo que o NADH produzido no citosol no pode penetrar diretamente na
mitocndria. Os dois eltrons do NADH, no entanto, so transportados do citosol para a mitocndria utilizando
mecanismos de lanadeiras. Na lanadeira do glicerofosfato, dois eltrons so transferidos do NADH para a
flavoprotena-desidrogenase, localizada na membrana interna. Essa enzima, ento, doa seus eltrons para a
cadeia de transporte, de modo semelhante ao que ocorre com a succinato-desidrogenase. Essa lanadeira
resulta, portanto, na sntese de dois ATPs para cada NADH citoslico oxidado. Na lanadeira do malato-
aspartato, ocorre produo de NADH (em vez de FADH2) na matriz mitocondrial, levando produo de 3
ATPs para cada NADH citoslico oxidado.

VIA DAS PENTOSES


- Oxidao da glicose-6-fosfato at
pentoses-fosfato pela via das pentoses-
fosfato (tambm chamada de via do
fosfogliconato ou via da hexose-
monofosfato). Nessa via de oxidao, NADP+
o aceptor de eltrons, gerando NADPH.
- As clulas que se dividem rapidamente,
como aquelas da medula ssea, da pele e da
mucosa intestinal, assim como aquelas de
tumores, utilizam a pentose ribose-5-fosfato
para fazer RNA, DNA e coenzimas como ATP,
NADH, FADH2 e coenzima A.
- Em outros tecidos, o produto essencial da
via das pentoses-fosfato no pentose, mas o
doador de eltrons NADPH, necessrio para as
redues biossintticas ou para contrapor os
efeitos deletrios dos radicais de oxignio. Os
tecidos em que ocorre a sntese de grande
quantidade de cidos graxos (fgado, tecido
adiposo, glndulas mamrias durante a
lactao) ou a sntese muito ativa de colesterol e hormnios esteroides (fgado, glndulas suprarrenais e
gnadas) utilizam o NADPH produzido por essa via.
- Os eritrcitos e as clulas da crnea e do cristalino esto diretamente expostos ao oxignio e, por
isso, aos efeitos danosos dos radicais livres gerados pelo oxignio. Por manter um ambiente redutor (uma
relao alta de NADPH para NADP+ assim como da forma reduzida para a forma oxidada da glutationa), essas
clulas podem impedir ou recuperar o dano oxidativo de protenas, lipdeos e outras molculas sensveis.
REAES DE OXIDAO
- Desidrogenao da glicose-6-fosfato:
- A glicose-6-fosfato-desidrogenase catalisa a reao de oxidao
irreversvel da glicose-6-fosfato a 6-fosfogliconolactona, em uma
reao que usa especificamente NADP+ como sua coenzima. A via das
pentoses regulada basicamente nessa reao.
- O NADPH um potente inibidor competitivo da enzima e, na maioria
das condies metablicas, a relao NADPH/NADP suficientemente
alta para inibir a atividade da enzima.
- Com o aumento da demanda por NADPH, essa relao diminui, e o
fluxo atravs do ciclo aumenta, devido maior atividade da enzima.
- A insulina aumenta a expresso do gene da G6PD, de modo que o
fluxo atravs da via cresce no estado alimentado.
- Formao da ribose-5-fosfato:
- A 6-fosfogliconolactona hidrolisada pela
6-fosfogliconolactona-hidrolase (lactonase), produzindo
6-fosfogliconato. Reao irreversvel e no-limitante.
- A subsequente descarboxilao oxidativa do 6-fosfogliconato
catalisada pela 6-fosfogliconato-desidrogenase, reao
irreversvel que produz ribulose-5-fosfato, CO2 e uma segunda
molcula de NADPH.
- A fosfopentose-isomerase converte a ribulose-5-fosfato ao seu
ismero aldose, ribose-5-fosfato.

- Em alguns tecidos, a via das pentoses-fosfato termina nesse


ponto, com produo lquida de 2 NADPH (agente redutor para as
reaes biossintticas), CO2, 2 H+ e ribose-5-fosfato (precursor
para a sntese de nucleotdeos.
FASE NO OXIDATIVA
- Em tecidos que requerem principalmente NADPH, as pentoses-fosfato produzidas na fase oxidativa
da via so recicladas em glicose-6-fosfato. Nessa fase no oxidativa, a ribulose-5-fosfato primeiro
epimerizada a xilulose-5-fosfato.
- A seguir, em uma srie de rearranjos dos esqueletos de carbono, seis molculas de acar-fosfato de
cinco tomos de carbono so convertidas a cinco molculas de acar-fosfato com seis tomos de carbono,
completando o ciclo e permitindo a oxidao contnua de glicose-6-fosfato com a produo de NADPH.
- A reciclagem contnua leva finalmente converso de glicose-6-fosfato a seis CO2.
- Duas enzimas exclusivas da via das pentoses-fosfato agem nessas interconverses.
- Transcetolase: catalisa a transferncia de um fragmento de dois carbonos de uma cetose doadora a
uma aldose receptora.
- Em sua primeira apario na via, a transcetolase transfere C-1 e C-2 da xilulose-5-fosfato para
a ribose-5-fosfato, formando o produto de sete carbonos sedoeptulose-7-fosfato. O fragmento de trs
carbonos remanescente da xilulose o gliceraldedo-3-fosfato.
- Transaldolase: catalisa a transferncia de um fragmento de trs carbonos.
- Um fragmento de trs carbonos removido da sedoeptulose-7-fosfato e condensado com o
gliceraldedo-3-fosfato, formando frutose-6-fosfato e a tetrose eritrose-4-fosfato.
- Nesse ponto, a transcetolase age novamente, transferindo dois carbonos da xilulose-5-fosfato para a
eritrose-4-fosfato, formando frutose-6-fosfato. O remanescente o gliceraldedo-3-fosfato.
- Duas molculas de gliceraldedo-3-fosfato formadas por duas repeties dessas reaes podem ser
convertidas a uma molcula de frutose-1,6-bifosfato como na gliconeognese, e finalmente a FBPase-1 e a
fosfo-hexose-isomerase convertem frutose-1,6-bifosfato a glicose-6-fosfato. No total, seis pentoses-fosfato
so convertidas a cinco hexoses-fosfato.

USOS DO NADPH
- Biossntese redutora: Parte da energia da glicose-6-fosfato convertida em NADPH uma molcula
que pode ser usada em reaes que demandam alto poder redutor de um doador.
- Reduo do perxido de hidrognio
- A glutationa reduzida (GSH) protege a clula por
destruir o perxido de hidrognio e os radicais
livres hidroxil. A regenerao de GSH a partir de
sua forma oxidada (GSSG) requer a produo de
NADPH na reao da glicose-6-fosfato-
desidrogenase.
- Os eritrcitos so totalmente dependentes da
via das pentoses para suprimento de NADPH, j
que no possuem fonte alternativa da coenzima.
Se a G6PD estiver deficiente, os nveis de NADPH
cairo, e a glutationa oxidada no poder ser
reduzida. Isso causar acmulo de H2O2,
aumentando a instabilidade da membrana e
causando a lise dos eritrcitos.
- Substncias antioxidantes (alguns agentes
redutores intracelulares, como ascorbato,
vitamina E, -caroteno) conseguem reduzir e,
assim, destoxificar intermedirios do oxignio.

- Sntese de xido ntrico: O NO o fator relaxante derivado do endotlio, que causa vasodilatao ao relaxar
os msculos vasculares. Alm disso, age como neurotransmissor, impede a agregao plaquetria e cumpre
um papel essencial na funo do macrfago (macrfagos ativados formam radicais superxido que se
combinam com o NO para formar intermedirios, os quais se decompem, formando o radical OH, altamente
bactericida).
PROBLEMA 2.3
GLICONEOGNESE
- Em mamferos, alguns tecidos dependem quase completamente de glicose para sua energia
metablica. Para o crebro humano e o sistema nervoso, assim como para os eritrcitos, os testculos, a
medula renal e os tecidos embrionrios, a glicose do sangue a principal ou a nica fonte de combustvel.
- Apenas o crebro requer em mdia 120 g de glicose por dia mais da metade de toda a glicose
estocada como glicognio nos msculos e no fgado. No entanto, o suprimento de glicose a partir desses
estoques no sempre suficiente; entre as refeies e durante perodos de jejum mais longos, ou aps
exerccio vigoroso, o glicognio se esgota. Para esses perodos, a sntese de glicose realizada pela
gliconeognese, que converte em glicose o piruvato e os compostos relacionados, com 3 e 4 carbonos.
- Em mamferos, a gliconeognese ocorre principalmente no fgado, e em menor extenso no crtex
renal e nas clulas epiteliais que revestem internamente o intestino delgado. A glicose assim produzida passa
para o sangue e vai suprir outros tecidos.

SUBSTRATOS DA GLICONEOGNESE
- Glicerol: A hidrlise de triacilgliceris leva formao de
cidos graxos e pequena quantidade de glicerol. O catabolismo
da maior parte dos cidos graxos gera apenas Acetil-CoA, que
no precursor de glicose (a reao da piruvato-desidrogenase
irreversvel e as clulas no possuem outra via para converter
acetil-CoA em piruvato).
- O glicerol liberado pela hidrlise fosforilado pela
glicerol-cinase, resultando em glicerol-fosfato. A oxidao do
carbono central do glicerol-fosfato pela glicerol-fosfato-
desidrogenase leva formao de di-hidroxiacetona-fosfato,
intermedirio da gliconeognese no fgado.
- O glicerol-fosfato um intermedirio essencial na sntese
de triacilgliceris nos adipcitos, mas esses no possuem a
glicerol-cinase. Em vez disso, os adipcitos realizam a
gliceroneognese: a converso de piruvato a di-hidroxiacetona-
fosfato pelas reaes iniciais da gliconeognese, seguida pela
reduo da di-hidroxiacetona-fosfato em glicerol-fosfato pela
glicerol-3-fosfato-desidrogenase.

- Lactato: Liberado no sangue pelo msculo esqueltico em exerccio e pelas clulas que no possuem
mitocndrias (eritrcitos). Pelo Ciclo de Cori, o lactato produzido captado pelo fgado, oxidado a piruvato e
reconvertido em glicose, que liberada de volta para circulao.
- Aminocidos: Alguns ou todos os tomos de carbono da maior parte dos aminocidos derivados das
protenas so basicamente catabolizados a piruvato ou em intermedirios do ciclo do cido ctrico. Tais
aminocidos podem, portanto, ser convertidos a glicose e so chamados de glicognicos.
- A alanina e a glutamina, as principais molculas que transportam grupos amino de tecidos
extra-hepticos at o fgado, so aminocidos glicognicos particularmente importantes em mamferos. Aps
a retirada de seus grupos amino da mitocndria dos hepatcitos, os esqueletos de carbono remanescentes
(piruvato e -cetoglutarato, respectivamente) so prontamente canalizados para a gliconeognese.
- Alm do catabolismo de cidos graxos, a acetil-CoA pode ser produzida por acetoacetato e AAs como
lisina e leucina. Como no podem formar glicose, esses compostos originam corpos cetnicos, por isso
chamados de cetognicos.
REAES DA GLICONEOGNESE
- Trs reaes da gliclise so essencialmente irreversveis e no
podem ser utilizadas na gliconeognese: a converso de glicose em
glicose-6-fosfato pela hexocinase, a fosforilao da frutose-6-fosfato
em frutose-1,6-bifosfato pela FPK-1 e a converso de
fosfoenolpiruvato em piruvato pela piruvato-cinase. Nas clulas, essas
trs reaes so caracterizadas por uma grande variao negativa da
energia livre, enquanto outras reaes glicolticas tm DG prximo de
zero.
- Na gliconeognese, as trs etapas irreversveis so contornadas
por um grupo distinto de enzimas, catalisando reaes
suficientemente exergnicas para serem efetivamente irreversveis no
sentido da sntese de glicose. Assim, tanto a gliclise quanto a
gliconeognese so processos irreversveis nas clulas.

CARBOXILAO DO PIRUVATO
- A primeira reao de contorno da gliconeognese a converso
de piruvato em fosfoenolpiruvato (PEP). A fosforilao do piruvato
alcanada por uma sequncia de reaes de desvio que, em
eucariotos, requer enzimas existentes tanto no citosol como nas
mitocndrias.
- O piruvato primeiro transportado do citosol para a mitocndria
ou gerado dentro da mitocndria a partir da transaminao da
alanina. A seguir, a piruvato-carboxilase, uma enzima mitocondrial
que requer a coenzima biotina, converte o piruvato a oxaloacetato.
- A reao ocorre em duas fases, em dois stios diferentes da
enzima. No stio cataltico 1, o on bicarbonato convertido a CO2 com
gasto de ATP. Em seguida, o CO2 reage com a biotina, formando
carboxibiotinil-enzima. O brao longo composto pela biotina e a cadeia
lateral da Lys transporta o CO2 da carboxibiotinil-enzima para o stio
cataltico 2 na superfcie da enzima, onde o CO2 liberado e reage com
o piruvato, formando oxaloacetato e regenerando o complexo biotinil-
enzima.
- A clivagem de um fosfato de alta energia do ATP impulsiona a
formao do intermedirio enzima-biotina-CO2.

- Essa reao ocorre na mitocndria de clulas hepticas e renais e tem dois propsitos: fornecer um
substrato importante para a gliconeognese e fornecer OAA, que pode repor os intermedirios do ciclo do
cido ctrico. As clulas musculares tambm contm piruvato-carboxilase, associando o uso de OAA mais com
a reposio de intermedirios que com a sntese de glicose.
- Regulao alostrica: A piruvato-carboxilase ativada alostericamente pela acetil-CoA. Nveis
elevados de acetil-CoA podem sinalizar um de diversos estados metablicos nos quais necessria uma
sntese aumentada de oxaloacetato (durante jejum, quando OAA utilizado na sntese de glicose no fgado e
rim). Por outro lado, com nvel de acetil-CoA, a piruvato-carboxilase encontra-se bastante inativada,
fazendo com que o piruvato seja oxidado pela piruvato-desidrogenase para produo de acetil-CoA, que pode
ser oxidada posteriormente pelo ciclo de Krebs.
- Como a membrana mitocondrial no tem
transportador para o oxaloacetato, antes de ser
exportado para o citosol o oxaloacetato formado a
partir do piruvato deve ser reduzido a malato pela
malato-desidrogenase mitocondrial, com o consumo
de NADH:
OAA + NADH + H+ L-malato + NAD+
- O malato deixa a mitocndria por meio de um
transportador especfico presente na membrana
mitocondrial interna, e no citosol ele reoxidado a
oxaloacetato, com a produo de NADH citoslico:
Malato + NAD+ OAA + NADH + H+
- O oxaloacetato ento convertido a PEP pela PEP-
carboxicinase. Esta reao dependente de Mg2+ e
requer GTP como doador de grupo fosforil (a reao
utiliza energia da hidrlise do GTP):
OAA + GTP PEP + GDP
- A reao reversvel em condies intracelulares;
a formao de um composto de fosfato de alta energia
(PEP) balanceada pela hidrlise de outro (GTP).
- Equao global:
Piruvato + ATP + GTP + HCO3- PEP + ADP + GDP + Pi + CO2

- Dois grupos fosfato de alta energia (um do ATP e um do GTP), cada um rendendo em torno de 50
kJ/mol em condies celulares, devem ser gastos para fosforilar uma molcula de piruvato a PEP. Ao contrrio,
quando PEP convertido a piruvato durante a gliclise, apenas um ATP gerado a partir de ADP.
- Embora a variao da energia livre padro da via de duas etapas de converso do piruvato a PEP seja
de 0,9kJ/mol, a variao real (calculada pela medida das concentraes celulares dos intermedirios)
altamente negativa. Assim, o consumo rpido do PEP em outras reaes, mantendo sua concentrao
relativamente baixa, contribui para tornar esse 1 contorno efetivamente irreversvel na clula.
- O CO2 adicionado ao piruvato na etapa catalisada pela piruvato-carboxilase a mesma molcula
perdida na reao da PEP-carboxicinase.
- Um segundo contorno predomina quando o lactato o precursor glicognico. Essa via faz uso do
lactato produzido pela gliclise nos eritrcitos ou no msculo em anaerobiose.
- A converso de lactato em piruvato no citosol de hepatcitos gera NADH, e a exportao de
equivalentes redutores (como malato) da mitocndria consequentemente desnecessria. Assim, o lactato
oxidado a piruvato, que segue a via normal para produzir OAA, mas esse OAA convertido em PEP dentro da
mitocndria pela PEP-carboxicinase mitocondrial.
DESFOSFORILAO DA FRUTOSE-1,6-BIFOSFATO
- A hidrlise essencialmente irreversvel da frutose-1,6-bifosfato contorna a reao irreversvel da
PFK-1 e fornece uma via energeticamente favorvel para formao de frutose-6-fosfato. Essa reao
catalisada pela frutose-1,6-bifosfatase (FBPase-1), que hidrolisa o fosfato em C-1 (no a transferncia do
grupo fosforil para o ADP).
- A FBPase-1 inibida por altos nveis de AMP (sinaliza baixa energia na clula). Altos nveis de ATP e
baixas concentraes de AMP, por sua vez, estimulam a gliconeognese.
- A FBPase-1 inibida por frutose-2,6-bifosfato, efetor alostrico cuja concentrao influenciada
pelos nveis de glucagon/insulina circulantes.

DESFOSFORILAO DA GLICOSE-6-FOSFATO
- O inverso da reao da hexocinase exigiria a transferncia de um grupo fosforil da glicose-6-fosfato
para ADP, formando ATP, reao energeticamente desfavorvel. A reao catalisada pela glicose-6-fosfatase
no requer a sntese de ATP, sendo a hidrlise simples de uma ligao ster fosfato:
Glicose-6-fosfato + H2O glicose + Pi
- O fgado e o rim so os nicos rgos que liberam glicose livre a partir de glicose-6-fosfato.
- Essa enzima ativada por Mg2+ encontrada no lmen do retculo endoplasmtico de hepatcitos, de
clulas renais e das clulas epiteliais do intestino delgado, mas no encontrada em outros tecidos, que so,
portanto, incapazes de fornecer glicose para o sangue.
- Se outros tecidos tivessem a glicose-6-fosfatase, essa atividade enzimtica hidrolisaria a glicose-6-
fosfato necessria para a gliclise nesses tecidos. A glicose produzida pela gliconeognese no fgado, nos rins
ou ingerida na dieta entregue a esses outros tecidos, inclusive o crebro e os msculos, pela corrente
sangunea.
- Transportadores especficos so responsveis pela liberao de glicose e fosfato de volta ao citosol e,
nos hepatcitos, para o sangue.
- Msculo no tem glicose-6-fosfatase no libera glicose para o sangue.
- Reao global:
2 piruvato + 4 ATP + 2 GTP + 2 NADH + 2H+ + 4 H2O glicose + 4 ADP + 2 GDP + 6 Pi + 2 NAD+
- Para cada molcula de glicose formada a partir do piruvato, 6 grupos fosfato de alta energia so
consumidos, 4 na forma de ATP e 2 na forma de GTP. Alm disso, duas molculas de NADH so necessrias
para a reduo de duas molculas de 1,3-bifosfoglicerato.
- A sntese de glicose a partir de piruvato um processo relativamente dispendioso. A maior parte
desse alto custo energtico necessria para assegurar a irreversibilidade da gliconeognese.
- Uma segunda vantagem em investir energia para converter piruvato em glicose que se o piruvato
fosse excretado, seu considervel potencial para formao de ATP pela completa oxidao aerbia seria
perdido (mais de 10 ATP so formados por piruvato).

REGULAO DA GLICONEOGNESE
- Glucagon: estimula a gliconeognese por 3 mecanismos:
- Alteraes em efetores alostricos: O glucagon estimula a adenilato-ciclase do fgado a
sintetizar 3,5-AMP cclico (cAMP) a partir de ATP. O AMP cclico ativa a protena-cinase dependente de cAMP,
a qual transfere um grupo fosforil do ATP para a protena bifuncional PFK-2/FBPase-2. A fosforilao desta
protena aumenta sua atividade de FBPase-2 e inibe a atividade de PFK-2. Dessa forma, o glucagon reduz o
nvel celular de frutose-2,6-bifosfato, inibindo a gliclise e estimulando a gliconeognese.
*** Insulina tem efeito oposto: estimula atividade de uma fosfoprotena-fosfatase que
remove o grupo fosforil adicionado, inibindo sua atividade de FBPase-2 e aumentando atividade de PFK-2,
aumentando o nvel de frutose-2,6-bifosfato, estimulando a gliclise e inibindo a gliconeognese.
- Modificao da atividade enzimtica por ligao covalente:
- O glucagon, via aumento dos nveis de AMPc e na atividade da protena-cinase
dependente de AMPc, estimula a converso da piruvato-cinase em sua forma inativa (fosforilada), diminuindo
a converso do PEP a piruvato, tendo o efeito de redirecionar o PEP para a sntese de glicose.
*** Insulina tem o efeito oposto: estimula atividade de fosfoprotena-fosfatase, que
remove o grupo fosforil adicionado, estimulando a converso da piruvato-cinase na sua forma ativa
(desfosforilada).
- Induo da sntese de enzimas:
- Glucagon aumenta a transcrio do gene da PEP-carboxicinase, aumentando assim a
disponibilidade de atividade enzimtica no momento em que os nveis de seu substrato aumentam, com o
jejum.
*** Insulina tem o efeito oposto: causa diminuio na transcrio do RNAm para essa
enzima.
- Disponibilidade de substrato: Nveis diminudos de insulina favorecem a mobilizao de aminocidos
glicognicos a partir de protenas musculares e fornecem esqueletos carbonados para a gliconeognese.
- Ativao alostrica pela acetil-CoA: Durante jejum, ocorre a ativao alostrica da piruvato-
carboxilase heptica pela acetil-CoA. Como resultado da liplise excessiva no tecido adiposo, o fgado
inundado com cidos graxos. Como a formao de acetil-CoA (atravs da -oxidao desses cidos graxos)
maior que a capacidade do fgado de oxid-la a CO2 e H2O, ocorre seu acmulo e ativao da piruvato-
carboxilase. ** Acetil-CoA inibe a piruvato-desidrogenase (piruvato acetil-CoA), estimulando a cinase a
fosforilar o complexo, o que causa a inativao. Desse modo, ela pode redirecionar o piruvato no sentido da
gliconeognese, removendo-o da oxidao no ciclo de Krebs.
- Inibio alostrica pelo AMP: A FBPase-1 inibida por AMP composto que ativa a FPK-1. Assim, um
aumento no AMP estimula vias que oxidam nutrientes, fornecendo energia para a clula.
METABOLISMO DO GLICOGNIO
- Nos organismos, o excesso de glicose convertido em formas polimricas de armazenamento
glicognio nos vertebrados e em muitos microrganismos, amido nas plantas.
- Nos vertebrados, o glicognio encontrado principalmente no fgado e no msculo esqueltico,
podendo representar at 10% do peso do fgado (~100g) e 1 a 2% do peso do msculo (~400g). Se toda essa
glicose fosse dissolvida no citosol de um hepatcito, a concentrao produzida seria suficiente para influenciar
nas propriedades osmticas da clula.
- O glicognio do msculo fornece uma fonte de energia rpida para o metabolismo aerbio e
anaerbio; pode ser gasto em menos de uma hora durante atividade intensa.
- O glicognio heptico serve como um reservatrio de glicose para os outros tecidos quando no h
glicose disponvel (entre as refeies ou no jejum); isto especialmente importante para os neurnios do
crebro, que no podem usar cidos graxos como combustvel.
- Os grnulos de glicognio so agregados complexos de glicognio mais as enzimas que os sintetizam
e os degradam, assim como a maquinaria de regulao dessas enzimas. Os mecanismos gerais de
armazenamento e mobilizao do glicognio so os mesmos no msculo e no fgado, mas as enzimas diferem
em aspectos sutis, mas importantes, que refletem os papis diferentes do glicognio nesses dois tecidos.

ESTRUTURA E FUNO
- Glicognio muscular: reserva de combustvel para sntese de ATP durante a contrao muscular.
- Glicognio heptico: manter a concentrao da glicose sangunea (principalmente no incio do jejum).
- Estrutura: Homopolissacardeo de cadeia ramificada formado, exclusivamente, por -D-glicose.
- Unio glicosdica primria uma ligao (14). Aps 8~10 resduos glicosil, h uma
ramificao (16). Essas molculas existem em grnulos citoplasmticos diferenciados, que contm a
maioria das enzimas necessrias para a sntese e para a degradao de glicognio.
- Variaes do estoque:
- Glicognio heptico aumenta durante o estado alimentado e diminui durante o jejum.
- Glicognio muscular no afetado por perodos curtos de jejum, s diminui moderadamente
em jejuns prolongados (semanas). sintetizado para repor os estoques dos msculos aps terem esgotado
por gastos extenuantes.

GLICOGNESE SNTESE DE UDP-GLICOSE


- O ponto de partida para a sntese de glicognio a glicose-6-fosfato (derivada da glicose livre que
fosforilada pelas hexocinases):
D-glicose + ATP D-glicose-6-fosfato + ADP
*** Parte da glicose ingerida pode seguir outra via: Glicose -> eritrcitos -> Gliclise -> formao de
lactato -> lactato no fgado -> glicose-6-fosfato (gliconeognese).
- Para iniciar a sntese do glicognio, a glicose-6-fosfato convertida em glicose-1-fosfato na reao da
fosfoglicomutase:
Glicose-6-fosfato Glicose-1-fosfato
- O produto desta reao convertido em UDP-glicose pela ao da UDP-glicose-pirofosforilase:
Glicose-1-fosfato + UTP UDP-glicose + PPi
- A -D-glicose ligada ao difosfato de uridina (UDP) a fonte de todos os resduos glicosil que so
adicionados molcula de glicognio em formao.
- Apesar da enzima ser denominada pela reao inversa, a hidrlise, pela pirofosfatase inorgnica, da
ligao rica em energia do pirofosfato (PPi) em dois fosfatos inorgnicos (Pi) impulsiona a reao para frente,
garantindo que a reao de sntese se d na direo da produo de UDP-glicose.
ALONGAMENTO DAS CADEIAS PELA GLICOGNIO-SINTASE
- A UDP-glicose o doador imediato dos resduos de glicose na reao catalisada pela glicognio-
sintase, envolvendo a transferncia de glicose para a extremidade no-redutora da cadeia em crescimento,
formando uma nova ligao glicosdica entre a hidroxila do carbono anmero (C-1) da glicose ativada (UDP-
glicose) e a hidroxila de C-4 do resduo glicosil aceptor.

- O UDP liberado pela formao da nova ligao (14) pode ser convertido novamente em UTP pela
nucleosdeo-difosfato-cinase:
UDP + ATP UTP + ADP
SNTESE DE SEGMENTO INICIAL PARA INICIAR A SNTESE DE GLICOGNIO
- A glicognio-sintase, formadora da ligao (14), no pode iniciar a sntese da cadeia usando glicose
livre como aceptora de uma molcula de glicose a partir de UDP-glicose. Assim, ela necessita de um iniciador,
geralmente uma cadeia poliglicosdica em (14) ou uma ramificao que tenha, pelo menos, oito resduos
de glicose.
- Na ausncia de um fragmento de glicognio, a protena glicogenina pode servir como aceptora de
resduos de glicose. A glicogenina ao mesmo tempo o iniciador, sobre o qual so montadas novas cadeias, e
a enzima que catalisa essa montagem.
- A primeira etapa na sntese de uma nova molcula de glicognio a transferncia de um resduo de
glicose da UDP-glicose para o grupo hidroxil de uma tirosina especfica da glicogenina, catalisada pela atividade
glicosil-transferase intrnseca da protena. A cadeia nascente se alonga pela adio sequencial de mais sete
resduos de glicose, cada um derivado de uma UDP-glicose; as reaes so catalisadas pela atividade de
extenso de cadeia da glicogenina. Neste ponto, a glicognio-sintase age, alongando ainda mais a cadeia de
glicognio.
- A glicogenina continua associada molcula completa de glicognio, unida covalentemente nica
extremidade redutora da molcula.
- As consequncias clnicas de uma mutao no gene da glicogenina que suprime essa atividade de
polimerizao da protena incluem fadiga muscular e fraqueza, depleo do glicognio no fgado e batimento
cardaco irregular (arritmia cardaca).
FORMAO DAS RAMIFICAES NO GLICOGNIO
- Se nenhuma outra enzima de sntese agir sobre a cadeia, a estrutura resultante ser uma molcula
linear de resduos de glicosil, j que a glicognio-sintase no pode formar as ligaes (16) encontradas nos
pontos de ramificao do glicognio.
- A enzima de ramificao do glicognio (glicosil-(46)-transferase) catalisa a transferncia de um
fragmento terminal de 6 a 7 resduos de glicose da extremidade no redutora de uma ramificao de
glicognio, contendo pelo menos 11 resduos, para o grupo hidroxil C-6 de um resduo de glicose em uma
posio mais interna da mesma ou de outra cadeia de glicognio, criando assim uma nova ramificao.
Resduos adicionais de glicose podem ser ligados nova ramificao pela glicognio-sintase.
- O efeito biolgico da ramificao tornar a molcula mais solvel e aumentar o nmero de stios
acessveis glicognio-fosforilase e glicognio-sintase, as quais agem somente nas extremidades no
redutoras, acelerando a velocidade de degradao e sntese do glicognio.

GLICOGENLISE
- Quando o glicognio degradado, o produto primrio a glicose-1-fosfato, obtida pela clivagem das
ligaes glicosdicas (14). Alm disso, glicose livre liberada a partir de cada resduo glicosil unido por
ligaes (16).
ENCURTAMENTO DAS CADEIAS
- A glicognio-fosforilase catalisa a reao na qual uma ligao glicosdica (14) entre dois resduos
de glicose em uma extremidade no redutora do glicognio atacada por um fosfato inorgnico (Pi),
removendo o resduo terminal na forma de -D-glicose-1-fosfato, por meio de fosforlise simples. (Na
fosforlise, parte da energia da ligao glicosdica preservada pela formao do ster de fosfato, glicose-1-
fosfato).
- O piridoxal-fosfato, que tem papel
mais caracterstico como cofator no
metabolismo dos AA, est ligado
covalentemente enzima, tambm como
cofator. Seu grupo fosfato atua como
catalisador cido geral, promovendo o
ataque pelo Pi sobre a ligao glicosdica.
- A glicognio-fosforilase age
repetidamente sobre as extremidades no
redutoras das ramificaes do glicognio
at que alcance um ponto a 4 resduos de
glicose de um ponto de ramificao
(16), onde interrompe sua ao.

REMOO DAS RAMIFICAES


- Os resduos prximos a uma ramificao so
removidos por um processo em duas etapas que requer a
enzima de desramificao bifuncional. (oligo (16)
(14) glican-transferase).
- 1) Na primeira, a atividade de transferase da
enzima oligo (14)(14)glican-transferase remove
um bloco de trs resduos de glicose da ramificao para
uma extremidade no redutora prxima, qual religado
por uma ligao (14).
- 2) O resduo remanescente no ponto de
ramificao, em ligao (16), ento liberado como
glicose livre pela atividade de glicosidase (16) da
enzima de desramificao
- A cadeia glicosdica est, novamente, disponvel
para a degradao pela glicognio-fosforilase, at que
sejam alcanadas 4 unidades glicosil antes da prxima
ramificao.

- A glicose-1-fosfato, o produto final da reao da


glicognio-fosforilase, convertida em glicose-6-fosfato
pela fosfoglicomutase.
- A glicose-6-fosfato formada no msculo
esqueltico a partir do glicognio pode entrar na gliclise
e serve como fonte de energia para a contrao muscular.
- No fgado, a degradao do glicognio serve a um propsito diferente: liberar glicose para o sangue
quando o nvel de glicose sangunea diminui, como acontece entre as refeies. Isso requer a presena da
enzima glicose-6-fosfatase. Assim, a glicose-6-fosfato transposta ao retculo endoplasmtico pela glicose-6-
fosfato-translocase. Nessa estrutura, convertida em glicose pela glicose-6-fosfatase. Os produtos
resultantes, Pi e glicose, so transportados de volta para o citosol por dois transportadores diferentes (T2 e
T3), e a glicose deixa o hepatcito pelo transportador GLUT2 na membrana plasmtica.

DEGRADAO LISOSSMICA
- Pequena quantidade de glicognio degradada pela enzima lisossmica (14)-glicosidase,
produzindo glicose livre. A deficincia dessa enzima gera acmulo de glicognio em vacolos no citosol,
resultando na doena de armazenamento do glicognio tipo II (doena de Pompe). generalizada (afeta
principalmente fgado, corao e msculos), causa cardiomegalia grave e morte precoce associada falha do
corao. Causa tambm distrofia muscular.

REGULAO DA SNTESE E DEGRADAO DO GLICOGNIO


- Regulao da sntese e degradao de glicognio no estado alimentado:
- No estado alimentado, a glicognio-sintase alostericamente ativada por glicose-6-fosfato
quando esta estiver presente em concentraes elevadas. Em contraste, a glicognio-fosforilase inibida,
bem como por ATP (sinal de alto nvel energtico na clula).
*** No fgado, a glicose inibe a glicognio-fosforilase.
- Ativao da degradao do glicognio no msculo pelo clcio:
- A contrao muscular causa liberao de Ca2+ para o sarcoplasma das clulas musculares
Ca2+ liga-se calmodulina 4 ons Ca2+ ligados calmodulina causam mudana conformacional, de modo
que o complexo Ca2+ - calmodulina ativado pode associar a molculas proteicas, funcionando como
subunidade essencial O complexo ativa a fosforilase-cinase (torna desnecessrio a fosforilao feita pela
protena cinase dependente).
- Quando o msculo relaxa, o Ca2+ retorna ao retculo sarcoplasmtico e a fosforilase-cinase
tonar-se inativa.
- Ativao da degradao no msculo pelo AMP: A glicognio-fosforilase muscular est ativa na
presena das altas concentraes de AMP que ocorrem no msculo, em condies extremas de anxia e de
depleo de ATP. O AMP se liga forma inativa da glicognio-fosforilase, causando sua ativao sem
fosforilao.
- Ativao da degradao pela via direcionada do AMPc:
- A ligao de glucagon (ou adrenalina) a seus receptores especficos ativa a protena-cinase
dependente de AMPc.
- Estmulo da adenilato-ciclase a aumentar os nveis de AMPc.
- AMPc ativa a protena-cinase, ligando-se s subunidades regulatrias e liberando as
subunidades catalticas.
- A glicognio-fosforilase-cinase pode ser ativa (ou A, fosforilada) ou inativa (ou B,
desfosforilada).
- A protena-cinase dependente fosforila a forma B, ativando a fosforilase-cinase A. (** a enzima
fosforilada pode ser desativada por meio da remoo hidroltica do fosfato pela protena-fosfatase 1).
- A glicognio-fosforilase pode ser ativa (ou A, fosforilada) ou inativa (ou B, desfosforilada).
- A fosforilase-cinase A fosforila a glicognio-fosforilase, gerando sua forma ativa (A). A forma
ativa, enfim, catalisa a degradao do glicognio. (** a enzima fosforilada pode ser desativada por meio da
remoo hidroltica do fosfato pela protena-fosfatase 1).
- O grande nmero de etapas sequenciais serve para amplificar o efeito do sinal hormonal
(muitas protenas-cinase ativam muitas fosforilases-cinase, que ativam muitas glicognio-fosforilase).
- Inibio da sntese de glicognio por uma via direcionada pelo AMPc:
- Glicognio-sintase pode ser ativa (ou A, desfosforilada) ou inativa (ou B, fosforilada).
- A associao de glucagon ou adrenalina nos hepatcitos (ou adrenalina, nos msculos) resulta
na ativao da adenilato-ciclase.
- Adenilato-ciclase aumenta os nveis de AMPc.
- Altos nveis de AMPc aumentam a atividade da protena-cinase dependente de AMPc.
- A protena-cinase fosforila a glicognio-sintase, formando a forma B (inativa). Assim, a sntese
de glicognio inibida.
*** A insulina, alm de diminuir os nveis de AMPc por processo inverso, estimula a remoo
hidroltica do fosfato pela protena-fosfatase 1, favorecendo a forma A (ativa, desfosforilada), estimulando a
glicognese.
PROBLEMA 3.1
CIDOS GRAXOS
- No organismo: livre (no-esterificado) ou como steres de acila (triacilgliceris).
- Em todos os tecidos: nveis de cidos graxos livres.
- c. graxos livres no plasma (transportados por albumina srica) originados de triacilgliceris do
tec. adiposo ou das lipoprotenas da circulao para o stio de consumo (outros tecidos).
- Os livres podem ser oxidados por muitos tecidos (fgado, msculo) para produo de energia.
- So tambm componentes estruturais dos lipdios de membrana (fosfolpides, glicolipdios).
- Podem se ligar a certas protenas intracelulares, aumentando a sua capacidade de se associar s
membranas.
- So precursores de prostaglandinas: c linoleico c araquidnico.
- c. graxos esterificados (triacilgliceris) principal reserva energtica do organismo.

CIDOS GRAXOS ESTRUTURA


- Consiste em uma cadeia hidrofbica de hidrocarboneto com grupo carboxila terminal com pKa
~4,8. Em pH fisiolgico, o grupo carboxila terminal ioniza, tornando-se COO-, que hidroflico natureza
anfiptica.
- Nos c. graxos de cadeia longa (AGCLs), a parte hidrofbica predominante solubilidade em
gua precisam ser transportados pela circulao associados a protenas.
- No plasma, mais de 90%: steres (triacilgliceris, steres de colesterol, fosfolpides), nas partculas
de lipoprotenas da circulao. Os no-esterificados so transportados em associao com a albumina.
- Saturao dos c. graxos: podem ser saturados ou mono/poli insaturados.
- Quando insaturado, quase sempre se encontram na configurao cis, que causa uma
dobra/curvatura no cido, naquela posio. 2 ou mais duplas espaadas em intervalos de 3 carbonos.
- Em geral, a adio da dupla diminui o ponto de fuso. O aumento do comprimento da
cadeia causa aumento.
* AGCLs em lipdios de membrana presena de duplas manter a fluidez dos lipdios.
- Comprimento da cadeia:
- Os tomos de carbono so numerados. Carbono da carboxila 1
- cido actico (2:0): 2 carbonos na cadeia, 0 ligaes duplas.
- Carbono unido carboxila (C-2) alfa. C-3 (beta), C-4 (gama)...
- Podem ser contados a partir do carbono (metila) terminal mega. Ex.: cido linoleico
(18:2, 9, 12): mega-6 (as duplas comeam 6 carbonos a partir da metila terminal).
- c. graxos essenciais: cido linoleico (precursor do cido araquidnico, substrato para sntese de
prostaglandinas) e cido linolnico (18:3, 9, 12, 15; precursor de outros c. graxos mega-3 importantes
para o crescimento e o desenvolvimento).

BIOSSNTESE DE CIDOS GRAXOS


PRODUO DE ACETIL-CoA CITOSLICA
- A primeira etapa a transferncia de unidades acetato para o citosol, a partir da acetil-CoA
mitocondrial, que pode ser originada: pela oxidao do piruvato, pelo catabolismo de c. graxos, corpos
cetnicos e alguns aminocidos (lisina, isoleucina, leucina, triptofano).
- A CoA no pode atravessar a membrana mitocondrial, apenas a poro acetila, da a necessidade
das converses.
- A acetil-CoA condensa com o oxaloacetato (OAA), pela citrato-sintase, originando citrato.
- O citrato translocado para o citosol, sendo clivado pela ATP-citrato-liase:
Citrato + CoA + ATP OAA + acetil-CoA + ADP + Pi
- O processo acima ocorre quando a [citatro] mitocondrial alta, j que ele um sinalizador de alta
energia. Essa alta se d pela inibio da isocitrato-desidrogenase por ATP, acumulando citrato/isocitrato.
Como necessrio ATP para a sntese de c. graxos, o aumento de ATP/citrato intensifica essa rota.
** Isocitrato-desidrogenase converte isocitrato em -cetoglutarato, no ciclo de Krebs.
FORMAO DA MALONIL-CoA
- A energia para as condensaes carbono-carbono na sntese de c. graxos suprida pela
carboxilao/descarboxilao dos grupos acetila no citosol.
- A formao de malonil-CoA a partir de acetil-CoA um processo irreversvel, catalisado pela acetil-
CoA-carboxilase.
- A reao semelhante converso do piruvato em oxaloacetato (prob. 2.3).
- Primeiramente, um grupo carboxil derivado do bicarbonato transferido para a biotina em uma
reao dependente de ATP (a clivagem do fosfato de alta energia do ATP impulsiona a formao do
intermedirio enzima-biotina-CO2). O grupo biotinila age como transportador temporrio de CO2,
transferindo-o para a acetil-CoA na segunda etapa, gerando malonil-CoA.
- Regulao a curto prazo: Essa carboxilao a etapa limitante na sntese de c. graxos.
- A forma inativa da enzima um protmero (dmero).
- Sofre ativao alostrica por citrato (provoca polimerizao dos dmeros).
- Sofre inativao alostrica por acil-CoA de cadeia longa (produto final da via
despolimerizao).
- Fosforilao reversvel (*semelhante glicognio-sintase): Na presena de adrenalina ou
glucagon, a enzima fosforilada (via AMP e protena-cinase), causando sua inativao. ** A insulina tem
efeito contrrio: induz a protena-fosfatase a hidrolisar o fosfato adicionado, desfosforilando a enzima e
causando sua ativao.
- Regulao de longa durao: Consumo prolongado de uma dieta com excesso de calorias
(principalmente com muitos carboidratos) sntese da enzima, sintetizando mais c. graxos. ** Dieta
com poucas calorias / jejum diminui a sntese da enzima, sintetizando menos c. graxos.

COMPLEXO DA CIDO GRAXO-SINTASE


- As reaes restantes de sntese so catalisadas por uma enzima dimrica multifuncional, a cido
graxo-sintase. As longas cadeias de carbono so construdas por uma sequncia de reaes repetitivas, em
4 etapas.
- Um grupamento acila saturado, produzido em cada srie de reaes em 4 etapas, torna-se o
substrato da condensao subsequente com um grupo malonila ativado. Em cada uma das passagens pelo
ciclo, a cadeia do grupo acila graxo aumenta em dois carbonos.
- Cada monmero da cido graxo-sintase um polipeptdeo multicataltico com 7 diferentes
atividades enzimticas, mais um domnio que se liga covalentemente a uma molcula de 4'-fosfopantetena
- 4-fosfopantetena: derivado do cido pantotnico, carrega unidades acila em seu grupo terminal
tiol (-SH) durante a sntese de c. graxos. tambm componente da acetil-CoA.
- Em procariotos, o domnio 4'-fosfopantetena uma protena separada, chamada protena
carregadora de acilas (ACP).
- Com o sistema da cido graxo-sintase, a sntese dos cidos graxos leva a um nico produto, e no
so liberados intermedirios.
- Quando o comprimento da cadeia atinge 16 carbonos, esse produto (palmitato, 16:0) deixa o ciclo.
- Os carbonos C-16 e C-15 do palmitato so derivados dos tomos de carbono dos grupos metil e
carboxil, respectivamente, de uma acetil-CoA utilizada diretamente para iniciar o sistema.
- Os outros tomos de carbono da cadeia so originados da acetil-CoA via malonil-CoA.
- Ao longo do processo de sntese dos cidos graxos, os intermedirios permanecem
covalentemente ligados como tiosteres a um de dois grupos tiol. Um ponto de ligao o grupo -SH de
um resduo de Cys em um dos domnios da sintase (-cetoacil-ACP-sintase; KS); o outro ponto o grupo -
SH de uma protena transportadora de grupos acila, domnio distinto do mesmo polipeptdeo.
- A hidrlise dos tiosteres altamente exergnica, e a energia liberada ajuda a tornar
termodinamicamente favorveis dois passos distintos da sntese dos cidos graxos (condensao).
- Adio de dois carbonos a uma cadeia acil graxo em
crescimento: uma sequncia de quatro etapas:
- Cada grupo malonila e acetila (ou acilas maiores) ativado por
um tioster que os une cido graxo-sintase. 1) A condensao
de um grupo acila ativado (um grupo acetil da acetil-CoA o
primeiro grupo acila) e dois carbonos derivados da malonil-CoA,
com a eliminao de CO2 do grupo malonila, alonga a cadeia acila
em dois carbonos. O mecanismo da primeira etapa dessa reao
est mostrado para ilustrar o papel da descarboxilao em
facilitar a condensao. O produto -cetnico dessa condensao
, ento, reduzido em trs etapas seguintes praticamente
idnticas s reaes de -oxidao, mas na sequncia inversa;
2) o grupo -cetnico reduzido a um lcool, 3) a eliminao de
H2O cria uma ligao dupla, e 4) a ligao dupla reduzida,
formando o grupo acil graxo saturado correspondente.
(RESUMIDO)
1) Uma molcula de acetato transferida da acetil-CoA para o
grupo -SH da ACP. Domnio: acetil-CoA-ACP-acetiltransacilase.
2) A seguir, esse fragmento de dois carbonos transferido para
um stio temporrio, o grupo tiol de um resduo de cistena da
enzima.
3) A ACP, agora livre, aceita uma unidade de trs carbonos
malonato da malonil-CoA. Domnio: malonil-CoA-ACP-
transacilase.
4) O grupo acetila sobre o resduo de cistena condensa com o
grupo malonila sobre a ACP, liberando o CO2. O resultado um
conjunto de 4 carbonos, unido ao domnio ACP. A perda de
energia livre pela descarboxilao favorece a reao. Domnio: 3-
cetoacil-ACP-sintase.
5) O grupo cetona reduzido a lcool. Domnio: 3-cetoacil-ACP
redutase.
6) Uma molcula de gua removida para introduzir uma
ligao dupla entre os carbonos 2 e 3 (os carbonos e ). Domnio: 3-hidroxiacil-ACP-desidratase.
7) A ligao dupla reduzida. Domnio: enoil-AC-redutase.

- O resultado dessas sete etapas a produo de um composto com quatro carbonos (butiril), cujos
trs carbonos terminais so totalmente saturados e que permanece unido ACP.
- As etapas so repetidas: Transferncia da cadeia butiril da ACP para o resduo de cistena (2);
Ligao da molcula de malonato ACP (3); Condensao das duas molculas, liberando CO2 (4).
- O grupo carbonila do carbono (carbono 3 - o terceiro carbono a partir do enxofre) ento
reduzido (5), desidratado (6) e novamente reduzido (7), gerando hexanoil-ACP.
- Esse ciclo de reaes repetido mais cinco vezes, cada vez incorporando uma unidade de dois
carbonos (derivada do malonil-CoA) na extremidade carboxila da cadeia do cido graxo em crescimento.
- Quando o cido graxo atinge o comprimento de 16 carbonos, o processo de sntese terminado
com palmitoil-S-ACP.
- A palmitoil-tioesterase cliva a ligao tioster, produzindo uma molcula de palmitato plenamente
saturada (16:0).
(DETALHADO)
1) Carregamento correto dos dois grupos tiis do complexo por acilas: Primeiro, o grupo acetila da
acetil-CoA transferido para a ACP, em uma reao catalisada pelo domnio malonil/acetil-CoA-ACP-
transferase.
2) O grupo acetila , ento, transferido para o grupo -SH da Cys da -cetoacil-ACP-sintase (KS).
3) A transferncia do grupo malonila da malonil-CoA para o grupo -SH da ACP (que est livre),
tambm catalisada pela malonil/acetil-CoA-ACP-transferase.
- No complexo sintase carregado, os grupos acetila e malonila so ativados para o processo de
alongamento da cadeia.
4) A primeira reao na formao da cadeia de um cido graxo uma condensao envolvendo os
grupos acetila e malonila ativados, formando acetoacetil-ACP, grupo acetoacetil ligado ACP pelo grupo
-SH da fosfopantetena; simultaneamente, uma molcula de CO2 produzida.
- Nesta reao, catalisada pela -cetoacil-ACP-sintase, o grupamento acetil transferido do
grupo -SH da Cys da enzima para o grupo malonila ligado ao grupo SH da ACP, tornando-se a unidade de
dois carbonos metil-terminal do novo grupo acetoacetila.
- O tomo de carbono do CO2 formado o mesmo que foi introduzido na malonil-CoA a
partir do HCO3- pela reao da acetil-CoA-carboxilase. Ou seja, a ligao covalente do CO2 transitria.
** O uso de grupos malonila ativados em vez de grupos acetil o que torna as reaes de
condensao termodinamicamente favorveis. (O carbono metileno (C-2) do grupo malonila, situado entre
os carbonos da carbonila e da carboxila, forma um bom nuclefilo. Na etapa de condensao, a
descarboxilao do grupo malonila facilita o ataque nucleoflico do carbono metileno sobre a ligao
tioster entre o grupo acetil e a -cetoacil-ACP-sintase, deslocando o grupo -SH da enzima.
- A energia extra necessria para tornar a sntese dos cidos graxos favorvel fornecida pelo ATP
utilizado na sntese de malonil-CoA a partir de acetil-CoA e HCO3.
5) O acetoacetil-ACP formado sofre reduo do grupo carbonil em C-3, formando d--hidroxibutiril-
ACP. Essa reao catalisada pela -cetoacil-ACP-redutase (KR) e o doador de eltrons o NADPH.
6) Os elementos da gua so removidos dos carbonos C-2 e C-3 da d--hidroxibutiril-ACP, formando
uma ligao dupla no produto, trans-butenoil-ACP. A enzima que catalisa essa desidratao a
-hidroxiacil-ACP-desidratase (DH).
7) A ligao dupla da trans-butenoil-ACP reduzida (saturada), formando butiril-ACP pela ao da
enzima enoil-ACP-redutase (ER). NADPH o doador de eltrons.
- Iniciando outro ciclo, outro grupo malonila liga-se ao grupo -SH da fosfopantetena da ACP, agora
desocupado. A condensao ocorre medida que o grupo butirila, atuando como o grupo acetil no 1 ciclo,
ligado aos dois tomos de carbono do grupo malonil-ACP, com a consequente perda de CO2.
- O produto dessa condensao um grupo acila com 6 carbonos, covalentemente ligado ao grupo
-SH da fosfopantetena. Seu grupo -cetnico reduzido nas trs etapas seguintes do ciclo da sintase,
formando o grupo acila saturado, exatamente como no 1 ciclo de reaes neste caso formando o
produto de 6 carbonos.
- Sete ciclos de condensao e reduo produzem o grupo palmitoila de 16 carbonos saturados,
ainda ligado ACP. O palmitato liberado da ACP pela ao de uma atividade hidroltica (tioesterase; TE)
da protena multifuncional.

- Considerando o processo global em duas etapas: (1) formao do malonil-CoA e (2) ciclos de
condensao/reduo.
7 acetil-CoA + 7 CO2 + 7 ATP 7 malonil-CoA + 7 ADP + 7 Pi

Acetil-CoA + 7 malonil-CoA + 14 NADPH + 14H+ palmitato + 7 CO2 + 8 CoA + 6 H2O + 14 NADP+

Soma: 8 acetil-CoA + 7 ATP + 14 NADPH + 14H+ palmitato + 8 CoA + 7ADP + 7 Pi + 6 H2O + 14 NADP+

- So formadas apenas 6 molculas de gua. Uma utilizada para hidrolisar a ligao tioster entre
o produto palmitato e a enzima.

- Assim, a biossntese dos cidos graxos como o palmitato requer acetil-CoA e o fornecimento de
energia qumica de duas formas: o potencial de transferncia de grupos do ATP e o poder redutor do
NADPH.
- O ATP necessrio para ligar o CO2 acetil-CoA formando malonil-CoA; as molculas de NADPH
so necessrias para reduzir o grupo -ceto e a ligao dupla.

FONTES DE NADPH
- Via das pentoses o maior fornecedor. 2 NADPH por molcula de glicose que entra na via.
- Converso citoslica do malato a piruvato (malato oxidado e descarboxilado pela enzima mlica
citoslica, ou malato-desidrogenase dependente de NADP+), produz NADPH e CO2:
OAA + NADH + H+ Malato + NAD+
Malato + NADP+ Piruvato + CO2 + NADPH + H+

ELONGAO POSTERIOR DA CADEIA


- O palmitato, produto principal do sistema da cido graxo-sintase nas clulas animais, o precursor
de outros cidos graxos de cadeia longa. Ele deve ser alongado formando estearato (18:0) ou cidos graxos
saturados ainda maiores pela adio de grupos acetil, pela ao do sistema de alongamento de cidos
graxos (sistema de enzimas separadas, em vez de uma enzima multifuncional) presente no retculo
endoplasmtico (RE) liso e na mitocndria.
- O sistema de alongamento mais ativo do RE alonga a cadeia de 16 carbonos da palmitoil-CoA em
dois tomos de carbono, formando estearoil-CoA.
- Malonil-CoA o doador de 2 carbonos e NADPH fornece os eltrons.
- Embora diferentes sistemas enzimticos estejam envolvidos e a coenzima A seja o transportador
de grupos acila, em lugar da ACP, o mecanismo de alongamento no RE idntico quele utilizado na sntese
do palmitato: doao de dois carbonos (um sai como CO2) a partir da malonil-CoA, seguindo-se reduo,
desidratao e nova reduo do produto saturado de 18 carbonos, a estearoil-CoA.
- O encfalo possui capacidade adicional de elongao, permitindo a produo dos cidos graxos de
cadeia muito longa (acima de 22 carbonos), necessrios para a sntese de lipdeos do sistema nervoso
central.
DESSATURAO DAS CADEIAS

- Enzimas (dessaturases) tambm presentes no REL so responsveis pela dessaturao dos cidos
graxos de cadeia longa (ou seja, pela adio de ligaes duplas na configurao cis).
- As reaes de dessaturao requerem NADH, citocromo b5 e sua redutase (citocromo b5 redutase)
ligada ao FAD.
- A ligao dupla (tipicamente inserida entre C-9 e C-10, produzindo principalmente 18:1(9) e pouca
quantidade de 16:1(9)) introduzida na cadeia do cido graxo por uma reao oxidativa catalisada pela
acil-CoA graxo-dessaturase, uma oxidase de funo mista.
- Dois substratos diferentes, o cido graxo e o NADPH, sofrem oxidao simultaneamente pelo O2,
perdendo dois eltrons.
- c. graxos poli-insaturados podem ser produzidos por dessaturao adicional + elongao.
** Humanos tm dessaturases para C-9, C-6, C-5 e C-4, mas no tm capacidade para introduzir a
partir de C-10 at o carbono terminal (mega) da cadeia. Da a razo dos cidos linoleico (mega 6) e
linolnico (mega 3) serem nutricionalmente essenciais.

- A maior parte dos cidos graxos sintetizados ou ingeridos por um organismo possui um de dois
destinos: a incorporao em triacilgliceris para o armazenamento de energia metablica ou a
incorporao nos componentes fosfolipdicos da membrana.
- As duas vias iniciam no mesmo ponto: a formao de steres acil graxo de glicerol.

ARMAZENAMENTO DE C. GRAXOS EM TRIACILGLICERIS (TAGs)


- Mono/di/tri acilgliceris consistem em 1, 2 ou 3 molculas de c. graxos esterificando uma
molcula de glicerol.
- Os c. graxos esterificados por meio de seus grupos carboxila, resultando na perda da carga
negativa e na formao de um lipdio neutro.
- Estrutura: Os 3 c. graxos que esterificam o glicerol normalmente so de tipos diferentes. Em geral,
o c. graxo de C-1 saturado, em C-2 insaturado e em C-3, pode ser saturado ou insaturado. A presena
de insaturaes diminui a temperatura de fuso do lipdio.
- Armazenamento: Como os TAGs so pouco solveis em gua e no formam micelas estveis por
contra prpria, eles coalescem dentro dos adipcitos, formando gotculas oleosas. Essas gotculas no citosol
so a maior reserva energtica do organismo.

SNTESE DE GLICEROL-FOSFATO
- O glicerol-fosfato o aceptor inicial dos c. graxos durante a sntese de TAG.
- A grande maioria do glicerol-3-fosfato derivada do intermedirio glicoltico di-hidroxiacetona-
fosfato (DHAP) pela ao da glicerol-3-fosfato-desidrogenase citoslica ligada ao NAD (reao de reduo).
Ocorre no fgado e no tecido adiposo.
- No fgado e nos rins, uma pequena parte do glicerol-3-fosfato tambm produzida a partir do
glicerol livre pela ao da glicerol-cinase. *adipcitos no possuem glicerol-cinase.
* Nos adipcitos, o transportador de glicose (GLUT-4) dependente de insulina. Ento, se h pouca
glicose no sangue, logicamente ela no entrar na gliclise, diminuindo a quantidade de DHAP, limitando a
sntese de glicerol-fosfato e de TAG.
- Gliceroneognese no tecido adiposo:
- A gliceroneognese uma verso mais curta da gliconeognese, partindo de piruvato a
DHAP, seguindo-se a converso de DHAP em glicerol-3-fosfato pela enzima citoslica glicerol-3-fosfato-
desidrogenase ligada ao NAD. Depois, o glicerol-3-fosfato utilizado na sntese de TAG.
- No tecido adiposo, a gliceroneognese, acoplada reesterificao dos cidos graxos livres,
controla a velocidade de liberao dos cidos graxos no sangue.
- No tecido adiposo marrom, a mesma via pode controlar a velocidade pela qual os cidos
graxos livres so enviados para a mitocndria para utilizao na termognese.
- Nos seres humanos em jejum, a gliceroneognese no fgado, sozinha, responde pela sntese
de glicerol-3-fosfato suficiente para a reesterificao de at 65% dos cidos graxos em TAG.
- Regulao importante pela PEP-carboxicinase, que limita a velocidade da gliconeognese e
da gliceroneognese.
- Atuando nos receptores de glicocorticoides, hormnios esteroides (como o cortisol e a
dexametasona) aumentam a expresso do gene que codifica a PEP-carboxicinase no fgado, aumentando a
gliconeognese e a gliceroneognese. Ao mesmo tempo, no tecido adiposo, os glicocorticoides suprimem
a expresso do gene que codifica a PEP-carboxicinase, o que resulta em um decrscimo na
gliceroneognese no tecido adiposo.
- Assim, a gliceroneognese regulada reciprocamente no fgado e no tecido adiposo,
afetando o metabolismo dos lipdeos de formas opostas: uma menor velocidade da gliceroneognese no
tecido adiposo leva a uma maior liberao de cidos graxos (e no reciclagem), enquanto uma alta
velocidade no fgado leva a uma maior sntese e exportao dos triacilgliceris.
** As tiazolidinedionas promovem a induo da PEP-carboxicinase no tecido adiposo, levando ao
aumento na sntese dos precursores da gliceroneognese. Portanto, o efeito teraputico das
tiazolidinedionas devido, pelo menos em parte, ao aumento na gliceroneognese, que, por sua vez,
aumenta a sntese de triacilgliceris no tecido adiposo e reduz a liberao de cidos graxos livres do tecido
adiposo para a corrente sangunea.

CONVERSO DO CIDO GRAXO LIVRE NA FORMA ATIVADA


- Os cidos graxos so convertidos na forma ativada (unidos CoA) para participarem da sntese de
TAGs. Essa reao catalisada por uma famlia de acil-CoA-sintetases (tiocinases).

SNTESE DE TAG A PARTIR DE GLICEROL-FOSFATO E ACIL-CoA


- A primeira etapa na biossntese dos TAGs a acilao dos dois grupos hidroxila livres do glicerol-
3-fosfato, por duas molculas de acil-CoA graxo, gerando diacilglicerol-3-fosfato, mais comumente
chamado de cido fosfatdico ou fosfatidato. Esse produto est presente apenas em quantidades muito
pequenas na clula, mas um intermedirio central na biossntese dos lipdeos (pode ser convertido tanto
em um TAG quanto em um glicerofosfolipdeo).
- Na via de sntese de TAGs, o cido fosfatdico hidrolisado pela cido fosfatdico-fosfatase (ou
lipina), formando 1,2-diacilglicerol. Os diacilgliceris so, ento, convertidos em triacilgliceris por
transesterificao com um terceiro acil-CoA graxo.

- Os carboidratos, as gorduras ou as protenas ingeridos em excesso necessidade energtica so


armazenados na forma de triacilgliceris, que podem ser mobilizados para o fornecimento de energia,
capacitando o organismo a suportar perodos de jejum.
- A insulina promove a converso de carboidrato em TAGs. Pessoas com diabetes melito grave,
devido falha na secreo ou na ao da insulina, alm de no serem capazes de utilizar glicose de modo
apropriado, falham tambm em sintetizar cidos graxos a partir de carboidratos ou aminocidos. Se o
diabetes no tratado, essas pessoas apresentam velocidade aumentada na oxidao de gorduras e na
formao de corpos cetnicos.
- O nvel de cidos graxos livres no sangue reflete tanto a velocidade de liberao dos cidos graxos
quanto o balano entre a sntese e a degradao dos triacilgliceris no tecido adiposo e no fgado.
- Quando a mobilizao dos cidos graxos necessria para satisfazer as necessidades energticas,
sua liberao do tecido adiposo estimulada pelos hormnios glucagon e adrenalina que,
simultaneamente, diminuem a velocidade da gliclise e aumentam a velocidade da gliconeognese no
fgado.
- O cido graxo liberado captado por diversos tecidos, incluindo os msculos, onde ele oxidado
para a gerao de energia. A maior parte do cido graxo captado pelo fgado no oxidada, mas reciclada
a TAG e retorna ao tecido adiposo.
DESTINOS DO TAG
- Tecido adiposo branco: armazenado no citosol em gotas lipdicas (depsito de gordura, fonte de
energia).
- Pouco armazenado no fgado. A maioria exportada, empacotada com outros lipdios e
apoprotenas para formar partculas de lipoprotenas (VLDL). A VLDL nascente secretada para o sangue,
onde amadurece e funciona entregando lipdios endgenos para tecidos perifricos.

CATABOLISMO DE TAGs E CIDOS GRAXOS


- Os c. graxos armazenados no tecido adiposo, na forma de TAG neutro, so a principal reserva de
combustvel do organismo. Os TAGs fornecem estoques concentrados de energia metablica, pois so
altamente reduzidos e muito anidros. A oxidao completa dos cidos graxos a CO2 e H2O gera 9 kcal/g de
gordura.

LIBERAO DE CIDOS GRAXOS DOS TAGs

- A superfcie das gotas lipdicas revestida por perilipinas, famlia de protenas que restringem o
acesso s gotculas lipdicas, evitando a mobilizao prematura dos lipdeos.
- Quando os baixos nveis de glicose no sangue ativam a liberao de glucagon, (1) o hormnio se
liga ao seu receptor na membrana do adipcito e assim (2) estimula a adenilato-ciclase, via uma protena
G, a produzir AMPc. Isso ativa a PKA, que fosforila (3) a lipase sensvel a hormnio (HSL) e (4) as molculas
de perilipina na superfcie da gotcula lipdica. A fosforilao da perilipina causa a (5) dissociao da protena
CGI da perilipina (coativador da triacilglicerol lipase do tecido adiposo).
- A CGI ento se associa com a enzima triacilglicerol lipase no adipcito, ativando-a. A triacilglicerol
lipase ativada (6) converte triacilgliceris em diacilgliceris. A perilipina fosforilada se associa com a HSL
fosforilada, permitindo o acesso superfcie da gotcula lipdica, onde (7) ela hidrolisa os diacilgliceris em
monoacilgliceris. Uma terceira lipase, a monoacilglicerol lipase (MGL) (8) hidrolisa os monoacilgliceris.
- (9) Os c. graxos saem do adipcito, se ligam albumina srica no sangue e so transportados no
sangue; eles so liberados da albumina e (10) entram em um micito por meio de um transportador
especfico de c. graxos. (11) No micito, os c. graxos so oxidados a CO2, e a energia da oxidao
conservada em ATP, que abastece a contrao muscular e outros tipos de metabolismo que necessitam de
energia no micito.
- Em suma, quando hormnios sinalizam a necessidade de energia metablica, os triacilgliceris
armazenados no tecido adiposo so mobilizados (retirados do armazenamento) e transportados aos
tecidos (musculatura esqueltica, corao e crtex renal) nos quais os cidos graxos podem ser oxidados
para a produo de energia. A protena-cinase dependente de AMPc (PKA) leva a mudanas que abrem a
gotcula de lipdio para a atividade de trs lipases, que atuam sobre tri-, di- e monoacilgliceris, liberando
cidos graxos e glicerol.
Triacilglicerol triacilglicerol lipase diacilglicerol HSL fosforilada monoacilglicerol
monoacilglicerol lipase c. graxos.
- Os c. graxos liberados passam dos adipcitos para o sangue, onde se ligam albumina srica,
protena solvel que pode se ligar no covalentemente a at 10 c. graxos por monmero de protena.
Ligados a ela, os c. graxos so transportados aos tecidos como o msculo esqueltico, o corao e o crtex
renal, se dissociam da albumina e so levados por transportadores da membrana plasmtica para dentro
das clulas para servir de combustvel.
- O glicerol liberado pela ao da lipase fosforilado e oxidado a di-hidroxiacetona fosfato (DHAP),
que pode entrar nas vias glicoltica ou gliconeognica. Alternativamente, o glicerol fosfato pode ser usado
na sntese de triacilgliceris ou de fosfolipdios.
- Cerca de 95% da energia biologicamente disponvel dos TAGs residem nas suas trs cadeias longas
de c. graxos; apenas 5% so fornecidos pela poro glicerol.
- Os c. graxos livres precisam se tornar ativados (derivados de CoA) para serem oxidados.
- Os c. graxos livres (AGL) no podem ser utilizados como combustvel pelos eritrcitos, pois estes
no possuem mitocndrias. O sistema nervoso central, por sua vez, tambm no pode utilizar cidos graxos
para obter energia.
* Mais de 50% dos c. graxos liberados dos TAGs do tecido adiposo so reesterificados ao glicerol-
3-fosfato.
* A gliceroneognese feita no tecido adiposo branco diminui os cidos graxos livres plasmticos,
que esto associados com a resistncia insulina na diabetes do tipo 2 e na obesidade.

ATIVAO E TRANSPORTE DE C. GRAXOS PARA A MITOCNDRIA


- Visto que a -oxidao ocorre na matriz mitocondrial, o cido graxo precisa ser transportado
atravs da membrana interna da mitocndria, que impermevel CoA. Assim, um transportador
especializado carrega o grupo acila de cadeia longa do citosol para a matriz da mitocndria. Esse processo
de transporte limitante da velocidade chama-se lanadeira (ou ciclo) da carnitina.
** c. graxos com menos de 12 carbonos entram na mitocndria sem transportadores. Uma vez na
matriz, eles so ativados em seus derivados de CoA por enzimas da matriz e so oxidados. Aqueles com 14
ou mais carbonos (que constituem a maioria dos cidos graxos livres obtidos na dieta ou liberados do tecido
adiposo) no passam livremente e usam da carnitina.

CICLO DA CARNITINA
- A primeira reao catalisada por uma famlia de isoenzimas presente na membrana mitocondrial
externa, as acil-CoA-sintetases, que catalisam:
cido graxo + CoA + ATP acil-CoA graxo + AMP + (PPi 2 Pi)
- Assim, as acil-CoA-sintetases catalisam a formao de uma ligao tioster entre o grupo carboxil
do cido graxo e o grupo tiol da coenzima A para produzir um acil-CoA graxo, acoplado clivagem do ATP
em AMP e PPi.
- As acil-CoA graxos (assim como acetil-CoA) so compostos de alta energia, sua hidrlise a c. graxos
livres e CoA bastante exergnica. Assim, sua formao favorecida pela hidrlise imediata do pirofosfato
pela pirofosfatase inorgnica.
- Os acil-CoA graxos formados no lado citoslico da membrana externa da mitocndria podem ser
transportados para dentro da mitocndria e oxidados para produzir ATP, ou podem ser utilizados no citosol
para sintetizar lipdios de membrana.
- A segunda reao a ligao transitria dos c. graxos ao grupo hidroxil da carnitina, formando
acil-graxo-carnitina. Essa transesterificao catalisada na membrana externa pela carnitina
aciltransferase I (CAT-I), tambm chamada carnitina-palmitoil-transferase I (CPT-I).
- A passagem para o espao intermembrana ocorre por meio de grandes poros (formados pela
protena porina) na membrana externa. O acil-graxo-carnitina entra na matriz por difuso facilitada atravs
do transportador acil-carnitina/carnitina da membrana mitocondrial interna.
- Na terceira reao, o grupo acil-graxo transferido da carnitina para a CoA intramitocondrial pela
CAT-II (ou CPT-II). Essa enzima, localizada na face citoslica da membrana mitocondrial interna, regenera a
acil-CoA graxo e a libera, juntamente com a carnitina livre, dentro da matriz. A carnitina retorna ao espao
intermembrana por meio do transportador acil-carnitina/carnitina.
- Esse processo mediado pela carnitina o passo limitante para a oxidao de c. graxos na
mitocndria, sendo um ponto de regulao.
- Inibidor do ciclo da carnitina: A malonil-CoA inibe a CPT-I, impedindo a entrada de grupos acila de
cadeia longa na matriz mitocondrial. Assim, a presena de malonil-CoA no citosol (indicando biossntese)
faz com que o palmitato recm-formado no possa ser transferido para o interior da mitocndria.
- Fontes de carnitina: Encontrada principalmente em carnes. Pode ser sintetizada a partir dos AAs
lisina e metionina por enzimas do fgado e rins, mas no nos msculos esqueltico e cardaco, sendo
totalmente dependentes (msculo esqueltico possui 97% de toda a carnitina do corpo).
- Deficincias da carnitina: Resulta na diminuio da capacidade do tecido de usar c. graxos de
cadeia longa (AGCL) como combustvel metablico. Razes secundrias: doenas hepticas (diminuio da
sntese de carnitina), indivduos subnutridos/estritamente vegetarianos, indivduos com maior necessidade
(gestao, infeces graves, traumas), pacientes em hemodilise (que remove carnitina do sangue).
- Deficincias congnitas num dos componentes do sistema da CPT-I, na reabsoro de
carnitina no tbulo renal ou na captao de carnitina pelas clulas podem causar deficincia primria de
carnitina.
- A deficincia gentica de CPT-1 afeta o fgado, incapacitando-o de utilizar AGCL como
combustvel e prejudicando, assim, a capacidade desse tecido de sintetizar glicose durante o jejum; causa
hipoglicemia grave, coma e morte.
- A deficincia de CPT-II afeta principalmente os msculos cardaco e esqueltico, onde os
sintomas de deficincia de carnitina variam desde cardiomiopatia at fraqueza muscular com
mioglobinemia aps exerccio prolongado. O tratamento inclui evitar jejum prolongado, adotando uma
dieta rica em carboidratos e baixa em AGCL, mas suplementada com cidos graxos de cadeia mdia e
carnitina.
OXIDAO DE CIDOS GRAXOS
- Ocorre em trs etapas. Na primeira etapa -oxidao , os cidos graxos sofrem remoo
oxidativa de sucessivas unidades de dois carbonos na forma de acetil-CoA, comeando pela extremidade
carboxlica da cadeia acil-graxo. O resultado global (no caso do palmitato) a converso da cadeia de 16
carbonos em 8 grupos acetil de dois carbonos (acetil-CoA). A formao de cada acetil-CoA requer a remoo
de 4 tomos de hidrognio (dois pares de eltrons e quatro H+) da poro acil-graxo pelas desidrogenases.
- Na segunda etapa, os grupos acetil da acetil-CoA so oxidados a CO2 no ciclo do cido ctrico, que
tambm ocorre na matriz mitocondrial. A acetil-CoA derivada dos cidos graxos ento entra em uma via de
oxidao final comum com a acetil-CoA derivada da glicose precedente da gliclise. As duas primeiras
etapas da oxidao dos cidos graxos produzem os transportadores de eltrons reduzidos NADH e FADH2,
que na terceira etapa doam eltrons para a cadeia respiratria mitocondrial, com a fosforilao
concomitante de ADP a ATP. A energia liberada pela oxidao dos cidos graxos , portanto, conservada
como ATP.

-OXIDAO
- Consiste em uma sequncia de quatro reaes
envolvendo o carbono (carbono 3), a qual resulta
na diminuio da cadeia do cido graxo em 2
carbonos.
- As etapas incluem uma oxidao que produz
FADH2, uma etapa de hidratao, uma segunda
oxidao que produz NADH e uma clivagem tilica
que libera uma molcula de acetil-CoA.
- As enzimas de cada etapa so especficas em
relao ao comprimento da cadeia.
- N de repeties: (N/2) - 1. Ex.: palmitato (16
carbonos) haver 7 repeties do ciclo.
- Na ltima clivagem: 2 acetil-CoA.
1) A desidrogenao da acil-CoA graxo produz
uma ligao dupla entre os tomos de carbono e
(C-2 e C-3), produzindo uma trans--enoil-CoA. (
= posio da dupla).
- Catalisada por 3 isoenzimas da acil-CoA
desidrogenase, cada uma especfica para uma srie
de comprimentos de cadeia acil-graxo:
- VLCAD (very-long): 12 a 18 carbonos.
- MCAD (medium): 4 a 14 carbonos.
- SCAD (short): 4 a 8 carbonos.
- Cada isoenzima uma flavoprotena com FAD
como grupo prosttico.
- Os eltrons removidos passam para o FAD, e o
FADH2 doa esses eltrons a um transportador de
eltrons da cadeia respiratria (flavoprotena de
transferncia de eltrons, ETF). ** ETF entrega os
eltrons ubiquinona, que transfere ao complexo III, continuando
a cadeia respiratria.
- Essa oxidao anloga desidrogenao do
succinato no ciclo do cido ctrico. Em ambas as reaes, a enzima
est ligada membrana interna, uma ligao dupla introduzida
em um cido carboxlico entre os carbonos e , FAD o aceptor
de eltrons, e os eltrons das reaes por fim entram na cadeia
respiratria e passam para o O2.
2) gua adicionada ligao dupla da trans--enoil-CoA para formar -hidroxiacil-CoA
(3-hidroxiacil-CoA). Essa reao, catalisada pela enoil-CoA hidratase, anloga reao da fumarase no
ciclo do cido ctrico, no qual H2O adicionada a uma ligao dupla -.
3) -hidroxiacil-CoA desidrogenada para formar -cetoacil-CoA, pela ao da -hidroxiacil-CoA
desidrogenase; NAD+ o aceptor de eltrons.
- O NADH formado na reao doa seus eltrons para a NADH-desidrogenase, um transportador de
eltrons da cadeia respiratria, e ATP formado a partir de ADP medida que os eltrons passam para o
O2.
- A reao catalisada pela -hidroxiacil-CoA desidrogenase anloga reao da malato-
desidrogenase do ciclo do cido ctrico.
4) A reao de -cetoacil-CoA com uma molcula de CoA livre separa o fragmento de 2 carbonos da
extremidade carboxlica do cido graxo original como acetil-CoA. catalisada pela
acil-CoA acetiltransferase, mais comumente chamada de tiolase. O outro produto o tioster de CoA do
cido graxo, agora encurtado em 2 tomos de carbono.
- Essa reao chamada de tilise, por analogia ao processo de hidrlise, j que a -cetoacil-CoA
clivada pela reao com o grupo tiol da CoA. A reao da tiolase o reverso da condensao de Claisen.
** A ligao simples entre grupos metileno (-CH2-) nos cidos graxos relativamente estvel. A
sequncia da -oxidao um mecanismo eficiente para desestabilizar e quebrar essas ligaes.
** As trs primeiras reaes da -oxidao criam uma ligao C-C muito menos estvel, na qual o
carbono (C-2) est ligado a dois carbonos carbonlicos (o intermedirio -cetoacil-CoA). A funo cetona
do carbono (C-3) faz dele um bom alvo para ataque nucleoflico pelo -SH da CoA, catalisado pela tiolase.
** A acidez do hidrognio e a estabilizao por ressonncia do carbnion gerado pela sada desse
hidrognio tornam o -CH2-CO-S-CoA terminal um bom grupo de sada, facilitando a quebra da ligao
-.
- Em uma passagem pela sequncia da -oxidao, 1 molcula de acetil-CoA, 2 pares de eltrons e
4 prtons (H+) so removidos da acil-CoA graxo de cadeia longa, encurtando-a em dois tomos de carbono.
Palmitoil-CoA + CoA + FAD + NAD+ + H2O miristoil-CoA + acetil-CoA + FADH2 + NADH + H+
- Ao todo, sete passagens pela sequncia da -oxidao so necessrias para oxidar uma molcula
de palmitoil-CoA em 8 molculas de acetil-CoA.
Palmitoil-CoA + 7CoA + 7FAD + 7NAD+ + 7H2O 8Acetil-CoA + 7FADH2 + 7NADH + 7H+
- A transferncia de eltrons do NADH ou FADH2 para o O2 produz uma H2O por par de eltrons.
Mas cada ciclo consome 1 molcula de H2O. Assim, 14 7 = 7 molculas de H2O.
- 7 x 2,5 (NADH) + 7 x 1,5 (FADH2) = 28 ATP. Logo, a oxidao do palmitoil-CoA:
Palmitoil-CoA + 7CoA + 7O2 + 28 Pi + 28 ADP 8Acetil-CoA + 28 ATP + 7 H2O
- Cada molcula de acetil-CoA que entra no ciclo de Krebs produz 3 NADH, 1 FADH2 e 1 ATP, dando
um saldo de 10 ATPs por acetil-CoA. Logo:
8Acetil-CoA + 16O2 + 80 Pi + 80 ADP 8CoA + 80 ATP + 16H2O + 16 CO2
- Juntando as duas ltimas reaes:
Palmitoil-CoA + 23O2 + 108 Pi + 108 ADP CoA + 108 ATP + 23 H2O + 16 CO2
- Como a ativao do palmitato a palmitoil-CoA quebra 2 ligaes fosfoanidro do ATP, o custo
energtico equivalente a 2 ATP, dando um saldo lquido de 106 ATP por palmitato oxidado.
- Deficincia de MCAD: Doena autossmica recessiva; um dos erros inatos mais comuns do
metabolismo e o erro inato mais comum na oxidao de cidos graxos (1:14000 nascimentos no mundo,
com incidncia maior em norte-europeus). Causa hipoglicemia grave. O tratamento inclui evitar jejum.

OXIDAO DE GRAXOS DE NMERO MPAR


- So oxidados pela mesma via que os graxos de nmero
par. Entretanto, o substrato da ltima passagem pela -
oxidao um acil-CoA graxo de 5 carbonos. Sua clivagem
libera acetil-CoA (que entra no ciclo de Krebs) e propionil-
CoA (que entra em uma via extra de 3 reaes).
1) A propionil-CoA primeiro carboxilada para formar o
estereoismero D da metilmalonil-CoA pela propionil-
CoA-carboxilase, que contm biotina como cofator.
- Como na reao da piruvato-carboxilase: 1) o CO2 (ou
seu on hidratado, HCO3-) ativado pela ligao biotina
antes de sua transferncia para o substrato, a poro
propionato e 2) A formao do intermedirio
carboxibiotina requer energia, que fornecida pelo ATP.
2) A D-metilmalonil-CoA assim formada epimerizada
ao seu estereoismero L pela metilmalonil-CoA-
epimerase.
3) A L-metilmalonil-CoA ento sofre um rearranjo
intramolecular para formar succinil-CoA (que pode entrar
no ciclo de Krebs). Esse rearranjo catalisado pela
metilmalonil-CoA-mutase, que requer a coenzima B12.
* Essa reao uma das duas nicas do organismo que
requer vit. B12 (a outra a remetilao da homocistena
em metionina). Em deficincia de B12, o propionato e o
metilmalonato so excretados na urina. Ausncia ou
deficincia da mutase (ou menor afinidade pela
coenzima) ou incapacidade de converter B12 na coenzima
tambm resultam em acidose metablica.
OXIDAO DE CIDOS GRAXOS INSATURADOS

- Ligaes na configurao cis no podem sofrer a ao da enoil-CoA hidratase, a enzima que catalisa
a adio de H2O s ligaes duplas trans da -enoil-CoA gerada durante a -oxidao. Assim, duas reaes
adicionais so necessrias, feitas por uma isomerase e uma redutase.
- Monoinsaturado: O oleato (18 carbonos), atravs de oleil-CoA, passa 3 vezes pelo ciclo de
oxidao, produzindo 3 acetil-CoA e um ster de CoA de 12 carbonos (cis--dodecenoil-CoA). Como esse
produto no serve de substrato para a enoil-CoA hidratase, a enzima enoil-CoA-isomerase o converte a seu
ismero trans, intermedirio normal na -oxidao. No total, 9 acetil-CoA so produzidas a partir de uma
molcula de oleato de 18 carbonos.
- Poli-insaturado: Ex.: linoleato (18 carbonos), com cis em C-9 e C-12. A linoleoil-CoA sofre trs
passagens pela sequncia de -oxidao para produzir 3 acetil-CoA e o ster acil CoA de 12 carbonos, com
cis em C-3 e C-6. As ligaes duplas esto na posio errada e configurao errada (cis). Logo, a ao
combinada da enoil-CoA-isomerase (que converte uma ligao cis a trans) e da 2,4-dienoil-CoA-redutase
(que oxida a ligao cis) permite a reentrada na -oxidao. Resultado: linoleato 9 acetil-CoA.
-OXIDAO EM PEROXISSOMOS
- Oxidao preliminar de cidos graxos de cadeia
muito longa nos peroxissomos. 22 carbonos
- Nos peroxissomos, a acil-CoA oxidase ligada ao
FAD, que introduz a dupla ligao, passa os eltrons
diretamente ao O2, produzindo H2O2, que clivado a
H2O e O2 pela catalase.
- Aqui, a energia liberada na primeira etapa
oxidativa da degradao dos cidos graxos no
conservada como ATP, mas sim dissipada como calor.
- Sndrome de Zellweger (resultando em
incapacidade de formar peroxissomos) ou
adrenoleucodistrofia ligada ao X (perda de um
transportador funcional para esses cidos graxos na
membrana peroxissomal) levam ao acmulo no
sangue de cidos graxos de cadeia muito longa,
especialmente 26:0.

***REGULAO

- Duas enzimas so essenciais na coordenao: a acetil-CoA-carboxilase (ACC), primeira enzima na


sntese dos cidos graxos, e a carnitina-aciltransferase I (CAT-I), que limita o transporte de cidos graxos
para dentro da matriz mitocondrial para a -oxidao.
- Ingesto rica em carboidratos insulina fosfatase dependente de insulina desfosforila a ACC,
ativando-a ACC catalisa a formao de malonil-CoA (1 intermedirio da sntese de c. graxos)
malonil-CoA inibe a CAT-I no entra c. graxos na matriz mitocondrial.
- Baixa glicose no sangue glucagon ativao da protena-cinase pelo AMPc fosforila a ACC,
inativando-a [malonil-CoA] c. graxos entram na matriz e se tornam o principal combustvel.
- Regulao por metablitos que sinalizam suficincia de energia: [NADH/NAD+] inibio da
-hidroxiacil-CoA-desidrogenase // [acetil-CoA] inibio da acil-CoA acetiltransferase (tiolase).
-OXIDAO

- A presena de um grupamento metil no


carbono de um cido graxo torna a -oxidao
impossvel, e esses cidos graxos ramificados so
catabolizados nos peroxissomos de clulas animais por
-oxidao.
- Na oxidao do cido fitnico (20 carbonos),
por exemplo, o fitanoil-CoA hidroxilado em seu
carbono pela fitanoil-CoA--hidroxilase, em uma
reao que envolve oxignio molecular; descarboxilado
para formar um aldedo mais curto em um carbono; e
ento oxidado ao cido carboxlico correspondente, que
agora no tem nenhum substituinte no carbono e
pode ser oxidado por -oxidao.
PROBLEMA 3.2
COLESTEROL
- Componente estrutural de todas as membranas celulares, modulando sua fluidez.
Em tecidos especficos, o precursor dos cidos biliares, dos hormnios esteroides e da vitamina D.
- No necessrio que esteja presente na dieta de mamferos todas as clulas so capazes de
sintetiz-lo a partir de precursores simples.
- O fgado tem papel central na regulao da homeostasia do colesterol (sistemas de transporte,
biossntese e mecanismos de regulao.
- O colesterol heptico formado a partir de vrias fontes, que incluem a dieta, a sntese de novo pelos
tecidos extra-hepticos e a sntese local.
- eliminado do fgado pela bile no lmen intestinal sem sofrer modificaes ou convertido em sais
biliares.
- enviado para os tecidos perifricos como componente das lipoprotenas plasmticas.
- O desequilbrio entre o influxo (que chega ao fgado) e efluxo (que sai do fgado) pode resultar em
deposio nos tecidos, especialmente no endotlio vascular. Deposio formao de placas
estreitamento dos vasos (aterosclerose) incidncia de doenas vasculares (crebro, corao, perifricas).

ESTRUTURA
- Consiste em 4 anis hidrocarbonados fundidos (A, B, C, D;
ncleo esteroide). Ligado a C-17 do anel D, h uma cadeia ramificada
de 8 carbonos. H ainda uma hidroxila em C-3 (A) e uma dupla ligao
entre C-5 e C-6 (B).
- Esteris: esteroides com 8 a 10 tomos de carbono na cadeia
lateral ligada a C-17 e uma hidroxila em C-3. O colesterol o principal
esterol dos tecidos animais. ** Como o colesterol pode ser transportado
junto com os esteris vegetais (pouco absorvidos em humanos), dietas
com fito esteroides podem ser usadas para tratar hipercolesterolemia,
reduzindo a absoro de colesterol presente na dieta.
- steres de colesterol: Maior parte do colesterol plasmtico est
nessa forma. ainda mais hidrofbico que o colesterol livre, existindo
em pequenas quantidades na maioria das clulas. Devido
hidrofobicidade, tanto ele quanto o livre devem ser transportados
associados a protenas (componentes de lipoprotenas) ou solubilizados
por fosfolipdios e sais biliares na bile.

SNTESE DE COLESTEROL
- Sintetizado por quase todos os tecidos (maior parte: fgado, intestino, crtex adrenal e tecidos
reprodutivos).
- Todos os tomos de carbono so derivados do acetato e o NADPH o doador dos equivalentes
redutores.
- A rota endergnica; a energia garantida pela hidrlise das ligaes tio ster de alta energia da
acetil-CoA e dos fosfatos terminais do ATP.
- Requer enzimas encontradas no citosol e nas membranas do retculo endoplasmtico liso (REL).
- Basicamente em 4 estgios: 1) condensao de 3 unidades de acetato, formando um intermedirio
de 6 carbonos, o mevalonato; 2) converso do mevalonato em unidades de isopreno ativadas; 3)
polimerizao das 6 unidades de isopreno com 5 carbonos, formando o esqualeno linear, com 30 carbonos; e
4) ciclizao do esqualeno para formar os quatro anis do ncleo esteroide, com uma srie de mudanas
adicionais (oxidaes, remoo ou migrao de grupos metil) para produzir o colesterol.
SNTESE DO MEVALONATO
- As duas primeiras reaes so similares s que produzem
corpos cetnicos. 2 molculas de acetil-CoA condensam-se
para formar acetoacetil-CoA, que se condensa com uma 3
molcula de acetil-CoA, gerando o composto de 6 carbonos
-hidroxi--metilglutaril-CoA (HMG-CoA). Essas reaes so
catalisadas, respectivamente, por acetil-CoA-acetil
transferase e HMG-CoA-sintase.
- HMG-CoA-sintase possui duas isoformas: citoslica
(sntese de colesterol) e mitocondrial (HMG-CoA para sntese
de corpos cetnicos).
- A reduo do HMG-CoA formando mevalonato,
catalisada pela enzima HMG-CoA-redutase, a etapa limitante
da velocidade e o passo regulador da sntese de colesterol.
Essa enzima uma protena intrnseca da membrana do REL,
com o domnio cataltico voltado para o citosol.
- Ocorre no citosol, usando 2 molculas de NADPH (4
eltrons) como agente redutor, liberando CoA (reao
irreversvel).

CONVERSO MEVALONATO ISOPRENOS

- 2 grupos fosfato so transferidos de 2 ATP para o


mevalonato. A seguir, pela enzima pirofosfo-
mevalonato-descarboxilase, ocorre adio de fosfato
em C-3 que, pela mesma enzima, sair na prxima
etapa, juntamente com a carboxila vizinha, produzindo
uma ligao dupla no produto de 5 carbonos
isopentenil-pirofosfato (IPP, 1 isopreno ativado).
- A isomerizao do isopentenil-pirofosfato gera o 2
isopreno ativado, o dimetilalil-pirofosfato (DPP).
CONDENSAO DOS ISOPRENOS

- O isopentenil-pirofosfato e o dimetilalil-pirofosfato sofrem, agora, uma condensao cabea com


cauda, em que um grupo pirofosfato deslocado, sendo formada uma cadeia de 10 carbonos, o geranil-
pirofosfato (GPP). (Obs.: cabea = extremidade onde o pirofosfato est ligado).
- O geranil-pirofosfato sofre outra condensao do tipo cabea com cauda com o isopentenil-
pirofosfato, gerando um intermedirio de 15 carbonos, o farnesil-pirofosfato (FPP). (Obs.: prenilao = ligao
covalente de uma molcula de farnesil a protenas, sendo um mecanismo de ancoragem delas membrana
plasmtica).
- Duas molculas de farnesil-pirofosfato condensam-se e sofrem reduo, com liberao de ambos os
grupos pirofosfato, formando o composto de 30 carbonos esqualeno.
** 3 molculas de ATP so utilizadas para criar cada um dos 6 isoprenos ativados necessrios para a
construo do esqualeno, em um custo total de 18 molculas de ATP.
** A liberao de pirofosfato em cada uma das reaes de condensao torna o processo irreversvel.
CONVERSO DO ESCALENO NO NCLEO ESTEROIDE DE 4 ANEIS

- A atividade da esqualeno-monoxigenase adiciona um tomo de oxignio do O2 extremidade da


cadeia do esqualeno, formando um epxido. NADPH reduz o outro tomo de oxignio do O2 a H2O.
- As ligaes duplas do produto, o esqualeno-2,3-epxido, e sua posterior hidroxilao so
responsveis por converter o esqualeno epxido linear em uma estrutura cclica, o lanosterol, que contm os
4 anis caractersticos do ncleo esteroide.
- O lanosterol finalmente convertido em colesterol em uma srie de aproximadamente 20 reaes,
que resultam na diminuio da cadeia carbonada de 30 para 27 tomos, na remoo de duas metilas de C-4,
na migrao da ligao dupla de C-8 para C-5 e na reduo da ligao dupla entre C-24 e C-25.

REGULAO
- Nos mamferos, a produo do colesterol regulada pela concentrao intracelular de colesterol e
pelos hormnios glucagon e insulina. A etapa comprometida na via de sntese do colesterol (e o principal local
de regulao) a converso de HMG-CoA em mevalonato pela HMG-CoA redutase.
- Fosforilao/desfosforilao independente de esteris: regulao a curto prazo da HMG-CoA
redutase por alterao covalente reversvel fosforilao pela protena-cinase dependente de AMP (AMPK),
sensvel alta concentrao de AMP (indicando baixa concentrao de ATP).
- Assim, quando os nveis de ATP declinam, a sntese de colesterol desacelera, e as vias
catablicas para a gerao de ATP so estimuladas.
- O glucagon estimula a sua fosforilao (inativao), e a insulina promove a desfosforilao,
ativando a enzima e favorecendo a sntese de colesterol.
- Regulao hormonal: A quantidade/atividade da HMG-CoA-redutase controlada por hormnios.
- O aumento da concentrao de insulina e tiroxina (T4) favorece o aumento da expresso do
gene da HMG-CoA-redutase. O glucagon e os glicocorticoides exercem efeito oposto.
- Inibio por frmacos: As estatinas (atorvastatina, fluvastatina, lovastatina, pravastatina,
rosuvastatina e sinvastatina) so anlogos estruturais do HMG-CoA e so (ou so metabolizados a) inibidores
competitivos e reversveis da HMG-CoA-redutase. So usados para diminuir os nveis plasmticos de colesterol
em pacientes com hipercolesterolemia.
- Regulao da expresso gnica por esteris: A regulao da sntese de HMG-CoA-redutase mediada
por um sistema de regulao da transcrio do gene da HMG-CoA, controlado por uma famlia de protenas
chamadas protenas de ligao aos elementos reguladores de esterol (SREBP, PLERE).
- Quando recm-sintetizadas, essas protenas esto inseridas no RE. Apenas o fragmento solvel
do domnio regulatrio de uma SREBP atua como ativador da transcrio gnica.
- Quando os nveis de colesterol e oxiesterol esto altos, as SREBP so mantidas no RE e
complexadas a outra protena (protena ativadora da clivagem da SREBP, ou SCAP), que, por sua vez, est
ancorada membrana do RE por sua interao com uma 3 protena de membrana, a Insig (protenas cujos
genes so induzidos pela insulina). SCAP e Insig atuam como sensores de esterol. Assim, nveis de colesterol
altos fazem com que o complexo Insig-SCAP-SREBP permanea retido no RE, pela prpria Insig.
- Quando os nveis de colesterol esto baixos, o complexo SCAP-SREBP escoltado por
protenas secretrias para o aparelho de Golgi, onde duas clivagens proteolticas de SREBP liberam um
fragmento regulatrio, que entra no ncleo e ativa a transcrio dos seus genes-alvo, incluindo a HMG-CoA
redutase.
** Quando os nveis de esteris aumentam suficientemente, a liberao proteoltica do
fragmento regulatrio da SREBP novamente bloqueada, e a degradao proteossmica do domnio ativo
existente resulta em rpido desligamento dos genes-alvos.

- Acelerao da degradao enzimtica dependente de esterol: A prpria HMG-CoA-redutase uma


protena integral da membrana do RE sensvel a esteris.
- Altos nveis de colesterol celular so detectados pela Insig, que dispara o acoplamento de
molculas de ubiquitina HMG-CoA-redutase, levando sua degradao pelo proteossomo.
** Protenas que devem ser degradadas so marcadas pela ubiquitina essa marcao
reconhecida pelo proteossomo citoslico desdobramento da protena, remoo da ubiquitina e transporte
para o respectivo centro proteoltico degradao dos fragmentos por proteases, gerando aminocidos.
DEGRADAO DO COLESTEROL
- Sua estrutura cclica no pode ser degradada at CO2 e H2O. O ncleo esteroide eliminado intacto
pela converso em cidos e sais biliares, que so excretados nas fezes, e pela secreo de colesterol na bile,
que o transporta at o intestino para eliminao. No intestino, parte do colesterol modificada por bactrias
antes da excreo.
- Os principais compostos formados so os ismeros coprostanol e colestanol, derivados reduzidos do
colesterol. Junto com o colesterol, esses compostos representam a maior parte dos esteris fecais neutros.

LIPOPROTENAS PLASMTICAS
- Complexos macromoleculares esfricos de lipdios
e protenas especficas (apolipoprotenas ou
apoprotenas).
- Incluem os quilomicra (Q), as lipoprotenas de
densidade muito baixa (VLDL), baixa (LDL) e alta
(HDL).
- Diferem na composio lipdica e proteica, no
tamanho, na densidade e no local de origem.
- Sua funo manter solveis seus componentes
lipdicos no plasma, alm de promover um
mecanismo eficiente de transporte de lipdios entre
os tecidos.
- Caso o sistema de transporte no seja eficiente,
ocorre deposio de lipdios (principalmente
colesterol) nos tecidos formao de placas no endotlio vascular.
COMPOSIO
- Constitudas por um ncleo de lipdios neutros (triacilgliceris, steres de colesterila, obtidos da dieta
ou da sntese) circundado por uma camada anfiptica de apolipoprotenas, fosfolipdios e colesterol livre (no
esterificado). Essa parte anfiptica orientada de forma a expor sua poro polar na superfcie da lipoprotena,
tornando a partcula solvel em meio aquoso.
* Os steres de colesterila so formados no fgado pela ao da acil-CoA-colesterol aciltransferase
(ACAT). Essa enzima catalisa a transferncia de um cido graxo da CoA para o grupo hidroxil do colesterol,
convertendo o colesterol em uma forma mais hidrofbica e prevenindo que eles entrem nas membranas.
- Tamanho e densidade: Quilomicra: menor densidade e maior tamanho; a maior % de lipdios e a
menor % de protenas. - Ordem crescente de densidade: VLDL < LDL < HDL.
- Podem ser separadas por sua mobilidade eletrofortica ou por ultracentrifugao (densidade).
- Apolipoprotenas: fornecem stios de reconhecimento para receptores nas superfcies celulares e
servem como ativadoras ou coenzimas para enzimas envolvidas no metabolismo das lipoprotenas.
- Algumas so componentes estruturais essenciais das lipoprotenas e no podem ser
removidas (no podem ser produzidas sem elas), enquanto outras so transferidas livremente entre as
lipoprotenas.
FFA: cidos graxos livres
TAG: triacilglicerol
C: colesterol
CE: ster de colesterila

METABOLISMO DOS QUILOMICRA


- Os quilomcrons (Triacilglicerol > steres de colesterila, colesterol) so sintetizados a partir de
gorduras da dieta no RE dos entercitos. Ento, movem-se pelo sistema linftico e entram na corrente
sangunea pela veia subclvia esquerda.
- As apolipoprotenas dos quilomcrons incluem a apoB-48 (exclusiva dessa classe de lipoprotenas), a
apoE e a apoC-II. A fonte dessas apolipoprotenas a HDL circulante.
- A lipase lipoproteica nos capilares do tecido adiposo, do corao, do msculo esqueltico e da
glndula mamria em lactao, ativada pela apoC-II das lipoprotenas circulantes, hidrolisa os triacilgliceris
carregados por essas partculas, formando cidos graxos livres e glicerol para esses tecidos. Os quilomcrons,
portanto, transportam cidos graxos da dieta para os tecidos onde eles sero consumidos ou armazenados
como combustvel.
- Se no forem imediatamente captados pelas clulas, os cidos graxos de cadeia longa so
transportados pela albumina srica at que sua captao ocorra.
- O glicerol usado pelo fgado, por exemplo, na sntese de lipdeos, na gliclise ou na gliconeognese.
** Pacientes com deficincia de lipase proteica ou de apoC-II (hiperlipoproteinemia do tipo 1 ou
deficincia familiar de lipase lipoproteica) apresentam um grande acmulo (1.000 mg/dL ou mais) de TAG-
quilomicra no plasma (hipertriacilglicerolemia), mesmo no jejum.
** Insulina estimula a sntese/transferncia da lipase lipoproteica para o lmen (superf.) dos capilares.
** Ismeros da lipase proteica possuem diferentes Km para TAGs: a enzima dos adipcitos tem Km
maior (s remove os c. graxos das lipoprotenas circulantes e os armazena como TAG quanto a concentrao
plasmtica de lipoprotenas estiver elevada). A enzima do msculo cardaco tem Km menor (ainda que a
concentrao de lipoprotenas seja baixa, o corao tem acesso contnuo aos combustveis circulantes,
mostrando a importncia dos c. graxos como fonte de energia para esse tecido).
- Aps a degradao de mais de 90% dos TAGs, a partcula diminui de tamanho/aumenta densidade.
Alm disso, as apoC retornam para as HDL.
- O remanescente se move pela corrente sangunea para o fgado, que contm receptores de
lipoprotenas que reconhecem a apoE. Liga-se nesses receptores e captado pelo hepatcito por endocitose.
- Ocorre fuso das vesculas endocticas com lisossomos. As apolipoprotenas, os steres de colesterila
e os outros componentes endocitados so degradados por hidrlise, liberando AAs, colesterol livre e cidos
graxos. O receptor reciclado.

METABOLISMO DAS VLDL


- Quando a dieta contm mais cidos graxos e colesterol do que a quantidade necessria para uso
imediato como combustvel ou como precursores de outras molculas, eles so convertidos em TAGs ou
steres de colesterila no fgado e empacotados com apolipoprotenas especficas, formando as lipoprotenas
de densidade muito baixa (VLDL). O excesso de carboidratos na dieta tambm pode ser convertido em
triacilgliceris no fgado e exportado como VLDL.
- Compostas predominantemente por TAGs (60%). Alm disso, possuem steres de colesterila; apoB-
100 (do fgado); apoC-I, apoC-II, apoC-III e apoE (provenientes da HDL circulante)
- Sua funo carregar esse lipdio do fgado (local de sntese) para os tecidos perifricos, onde os
triacilgliceris so degradados pela lipase lipoproteica.
** Abetalipoproteinemia: tipo raro de hipolipoproteinemia causada por um defeito na protena
microssomal transferidora de triacilgliceris (PTM), que impede o carregamento da apoB com lipdios. Como
consequncia, no existe formao dos quilomicra e das VLDL, o que causa acmulo de triacilgliceris no
fgado e no intestino.
- As VLDL so transportadas pelo sangue do fgado para o msculo e o tecido adiposo. Nos capilares
desses tecidos, apoC-II ativa a lipase lipoproteica, que catalisa a liberao dos cidos graxos a partir dos
triacilgliceris das VLDL. Adipcitos captam os cidos graxos para formar TAGs e gotculas de gordura; os
micitos, para oxidao.
** insulina VLDL lipdio da dieta para tecido adiposo (armazenamento).
** Jejum VLDL formados usam cidos graxos do tecido adiposo, principalmente msculos.
- A perda de TAGs gera modificaes na partcula, ficando menores e mais densas. Componentes da
superfcie (apoC, apoE) retornam para as HDL, mas as partculas retm a apoB-100. Assim, esse remanescente
de VLDL chamado IDL (lipoprotenas de intensidade intermediria).
- As IDL tambm podem ser captadas pelas clulas, por endocitose mediada por receptor que usa apoE
como ligante.
* apoE: possui trs alelos: apoE-2 (7%), apoE-3 (78%) e apoE-4 (15%)
** apoE-2 tem pouca afinidade pelos receptores, e pacientes homozigotos para apoE-2
apresentam deficincia na depurao dos remanescentes de quilomicra e IDL hipolipoprotenemia familiar
tipo III (disbetaliprotenemia familiar ou doena beta larga). Causa hipercolesterolemia e aterosclerose
aumentada.
** Homozigotas para apoE-4 tm o risco de desenvolver Alzheimer aumentado em 16 vezes;
para aquelas que a desenvolvem, a idade mdia do incio da doena pouco abaixo dos 70 anos. Hiptese:
estabilizao da estrutura do citoesqueleto dos neurnios As apoE-2 e apoE-3 ligam-se a diversas protenas
associadas com os microtbulos neuronais, enquanto apoE-4 no o faz. Isso pode acelerar a morte dos
neurnios.
- Depois, alguns TAGs so transferidos das VLDL para as HDL em uma reao de troca que,
simultaneamente, transfere steres de colesterila das HDL para as VLDL. Essa troca mediada pela protena
transferidora de steres de colesterol (PTEC). Essa reao final converte as IDL em LDL (lipoprotena de baixa
densidade).

METABOLISMO DAS LDL


- Contm concentrao de TAGs bem menor em relao s VLDL; so ricas em colesterol e steres de
colesterila. A apoB-100 a sua principal apolipoprotena.
- LDL transporta colesterol para os tecidos extra-hepticos, como msculo, glndulas suprarrenais e
tecido adiposo. Esses tecidos tm receptores na membrana plasmtica que reconhecem a apoB-100 e
controlam a captao de colesterol e steres de colesterila. A LDL tambm entrega colesterol para os
macrfagos, algumas vezes os convertendo em clulas espumosas.
- A LDL no captada pelos tecidos perifricos retornam ao fgado onde so captados via receptores de
LDL na membrana plasmtica dos hepatcitos. O colesterol que entra no hepatcito por essa via pode ser
incorporado nas membranas, convertido em cidos biliares ou reesterificados pela ACAT (formando steres
de colesterila) para armazenamento nas gotculas lipdicas citoslicas.
- Essa via, da formao de VLDL no fgado ao retorno de LDL para o fgado a via endgena do
metabolismo e transporte do colesterol.
- O acmulo do excesso de colesterol intracelular prevenido pela diminuio da velocidade de sntese
quando colesterol suficiente est disponvel a partir de LDL no sangue.
- Endocitose mediada por receptores: Cada partcula de LDL na corrente sangunea contm apoB-100,
a qual reconhecida por receptores de LDL presentes na membrana plasmtica de clulas que precisam captar
colesterol.
- Receptores de LDL so sintetizados no aparelho de Golgi e so transportados para a membrana
plasmtica, onde ficam disponveis para ligar apoB-100.
- A ligao da LDL ao receptor de LDL inicia a endocitose, que transfere a LDL e o seu receptor
para o interior da clula dentro de um endossomo. As pores da membrana do endossomo que contm o
receptor brotam na membrana plasmtica e os receptores retornam superfcie celular, para funcionar de
novo na captao de LDL.
- O endossomo funde-se com um lisossomo, o qual contm enzimas que hidrolisam os steres
de colesterila, liberando colesterol e cidos graxos no citosol. A protena apoB-100 tambm degradada em
aminocidos, liberados para o citosol.
** A apoB-100 tambm est presente na VLDL, mas o seu domnio de ligao ao receptor no
est disponvel para a interao com o receptor de LDL; a converso de VLDL em LDL expe o domnio de
ligao ao receptor da apoB-100.
- Indivduos com a doena gentica hipercolesterolemia familiar (HF) tm mutaes no receptor de
LDL, que previne a captao normal de LDL pelo fgado e pelos tecidos perifricos. O resultado da captao
defeituosa de LDL so nveis muito altos de LDL no sangue (e de colesterol que ela carrega), aumentando (e
muito) a probabilidade de desenvolver aterosclerose. OBS.: Tambm pode ser causada pelo aumento da
atividade de uma protease que degrada o receptor e por defeitos na apoB-100, os quais reduzem a ligao da
LDL ao receptor.
- Doena de Niemann-Pick tipo C (NPC): defeito hereditrio no armazenamento de lipdeos, em que o
colesterol no transportado para fora dos lisossomos e, ao contrrio, acumula nos lisossomos do fgado,
crebro e pulmes, levando a morte prematura.
- Doena de Wolman: defeito na capacidade de hidrlise lisossomal dos steres de colesterila.
- Efeitos do colesterol endocitado: O colesterol originrio dos remanescentes de quilomicra e das IDL
e LDL afeta o contedo celular de colesterol.
- A HMG-CoA-redutase inibida por nveis de colesterol e, como resultado, a sntese de
colesterol diminui.
- A sntese de novos receptores para LDL reduzida, devido menor expresso do gene do
receptor de LDL, limitando assim a entrada de colesterol-LDL nas clulas (semelhante ao mecanismo de
regulao gnica da HMG-CoA-redutase).
- Se no existe necessidade imediata de colesterol para funes estruturais ou para sntese de
substncias derivadas de colesterol, ele esterificado pela ACAT (lembrando: ACAT transfere um cido graxo
de uma acil-CoA para o colesterol, produzindo um ster de colesterila que pode ser armazenado na clula). A
atividade da ACAT estimulada pelo aumento do colesterol intracelular.
- A questo do macrfago: macrfagos possuem altos nveis de um receptor "removedor" (classe A,
RR-A), podem reconhecer uma grande variedade de substncias e intermediar a endocitose das LDL
quimicamente modificadas, nas quais os componentes lipdicos ou a apoB esto oxidados. Assim, steres de
colesterila se acumulam nos macrfagos, causando sua transformao em "clulas espumosas", as quais
participam da formao da placa aterosclertica. OBS.: a expresso do receptor "removedor'' no regulada
pelo aumento dos nveis intracelulares de colesterol.

METABOLISMO DAS HDL


- Origina-se no fgado e no intestino delgado como pequenas partculas ricas em protena que contm
relativamente pouco colesterol e no contm steres de colesterila.
- Contm principalmente apoA-I (70%). Alm disso, servem de reservatrio circulante de apoC-II
(transferida para as VLDL e os quilomicra, para atuar como ativadora da lipase lipoproteica) e de apoE
(necessria para a endocitose mediada por receptor das LDL e dos remanescentes de quilomicra).
- Contm tambm a enzima lecitina-colesterol-aciltransferase (LCAT), que catalisa a formao de
steres de colesterila a partir de lecitina (fosfatidilcolina) e de colesterol.
- A LCAT na superfcie das partculas de HDL nascentes converte o colesterol e a fosfatidilcolina dos
remanescentes do quilomcron e da VLDL encontradas na corrente sangunea em steres de colesterila, dando
incio formao do ncleo da HDL, transformando a HDL nascente em forma de disco em uma partcula de
HDL madura de forma esfrica.
- A HDL nascente tambm pode captar colesterol de clulas extra-hepticas ricas em colesterol,
inclusive de macrfagos e de clulas espumosas formadas a partir dele.
** As HDL so excelentes aceptores de colesterol no esterificado por possurem alta concentrao de
fosfolipdios, que agem solubilizando o colesterol.
- Quando o colesterol captado pelas HDL, ele
imediatamente esterificado pela LCAT no plasma,
sintetizada no fgado e ativada por ApoA-I.
- A LCAT transfere o cido graxo de C-2 da
fosfatidilcolina para o colesterol. Os produtos
resultantes so um ster de colesterila hidrofbico, que
sequestrado no ncleo da HDL, e lisofosfatidilcolina
(lisolecitina), que se liga albumina.
** A esterificao mantm um gradiente de
concentrao de colesterol, permitindo o fluxo contnuo
de colesterol para a HDL.
- Com essa reao, o HDL se torna uma partcula
rica em ster de colesterila, transportando-o para o
fgado.
- Parte dos steres de colesterila no HDL tambm
pode ser transferida ao LDL pela protena transportadora
de ster de colesterila (PTEC), em uma permuta por
TAGs.
- A maior parte desse colesterol convertido em
sais biliares no fgado e armazenado na vescula biliar.
- A transferncia seletiva do colesterol dos tecidos
perifricos para as HDL e das HDL para o fgado (para a
sntese de cidos biliares ou para descarte via bile) e/ou
para os tecidos esteroidognicos (para sntese de
hormnios) um processo crucial na homeostasia do
colesterol.

- O mecanismo pelo qual o colesterol descarregado no fgado e em outros tecidos via receptor SR-BI
no envolve endocitose; esses receptores controlam a transferncia parcial e seletiva do colesterol e de outros
lipdios do HDL para a clula.
- A HDL descarregada ento se dissocia e recircula na corrente sangunea para extrair mais lipdios dos
remanescentes de quilomcrons e VLDL e de clulas sobrecarregadas com colesterol.

OBESIDADE
- Distrbio dos sistemas reguladores do peso corporal e caracterizado por armazenamento de
excesso de gordura corporal.
- Fatores: estilo de vida sedentrio e a abundante e ampla variedade de alimento palatvel a custos
baixos nas sociedades industrializadas.
- medida que a obesidade aumenta, tambm aumenta o risco de desenvolvimento de doenas
associadas, como a artrite, o diabetes, a hipertenso, a doena cardiovascular e o cncer.
- IMC: fornece uma medida do peso relativo, ajustado altura. Permite comparaes tanto dentro de
populaes quanto entre populaes.
- Saudvel: 19,5 24,9 / Sobrepeso: 25 29,9 / Obeso: igual ou superior a 30 / Acima de 40:
extremamente obeso.
- Diferenas anatmicas na deposio da gordura:
- Uma razo cintura/quadril de mais de 0,8 para mulheres e mais de 1,0 para homens
denominada obesidade androide, em "forma de ma" ou na regio da cintura e est associada a uma maior
deposio de gordura no tronco.
- Associado a maior risco de hipertenso, diabetes, dislipidemia e doena cardaca coronariana.
- Uma razo cintura/quadril menor que 0,8 para mulheres e menor que 1,0 para homens reflete
preponderncia de gordura distribuda ao redor do quadril ou da regio das coxas e denominada obesidade
ginecoide, em ''forma de pera" ou na regio do quadril.
- O formato de pera, mais comumente encontrado em mulheres, apresenta um risco bem
menor de doena metablica e alguns estudos indicam que possa ser relativamente benigno para a sade.
* Medida de cintura + 101 (homens) e +90 (mulheres) -> obesidade
- Excesso de gordura nos depsitos viscerais (omental e mesentrico) e tambm de gordura
abdominal subcutnea aumenta os riscos sade associados obesidade.
- Diferenas bioqumicas nos depsitos localizados de gordura:
- Especialmente em mulheres, as clulas adiposas subcutneas da regio gluteofemoral so
maiores, mais eficientes para armazenar gordura e tendem a mobilizar cidos graxos mais lentamente que os
adipcitos subcutneos abdominais. Adipcitos viscerais so mais ativos metabolicamente.
- Ambos os depsitos, subcutneo abdominal e visceral, apresentam altas taxas de liplise em
indivduos obesos e contribuem para aumentar a disponibilidade de cidos graxos livres. Essas diferenas
metablicas podem contribuir para o maior risco observado em indivduos com maior adiposidade abdominal.
- Funo endcrina: Leptina (regula o apetite e o metabolismo) / Adiponectina (citocina
produzida pelos adipcitos, reduz os nveis de cidos graxos livres no sangue e tem sido associada a melhora
do perfil lipdico e do controle glicmico e reduo da inflamao em pacientes diabticos.)
- Importncia da circulao porta: citocinas secretadas pelo tecido adiposo, assim como cidos
graxos livres liberados da gordura abdominal, entram na veia porta e, assim, tm acesso direto ao fgado.
cidos graxos e citocinas inflamatrias liberadas do tecido adiposo visceral so captados pelo fgado, o que
pode levar resistncia insulina e ao aumento na sntese de TAGs, liberados como VLDL. Em contraste, os
cidos graxos livres originrios de depsitos subcutneos de gordura entram na circulao geral, de modo que
podem ser oxidados pelo msculo, alcanando o fgado em menor concentrao.
- Tamanho e nmero de adipcitos: Quando os TAGs so depositados nos adipcitos, essas clulas
podem expandir para um tamanho duas a trs vezes maior que o inicial, mas essa expanso limitada.
- Em uma situao de superalimentao prolongada, pr-adipcitos presentes no tecido
adiposo so estimulados a proliferar e diferenciar em clulas adiposas maduras, aumentando o nmero de
adipcitos.
** Assim, a maioria dos casos de obesidade deve-se a uma combinao de hipertrofia e
hiperplasia de adipcitos.
- Se o excesso de calorias no pode ser acomodado no tecido adiposo, o excesso de cidos
graxos ''vaza" para outros tecidos, como msculo e fgado. A quantidade dessa "gordura ectpica" est
fortemente associada resistncia insulina.
- Regulao do peso corporal: cada indivduo tem um "ponto fixo", biologicamente predeterminado,
para seu peso corporal.
- O corpo tenta adicionar tecido adiposo quando seu peso cai abaixo desse ponto fixo (o apetite
aumenta e o gasto de energia diminui) e tenta perder peso quando o peso corporal est acima desse ponto
fixo (diminuio no apetite e a um leve aumento no gasto energtico).
- Contribuies genticas: Mutaes no gene do hormnio leptina, produzido pelo tecido adiposo, ou
em seu receptor produzem hiperfagia (aumento do apetite e do consumo de alimento) e marcante obesidade,
enfatizando a importncia do sistema da leptina na regulao do peso corporal humano. A maioria dos
humanos obesos apresenta nveis elevados de leptina, mas parece ser resistente aos efeitos reguladores do
apetite desse hormnio.
- Contribuies ambientais e comportamentais: Fatores que levam a uma vida sedentria, diminuindo
a atividade fsica e aumentando a tendncia em ganhar peso; fatores que interfiram no comportamento
alimentar do indivduo (variedade de alimentos, tamanho das pores, consumo de lanches).
- Sinais de regulao a longo prazo:
- Leptina: hormnio produzido pelo adipcito e secretado em proporo ao tamanho das
reservas de gordura.
- Quando a massa do tecido adiposo aumenta, a leptina liberada inibe o consumo de alimentos
e a sntese de gordura, estimulando a oxidao dos cidos graxos.
- Quando a massa do tecido adiposo diminui, a produo reduzida da leptina favorece uma
maior ingesto de alimento e uma reduo na oxidao dos cidos graxos.
- Insulina: Indivduos obesos so tambm hiperinsulinmicos. Assim como a leptina, a insulina
atua em neurnios hipotalmicos, diminuindo o apetite.
- Sinais de regulao a curto prazo:
- Na ausncia de ingesto (entre as refeies), o estmago produz grelina, um hormnio
orexignico (estimulador do apetite), que induz fome.
- Durante uma refeio, medida que o alimento consumido, hormnios do intestino,
incluindo a colecistocinina (CCK) e o peptdeo YY (PYY), entre outros, podem atuar como sinais de saciedade
por meio de aes sobre o esvaziamento gstrico e de sinais neurais ao hipotlamo, e a refeio encerrada.
- No hipotlamo, neuropeptdios como o NPY e o hormnio estimulador de -melancitos
(-MSH), e neurotransmissores, como serotonina e dopamina, so importantes reguladores da
fome/saciedade. A leptina pode afetar a sensibilidade de neurnios hipotalmicos a sinais de curto prazo.
- Sndrome metablica: Grupo de anormalidades metablicas que inclui intolerncia glicose,
resistncia insulina, hiperinsulinemia, dislipidemia (baixo nvel de HDL e aumento do nvel de TAG) e
hipertenso. Tambm est associada a um estado de inflamao sistmica crnica, que contribui para a
patognese da resistncia insulina e para a aterosclerose. Baixos nveis de adiponectina podem contribuir
para a sndrome metablica e, assim, para o risco de diabetes tipo 2 e de doena cardaca, pois esse hormnio,
produzido no tecido adiposo, normalmente freia a inflamao e aumenta a sensibilidade dos tecidos,
especialmente do fgado, insulina.

- Adiponectina: inibe processos que consomem energia e estimula processos geradores de energia. A
adiponectina age no crebro, por meio de seus receptores, estimulando a ingesto de alimentos e inibindo a
atividade fsica que consome energia e a termognese na gordura marrom.
PFK-2: fosfofrutocinase 2 / FAS 1: cido graxo-sintase 1 / ACC: acetil-CoA-carboxilase (formao do
malonil-CoA) / HSL: lipase sensvel a hormnio (di -> monoacilglicerol) / HMGR: HMG-CoA-redutase (formao
do mevalonato) / GPAT: aciltransferase / GS: glicognio-sintase
PROBLEMA 4
VISO GERAL
- Em animais, os aminocidos sofrem
degradao oxidativa em trs
circunstncias metablicas diferentes:
1) Durante a sntese e a degradao
normais de protenas celulares, AAs que
no so necessrios para biossntese de
novas protenas sofrem degradao.
2) Dieta rica em protenas:
necessidade do organismo excedida e os
AAs sofrem degradao, j que no podem
ser armazenados.
3) Jejum ou diabetes melito no
controlado, quando carboidratos esto
indisponveis ou so utilizados de modo
inadequado, as protenas celulares so
utilizadas como combustvel.
- Em todas essas condies
metablicas, os AAs perdem seu grupo
amino para formar -cetocidos, os
esqueletos de carbono dos AAs.
- Os a-cetocidos sofrem oxidao
(CO2/H2O) ou fornecem unidades de 3 e 4
carbonos que podem ser convertidas em
glicose (gliconeognese).
- Assim como no catabolismo dos carboidratos e dos cidos graxos, os processos de degradao de AAs
convergem para vias catablicas centrais, com os esqueletos de carbono da maioria dos AAs encontrando uma
via para o ciclo do cido ctrico.
- Caracterstica importante que difere a degradao dos AAs de outros processos catablicos: todos os
AAs contm um grupo amino, e as vias para a degradao dos aminocidos incluem, portanto, uma etapa
fundamental, na qual o grupo -amino separado do esqueleto de carbono e desviado para as vias do
metabolismo do grupo amino.

DESTINOS METABLICOS DOS GRUPOS AMINO


- A maior parte dos AAs metabolizada no fgado. Parte da amnia gerada nesse processo reciclada
e utilizada em uma variedade de vias biossintticas; o excesso excretado diretamente ou convertido em
ureia ou cido rico para excreo, dependendo do organismo.
- O excesso de amnia produzido em tecidos extra-hepticos enviado ao fgado (na forma de grupos
amino) para converso em sua forma de excreo.
- Quatro AAs desempenham papis centrais no metabolismo do nitrognio: glutamato, glutamina,
alanina e aspartato.
- Esses AAs em especial so aqueles mais facilmente convertidos em intermedirios do ciclo do cido
ctrico: glutamato e glutamina so convertidos em -cetoglutarato, alanina em piruvato e aspartato em
oxaloacetato. Glutamato e glutamina atuam como uma espcie de ponto de encontro para os grupos amino.
- No citosol dos hepatcitos, os grupos amino da maior parte dos AAs so transferidos para o
-cetoglutarato, formando glutamato, que entra na mitocndria e perde seu grupo amino para formar NH4+.
- O excesso de amnia produzido na maior parte dos demais tecidos convertido no nitrognio amdico
da glutamina, que circula at chegar ao fgado, entrando na mitocndria heptica. Glutamina, glutamato ou
ambos esto presentes na maior parte dos tecidos em concentraes maiores que os demais AAs.
- No msculo esqueltico, os grupos amino que excedem as necessidades geralmente so transferidos
ao piruvato para formar alanina, transportando o grupo amino para o fgado. O aspartato participa dos
processos metablicos que ocorrem to logo os grupos amino sejam entregues no fgado.
TRANSPORTE DE AAs PARA O INTERIOR DAS CLULAS
- As concentraes de aminocidos
livres nos fluidos extracelulares so mais
baixas que aquelas dentro das clulas do
organismo. Esse gradiente de
concentrao mantido por sistemas de
transporte ativo, impulsionados pela
hidrlise do ATP, necessrios para o
movimento dos aminocidos do espao
extracelular para dentro das clulas.
- O intestino delgado e os tbulos
proximais dos rins apresentam sistemas de
transporte em comum para a captao de
aminocidos; portanto, um defeito em
qualquer desses sistemas resulta na
incapacidade de absorver determinados
AAs pelo intestino e pelos tbulos renais.
- Sistema de transporte de cistina,
ornitina, arginina e lisina. Na doena
herdada cistinria, esse sistema de
carreador apresenta-se deficiente,
resultando no aparecimento de todos os 4
AAs na urina. Expressa-se clinicamente
pela precipitao de cistina, formando
pedras renais que bloqueiam o trato
urinrio.

TRANSAMINAO
- Chegando ao fgado, a primeira etapa no catabolismo da
maioria dos AAs a remoo de seus grupos -amino,
realizada por enzimas denominadas aminotransferases ou
transaminases. Nessas reaes de transaminao, o grupo
-amino transferido para o carbono do -cetoglutarato,
liberando o correspondente -cetocido, anlogo do
aminocido.
- No ocorre desaminao efetiva nessas reaes, pois o
-cetoglutarato torna-se aminado enquanto o -aminocido
desaminado. O efeito das reaes de transaminao coletar
grupos amino de diferentes aminocidos, na forma de
glutamato. O glutamato ento funciona como doador de
grupos amino para vias biossintticas ou para vias de excreo,
que levam eliminao de produtos de excreo nitrogenados.
- Essas enzimas so encontradas no citosol e na
mitocndria das clulas em todo o organismo especialmente
aquelas do fgado, rins, intestino e msculo.
- As reaes catalisadas pelas aminotransferases so livremente reversveis, tendo uma constante de
equilbrio de cerca de 1,0 (G ~ 0 kJ/mol).
- Especificidade das aminotransferases: Cada aminotransferase especfica para um ou, no mximo,
uns poucos doadores de grupos amino, sendo designadas a partir do doador especfico do grupo amino (j
que o aceptor do grupo amino quase sempre o -cetoglutarato). As duas reaes mais importantes de
aminotransferases so catalisadas pela alanina-aminotransferase (ALT) e aspartato-aminotransferase (AST).
- Alanina-aminotransferase (ALT): Catalisa a transferncia do grupo amino da alanina para o
-cetoglutarato, resultando na formao de piruvato e glutamato.
- A reao facilmente reversvel, no entanto, durante o catabolismo de
AAs, a enzima funciona na direo de sntese de glutamato. Assim, o
glutamato atua, efetivamente, como um coletor de nitrognio.
- Aspartato-aminotransferase (AST): Durante o catabolismo dos
aminocidos, a AST transfere grupos amino do glutamato para o oxalacetato,
formando aspartato.
- Essa enzima a aminotransferase mais ativa na maioria dos
tecidos de mamferos, evidenciando a importncia da transaminao entre
glutamato e aspartato. O aspartato funciona como a segunda fonte de grupo
amino no ciclo da ureia.

- Mecanismo de ao das aminotransferases: Todas as aminotransferases apresentam o mesmo grupo


prosttico e o mesmo mecanismo de reao. O grupo prosttico o piridoxal-fosfato (PLP), a forma de
coenzima da piridoxina (vitamina B6).
- O PLP funciona como carreador intermedirio de grupos amino, no
stio ativo das aminotransferases. Ele sofre transformaes
reversveis entre sua forma aldedica, o piridoxal-fosfato, que pode
aceitar um grupo amino, e sua forma aminada, a piridoxamina-
fosfato, que pode doar seu grupo amino para um -cetocido.
- Geralmente, o piridoxal-fosfato encontra-se ligado covalentemente
ao stio ativo da enzima por meio de uma ligao aldimina (base de
Schiff) com o grupo amino de um resduo especfico de lisina.
- Equilbrio das reaes: Livremente reversveis, permitindo que a
reao funcione em ambos os sentidos: degradao do AA pela
remoo do grupo -amino (Ex.: aps o consumo de uma refeio rica
em protenas) e biossntese pela adio de grupos amino a esqueletos
carbonados de -cetocidos (Ex.: quando o suprimento de AAs a
partir da dieta no for adequado para satisfazer as necessidades de
sntese das clulas).
- Valor diagnstico das aminotransferases plasmticas: As
aminotransferases so, normalmente, enzimas intracelulares, de
modo que os baixos nveis observados no plasma representam a
liberao de contedos celulares durante a renovao celular normal.
- Os nveis plasmticos de AST (TGO) e ALT (TGP) esto elevados
em quase todas as doenas hepticas, mas esto especialmente altos
em condies que causam ampla necrose celular, como hepatite viral
grave, hepatite isqumica ou intoxicao pelo paracetamol
(acetaminofeno). ** A ALT mais especfica que a AST para doenas hepticas, mas esta ltima mais sensvel,
pois o fgado contm maiores quantidades de AST.
- Faixa normal: ALT = 7-41 U/L /// AST = 12-38 U/L
- As aminotransferases podem estar elevadas em doenas no hepticas, como infarto do miocrdio,
doenas musculares e pancreatite aguda.
GLUTAMATO-DESIDROGENASE
- Os grupos amino de muitos -aminocidos so coletados,
no fgado, na forma do grupo amino de molculas de glutamato.
Esses grupos amino devem ser removidos do glutamato e
preparados para excreo. Nos hepatcitos, o glutamato
transportado do citosol para a mitocndria, onde sofre
desaminao oxidativa, catalisada pela glutamato-
desidrogenase. Nos mamferos, essa enzima est presente na
matriz mitocondrial.
- Essas reaes ocorrem principalmente no fgado e no rim.
Elas fornecem -cetocidos, que podem entrar nas vias centrais
do metabolismo energtico, e amnia, fonte de nitrognio na
sntese de ureia.
* A glutamato-desidrogenase especfica para glutamato,
e no se conhecem desidrogenases anlogas para outros AAs.
- Assim, a ao sequencial da transaminao com a
subsequente desaminao oxidativa desse glutamato
(transaminao + desaminao oxidativa = transdesaminao),
regenerando -cetoglutarato, fornece uma via por meio da qual
os grupos amino da maioria dos AAs convergem para dois nicos
compostos: NH4+ e aspartato.
- Coenzimas: A glutamato-desidrogenase a nica enzima
que utiliza NAD+ ou NADP+ como aceptor de equivalentes
redutores. O NAD+ utilizado principalmente na desaminao
oxidativa. Por sua vez, o NADPH utilizado na aminao redutora.
- Sentido das reaes: Depende das concentraes
relativas de glutamato, -cetoglutarato e amnia e da razo entre
coenzimas oxidadas e reduzidas. Aps refeio contendo
protena, os nveis de glutamato no fgado esto elevados, e a
reao ocorre no sentido da degradao de AAs e da formao
de amnia. A reao tambm pode ser utilizada para sintetizar AAs a partir dos -cetocidos correspondentes.
- Reguladores alostricos: GTP inibidor alostrico da glutamato-desidrogenase, ao passo que ADP
ativador dessa enzima. Desse modo, quando os nveis energticos esto baixos na clula, a degradao dos
aminocidos pela glutamato-desidrogenase est aumentada, facilitando a produo de energia a partir dos
esqueletos carbonados derivados dos aminocidos.
** Os D-AAs no so utilizados na sntese de protenas de mamferos, mas podem ser metabolizados.
A D-aminocido-oxidase (DAO) uma enzima peroxissomal dependente de FAD que catalisa a desaminao
oxidativa desses ismeros de aminocidos, produzindo -cetocidos, amnia e perxido de hidrognio.
CICLO DA UREIA
- A ureia a principal forma de eliminao dos grupos amino oriundos dos aminocidos e perfaz cerca
de 90% dos componentes nitrogenados da urina.
- Um tomo de nitrognio da molcula de ureia fornecido por NH4+ livre, e o outro nitrognio
fornecido pelo aspartato.
- O glutamato o precursor imediato tanto do nitrognio da amnia (desaminao oxidativa pela
glutamato-desidrogenase) quanto do nitrognio do aspartato (por transaminao do oxalacetato pela AST)
- O fgado tambm recebe parte da amnia pela veia porta, sendo essa amnia produzida no intestino
pela oxidao bacteriana de aminocidos.
- O carbono e o oxignio da ureia so derivados do CO2. A ureia produzida pelo fgado e ento
transportada pelo sangue at os rins, para ser excretada na urina.
- As snteses de carbamoil-fosfato e citrulina ocorrem dentro da mitocndria heptica, ao passo que as
demais enzimas do ciclo esto localizadas no citosol.
FORMAO DO CARBAMOIL-FOSFATO
- O NH4+ presente na mitocndria heptica utilizado imediatamente, juntamente com o CO2 (como
HCO3-) produzido pela respirao mitocondrial, para formar carbamoil-fosfato na matriz. Essa reao,
catalisada pela enzima carbamoil-fosfato-sintetase 1, impulsionada pela clivagem de 2 molculas de ATP.
- No final, o tomo de nitrognio originrio dessa amnia torna-se um dos nitrognios da ureia.
- A carbamoil-fosfato-sintetase 1 requer N-acetil-glutamato como ativador alostrico.
* A carbamoil-fosfato-sintetase 2 participa na biossntese das pirimidinas. Ela no necessita de
N-acetil-glutamato, utiliza glutamina como fonte de nitrognio e a reao ocorre no citosol.
- O carbamoil-fosfato, que funciona como doador ativado de grupos carbamoila, entra no ciclo da ureia.

FORMAO DA CITRULINA
- A poro carbamoila do carbamoil-fosfato transferida para a ornitina pela ornitina
transcarbamoilase (OTC) ao mesmo tempo em que hidrolisada a ligao fosfato de alta energia do substrato,
sendo o fosfato liberado como Pi e formando citrulina.
- A ornitina e a citrulina so AAs bsicos que participam do ciclo da ureia, movendo-se atravs da
membrana mitocondrial interna via um cotransportador. A ornitina sintetizada a partir do glutamato, em
uma via com cinco etapas.
- Esses aminocidos no so incorporados nas protenas celulares, pois no h cdons para eles.
- A ornitina desempenha um papel que se assemelha quele do oxaloacetato no ciclo do cido ctrico,
aceitando material a cada volta do ciclo da ureia.
- A citrulina produzida passa da mitocndria para o citosol.
SNTESE DO ARGININO-SUCCINATO
- Os prximos dois passos trazem o segundo grupo amino que ser incorporado na ureia. A fonte o
aspartato produzido na mitocndria por transaminao e transportado para o citosol.
- O grupo carbonila da citrulina condensa com o grupo amino do aspartato, numa reao catalisada
pela arginino-ssuccinato-sintetase, para formar arginino-ssuccinato.
- A formao do arginino-ssuccinato possibilitada pela clivagem do ATP (3 e ltimo consumido) em
AMP e pirofosfato (PPi).

CLIVAGEM DO ARGININO-SUCCINATO
- O arginino-ssuccinato clivado pela arginino-ssuccinato liase (ou arginino-succinase), produzindo
arginina e fumarato.
- A arginina formada nessa reao serve como precursor imediato da ureia. O fumarato produzido no
ciclo da ureia hidratado, gerando malato, fornecendo um elo de ligao com diversas vias metablicas
(Malato pode ser transportado para mitocndria e entrar no ciclo de Krebs, sendo oxidado a oxaloacetato,
que entra na gliconeognese. Alm disso, o oxaloacetato pode ser convertido em aspartato via transaminao
e entrar no ciclo da ureia. Essas reaes constituem a lanadeira aspartato-arginino-succinato).
CLIVAGEM DA ARGININA
- A arginase cliva a arginina em ornitina e ureia.
- Essa enzima ocorre quase exclusivamente no fgado. Dessa forma, enquanto outros tecidos (como o
rim) podem sintetizar arginina por meio dessas reaes, apenas o fgado pode clivar a arginina e, na sequncia,
sintetizar a ureia.

- Destino da ureia: Depois de sair do fgado por difuso, a ureia transportada no sangue at os rins,
onde filtrada e excretada na urina.
- Parte da ureia difunde do sangue para o intestino, onde clivada em CO2 e NH3 pela urease
bacteriana. Essa amnia parcialmente perdida nas fezes e parcialmente reabsorvida para o sangue.
* Em pacientes com insuficincia renal, os nveis de ureia no plasma aumentam, promovendo maior
transferncia de ureia do sangue para o intestino. A ao da urease intestinal sobre essa ureia torna-se uma
fonte clinicamente importante de amnia, contribuindo para a hiperamonemia frequentemente observada
nesses pacientes.
* Administrao oral de neomicina reduz o nmero de bactrias intestinais responsveis pela produo
de NH3.
- Estequiometria geral: 2 molculas de ATP so necessrias na formao do carbamoil-fosfato e 1 ATP
para produzir arginino-succinato este ltimo ATP sendo clivado em AMP e PPi, que hidrolisado em 2 Pi.
2 NH4+ (glutamato/aspartato) + HCO3- (CO2) + 3 ATP + H2O ureia + fumarato + 2 ADP + AMP + 4 Pi
- O glutamato o precursor imediato de ambos os nitrognios, o da amnia (por desaminao
oxidativa, catalisada pela glutamato-desidrogenase) e o do aspartato (por transaminao a partir do
oxalacetato, catalisada pela AST).
- O fumarato convertido em malato e este transportado para dentro da mitocndria. Dentro da
matriz, NADH gerado na reao da malato-desidrogenase. Cada molcula de NADH pode gerar at 2,5 ATP
durante a respirao mitocondrial, reduzindo muito o custo energtico geral da sntese de ureia.
- Regulao: A longo prazo, a regulao se d pela sntese das enzimas envolvidas, desde a formao
do carbamoil-fosfato. (Ex.: so sintetizadas em taxas
mais altas em animais em jejum e em animais com dietas
de alto contedo proteico).
- Curto prazo: O N-acetil-glutamato um ativador
essencial da carbamoil-fosfato-sintetase1, o passo
limitante da velocidade para o ciclo da ureia.
- O N-acetil-glutamato sintetizado a partir da
acetil-CoA e do glutamato pela N-acetil-glutamato-
sintase, em uma reao ativada pela arginina.
- Assim, a concentrao intra-heptica de N-acetil-
glutamato aumenta aps a ingesto de uma refeio rica
em protena, que fornece tanto o substrato (glutamato)
quanto o regulador da sntese de N-acetil-glutamato
(arginina). Isso leva ao aumento na velocidade de sntese
de ureia.
METABOLISMO DA AMNIA
- A amnia produzida por todos os tecidos durante o metabolismo de uma variedade de compostos
e eliminada principalmente pela formao de ureia no fgado.
- O nvel de amnia no sangue, no entanto, deve ser mantido muito baixo, pois mesmo concentraes
ligeiramente aumentadas (hiperamonemia) so txicas para o sistema nervoso central (SNC).
FONTES DE AMNIA
- Os aminocidos so, quantitativamente, a mais importante fonte de amnia. O excesso de
aminocidos viaja ao fgado e sofre transdesaminao (transaminao + desaminao oxidativa), produzindo
amnia.
- Glutamina: Os rins produzem amnia a partir da glutamina (pela ao da glutaminase renal) e da
glutamato-desidrogenase. A maior parte dessa amnia excretada na urina como NH4 +, o que fornece um
mecanismo importante para a manuteno do balano acidobsico do organismo, pela excreo de prtons.
- A amnia tambm obtida pela hidrlise da glutamina pela glutaminase intestinal. As clulas
da mucosa intestinal obtm glutamina a partir do sangue ou da digesto de protena da dieta.
- Ao bacteriana no intestino: A amnia produzida a partir da ureia por ao da urease bacteriana
no lmen intestinal. Essa amnia absorvida pelo intestino, atinge a veia porta e quase quantitativamente
removida pelo fgado, pela converso em ureia.
- Aminas: As aminas obtidas da dieta e as monoamidas utilizadas como hormnios ou
neurotransmissores podem produzir amnia por ao da aminoxidase.
- Purinas/Pirimidinas: No catabolismo das purinas e das pirimidinas, os grupos amino ligados aos anis
so liberados como amnia.
TRANSPORTE DA AMNIA NA CIRCULAO
- Ureia: A formao de ureia no fgado , quantitativamente, a via mais importante de eliminao da
amnia. A ureia circula no sangue, do fgado para os rins, onde passa pela filtrao glomerular.
- Glutamina: Essa amida do cido glutmico fornece uma forma de armazenamento e de transporte
no txicos para a amnia.
- A amnia livre produzida nos tecidos combina-se com o
glutamato, produzindo glutamina, pela ao da glutamina-
sintetase. Essa reao requer ATP e ocorre em duas etapas.
- Inicialmente, o glutamato e o ATP reagem para formar
ADP e um intermedirio glutamil-fosfato, que ento reage
com a amnia, produzindo glutamina e fosfato inorgnico.
- Essa formao ocorre principalmente no msculo e no
fgado, mas tambm importante no sistema nervoso central,
sendo o principal mecanismo de remoo da amnia no
encfalo.
- A glutamina encontrada no plasma em concentraes
mais altas que outros aminocidos. Tambm serve como fonte
de grupos amino em vrias reaes biossintticas.
- A glutamina que excede as necessidades de biossntese
transportada pelo sangue para o intestino, o fgado e os rins,
para ser processada. O nitrognio amdico liberado como on
amnio na mitocndria, onde a enzima glutaminase converte
glutamina em glutamato e NH4+.
- O NH4+ do intestino e dos rins transportado no sangue
para o fgado. No fgado, a amnia de todas essas fontes
utilizada na sntese da ureia.
- O glutamato pode ser processado pela glutamato-
desidrogenase (liberando mais amnia e esqueletos de
carbono para combustvel) ou pode entrar em reaes de
transaminao para a biossntese de AAs e para outros
processos.
- Ciclo da glicose-alanina:

- No msculo e em alguns outros tecidos que


degradam aminocidos como combustvel, os
grupos amino so coletados na forma de glutamato,
por transaminao.

- O glutamato pode ser convertido em glutamina


para transporte ao fgado, como descrito
anteriormente, ou pode transferir seu grupo -
amino para o piruvato, produto da gliclise
muscular facilmente disponvel, pela ao da
alanina-aminotransferase. A alanina assim
produzida passa para o sangue e segue para o
fgado.
- No citosol dos hepatcitos, a alanina-aminotransferase transfere o grupo amino da alanina para o
-cetoglutarato, formando piruvato e glutamato. O glutamato ento entra na mitocndria, onde a reao da
glutamato-desidrogenase libera NH4+, ou sofre transaminao com o oxaloacetato para formar aspartato,
outro doador de nitrognio para a sntese de ureia.
- Hiperamonemia: A capacidade do ciclo heptico da ureia excede as velocidades normais de produo
de amnia, e os nveis de amnia srica so normalmente baixos (5 a 35 mol/L).
- No entanto, quando a funo heptica est comprometida, devido a defeitos genticos no ciclo da
ureia ou doena heptica, os nveis sanguneos de amnia podem elevar-se acima de 1.000 mol/L.
- Sintomas de intoxicao por amnia: tremores, discurso inarticulado, sonolncia, vmito, edema
cerebral e viso borrada.
- A remoo do excesso de amnica no crebro requer a aminao redutora do -cetoglutarato a
glutamato (inverso da degradao) e a converso de glutamato em glutamina (via mais importante no crebro)
- Os estgios finais da intoxicao so caracterizados por induo de um estado de coma, acompanhado
por edema cerebral (aumento no contedo de gua do crebro) e aumento da presso intracraniana.
- NH4+ glutamina soluto osmoticamente ativo astrcitos (cls de nutriente e suporte dos
neurnios) captam mais gua (para manter o equilbrio osmtico) edema das clulas e do crebro coma.
- Glutamato e seu derivado (y-aminobutirato, GABA) so importantes neurotransmissores. Assim, a
depleo do glutamato pela reao da glutamina-sintase pode ter efeitos adicionais no crebro.
- Adquirida: doenas hepticas (hepatite viral, hepatotoxinas, como o lcool). Cirrose heptica pode
resultar na formao de circulao colateral ao redor do fgado, fazendo com que o sangue da circulao porta
seja lanado diretamente na circulao sistmica e no tenha acesso ao fgado, aumento os nveis de amnia
circulante.
- Gentica: deficincia de ornitina-transcarbamoilase a mais comum. Em todos os casos, a falha em
sintetizar ureia leva hiperamonemia durante as primeiras semanas aps o nascimento.
- O tratamento inclui a limitao de protena na dieta, na presena de calorias em quantidade suficiente
para prevenir o catabolismo.
- A administrao de compostos que se ligam covalentemente a aminocidos, produzindo molculas
com nitrognio que so excretadas na urina, tem aumentado a sobrevida dos pacientes.
- A administrao cuidadosa na dieta dos cidos aromticos benzoato ou fenilbutirato pode ajudar a
diminuir os nveis de amnia no sangue.
- O benzoato convertido em benzoil-CoA, que se combina com a glicina, formando hipurato. A glicina
utilizada nessa reao deve ser regenerada, e a amnia captada pela reao da glicina-sintase.
- O fenilbutirato convertido em fenilacetato pela -oxidao. O fenilacetato ento convertido em
fenilacetil-CoA, que se combina com a glutamina, formando fenilacetilglutamina. A resultante remoo de
glutamina desencadeia um aumento em sua sntese pela glutamina-sintetase, em uma reao que capta
amnia.
- Tanto o hipurato quanto a fenilacetilglutamina so compostos no txicos e so excretados na urina.
- A deficincia de N-acetilglutamato-sintase resulta na ausncia do ativador normal para a carbamoil-
fosfato-sintetase 1. Essa condio pode ser tratada pela administrao de carbamoil-glutamato, um anlogo
do N-acetilglutamato, que ativa efetivamente a carbamoil-fosfato-sintetase 1.
- A suplementao da dieta com arginina til no tratamento de deficincias de ornitina-
transcarbamoilase, arginino-succinato-sintetase e de arginino-succinase.
- Nos raros casos de deficincia de arginase, a arginina,
substrato da enzima defeituosa, deve ser suprimida da dieta.
PROBLEMA 5
METABOLISMO DO ETANOL VISO GERAL
- A principal rota do metabolismo do etanol atravs da lcool-desidrogenase heptica, que oxida o
etanol a acetaldedo no citosol. Posteriormente, o acetaldedo oxidado a acetato pela acetaldedo-
desidrogenase (principalmente na mitocndria).
- O acetaldedo, txico, pode entrar na corrente sangunea, gerando efeitos desagradveis.
- O NADH produzido nessas reaes utilizado para gerar ATP na fosforilao oxidativa.
- A maior parte do acetato que atinge o sangue captada para clivagem a acetil-CoA (que oxidado).
- 10 a 20% do etanol ingerido oxidado pelo sistema microssomal oxidante (MEOS), que
compreende as enzimas do citocromo P450 no RE (principalmente CYP2E1). A CYP2E1 tem alto Km para o
etanol e induzida por ele, ou seja, essa rota maior em altas concentraes de lcool/consumo crnico.
- Os principais efeitos agudos da ingesto do lcool so produzidos pelo grande aumento da frao
NADH/NAD+. Como consequncia, tem-se:
- Inibio da -oxidao (por consequncia, aumento da sntese de triacilgliceris, j que h
alta de glicerol-fosfato e cidos graxos);
- Diminuio da gliclise (por consequncia, aumento da sntese de glicerol-fosfato pelos
intermedirios glicolticos);
- Inibio da gliconeognese (so necessrias reaes que consumam NADH, logo, substratos
da gliconeognese sero desviados: piruvato lactato; oxaloacetato malato; glicerol-fosfato sntese
de triacilgliceris);
- Aumento da cetognese (se oxaloacetato malato, haver pouco oxaloacetato para reagir
com acetil-CoA, formando citrato. Logo, acetil-CoA passa para rota de sntese de corpos cetnicos).
- Diminuio da excreo de cido rico (o lactato, elevado, compete com o cido rico para
ser eliminado na urina, gerando uricemia, predispondo o desenvolvimento de gota).
- Pode resultar em doenas hepticas induzidas pelo lcool, incluindo esteatose heptica, hepatite
induzida por lcool e cirrose.
- O principal produto txico do metabolismo inclui acetaldedo e radicais livres. O acetaldedo forma
aductos com protenas e outros compostos. O radical hidroxietila produzido por MEOS e outros radicais
produzidos durante a inflamao causam alteraes irreversveis no fgado.

METABOLISMO DO ETANOL
- Etanol: molcula pequena solvel tanto em leo quanto em gua (assim, rapidamente absorvido
pelo intestino por difuso passiva). 0 a 5% do etanol ingerido entra na mucosa gstrica de clulas do trato GI
alto (lngua, boca, esfago e estmago), onde metabolizado, e o restante passa para o sangue. Desse, 85 a
95% so metabolizados no fgado, e 2 a 10% so excretados pelos pulmes e rins.
- Principal rota de metabolismo no fgado: lcool-desidrogenase no citosol, que oxida etanol a
acetaldedo, com reduo do NAD+ a NADH. Acetaldedo no metabolizado efeito txico no fgado/outros.
- 90% do acetaldedo acetato, com formao de NADH, pela acetaldedo-desidrogenase na
mitocndria. Possui baixo Km.
- Acetato, geralmente no txico, pode ser ativado a acetil-CoA no fgado (ciclo Krebs/sntese de
cidos graxos), mas a maior parte ativado a acetil-CoA no msculo esqueltico e em outros tecidos.
- Outra importante rota de degradao: sistema microssomal oxidante de etanol (MEOS), que
tambm oxida lcool a acetaldedo. A principal enzima microssomal envolvida a isoenzima oxidase do
citocromo P450 com funo mista (CYP2E1). Utiliza NADPH como um doador adicional de eltrons e O2 como
receptor de eltrons. Essa rota responsvel pela oxidao de 10/20% do lcool em consumo moderado.
- Variaes individuais na quantidade das isoenzimas responsveis pela oxidao do etanol podem
determinar: frao de depurao de etanol pelo sangue, grau de embriaguez, diferenas na suscetibilidade
do indivduo em desenvolver doenas hepticas causadas pelo lcool.
LCOOL-DESIDROGENASE (ADH)
- Famlia de isoenzimas com variao especfica no comprimento da cadeia do substrato do lcool. O
etanol metabolizado de forma no-especfica por vrios membros da famlia ADH.
- As ADHs que apresentam maior afinidade pelo etanol so da classe ADH-I. Possuem baixo Km para
o etanol e esto presentes em altas quantidades no fgado, ou seja, o fgado o principal stio de
metabolizao do lcool e de formao do acetaldedo.
- Embora as ADHs das classes II e IV contribuam menos para o metabolismo, podem contribuir para
seu efeito txico apesar de possurem alto Km, concentraes de etanol podem ser muito altas na parte
superior do trato GI, e a formao de acetaldedo nesse local pode contribuir para o risco de cncer associado
a bebidas leves.
ACETALDEDO-DESIDROGENASE (ALDH)
- Mais de 80% da oxidao do acetaldedo no fgado feita pela ALDH mitocondrial (ALDH 2), que
apresenta alta afinidade e bastante especfica. O restante oxidado pela ALDH citoslica (ALDH 1). ALDHs
adicionais agem em uma variedade de lcoois orgnicos, toxinas e poluentes.
- Acmulo de acetaldedo nusea, vmitos. ALDH inativa considerada fator antialcoolismo.
- Tratamento para alcoolistas envolve inibio da ALDH (Dissulfiram), gerando um efeito antabuse: a
droga inibe o ALDH, gerando acmulo de acetaldedo, responsvel pelos efeitos desagradveis do lcool
(causa extrema vasodilatao e consequente queda de presso arterial, taquicardia e cefaleia). O paciente
rejeita o lcool por associao aos efeitos relatados, que se manifestam quando se utiliza da bebida.
DESTINO DO ACETATO
- Metabolismo do acetato requer ativao a acetil-CoA pela acetil-CoA sintase (ou ACS 1, em reao
similar catalisada pela acetil-CoA-sintetase graxa).
- No fgado, a principal isoforma da ACS 1 uma enzima citoslica que forma acetil-CoA pela rota
citoslica da sntese de colesterol e cidos graxos. A entrada de acetato nessa rota est sob o controle
regulatrio de mecanismos envolvendo colesterol e insulina, desse modo, a maioria do acetato passa para o
sangue.
- O acetato captado e oxidado por outros tecidos, principalmente corao e msculo esqueltico,
que possuem alta concentrao mitocondrial da isoforma da ACS (ACS 2), presente na matriz, formando
acetil-CoA, que pode entrar diretamente no ciclo de Krebs e ser oxidada a CO2.
SISTEMA MICROSSOMAL OXIDANTE DE ETANOL
(MEOS)
- Envolve membros de superfamlias de enzimas do
citocromo P450.
- Etanol e NADPH doam eltrons na reao, que reduz O2
a H2O, com formao de acetaldedo.
- Todas as enzimas do citocromo P450 tm 2
componentes de protena cataltica principais: um sistema
doador de eltrons que os transfere a partir do NADPH
(citocromo P450-redutase) e um citocromo P450. A protena
do citocromo P450 contm os stios de ligao para o O2 e o
substrato (etanol).
CYP2E1
- Dentro da superfamlia, a isoenzima com maior atividade para o etanol a CYP2E1 (mas no a nica).
O MEOS refere a atividade de oxidao do etanol combinada por todas as enzimas P450.
- Possui Km muito maior para o etanol que ADHs-I. Assim, essa via mais ativa em nveis maiores de
consumo de etanol.
INDUO DE ENZIMAS P450
- O consumo crnico de etanol aumenta os nveis hepticos de CYP2E1 cerca de 5 a 10x e causa um
aumento de 2 a 4x em algumas das outras P450 da mesma famlia e subfamlias. O retculo endoplasmtico
sofre proliferao, com um aumento geral no contedo das enzimas microssomais, incluindo aquelas que
no esto diretamente envolvidas no metabolismo do etanol.
- Embora o aumento de CYP2E1 aumente a depurao do etanol do sangue, ela tem consequncias
negativas: produo de acetaldedo maior que degradao pelo ALDH, aumentando o risco de danos
hepticos, e uma maior quantidade dele pode passar para o sangue e prejudicar outros tecidos. Alm disso,
as enzimas do citocromo P450 so capazes de formar radicais livres, que tambm podem levar a aumento
de danos hepticos e cirrose.
VARIAO NO PADRO DO METABOLISMO DO ETANOL
- Diferenas no metabolismo do etanol podem influenciar se um indivduo passa a ser um alcolatra,
desenvolve doenas hepticas induzidas ou adquire outras patologias associadas ao aumento do consumo
(como hepatocarcinogneses, cncer de pulmo ou de mama).
- Gentipo: Formas polimorfas de ADH e ALDH podem afetar a velocidade de oxidao e a
acumulao de acetaldedo. A atividade da CYP2E1 pode variar em at 20x entre indivduos.
- Histrico de bebida: Da fase aguda at um consumo crnico de lcool, ocorre diminuio de ADH
gstrica e aumento de CYP2E1, progressivamente.
- Sexo: Mulheres possuem menor atividade da ADH gstrica, so em geral menores e tm uma
composio corporal com mais gordura e menos gua, gerando assim, um nvel sanguneo maior de etanol
no sangue aps o consumo de uma bebida em relao aos homens.
- Quantidade: A quantidade de etanol consumida por um indivduo em um pequeno espao de tempo
determina a rota metablica dessa substncia.
- quantidade rota de baixo Km ADH-I e ALDH-II
- quantidade rota de alto Km MEOS ( acetaldedo/radicais doenas)
ENERGIA PRODUZIDA
- Rota ADH/ALDH: Cada enzima produz um NADH, logo so formados 5 ATP. A oxidao do acetil-
CoA no ciclo de Krebs produz 10 ATP (3 NADH, 1 FADH2 e 1 GTP). Assim, a produo bruta de 15 ATP. Mas
a ativao a acil-CoA requer 2 ligaes fosfato de alta energia (Uma na quebra de ATP AMP + pirofosfato
e outra na quebra de PPi 2 Pi). Assim, a produo lquida de 13 ATP / mol de etanol.
- Rota MEOS: A oxidao a acetaldedo gasta 2,5 ATP na forma de NADPH. A oxidao a acetato pela
ALDH produz 2,5 ATP na forma de NADH. Assim, produo lquida s ocorre pela oxidao do acetil-CoA no
ciclo de Krebs (10 ATP, menos 2 ATP para ativao do acetato) gerando 8 ATP / mol de etanol.

EFEITO TXICO DO METABOLISMO


- Doenas induzidas podem se manifestar em 3 formas: esteatose heptica, hepatite induzida ou
fibrose (cirrose).
- Efeito agudo no metabolismo heptico: Inibio da oxidao de cidos graxos e estmulo da sntese
de TAGs, levando esteatose. Isso pode resultar tambm em cetoacidose ou acidose lctica e causar
hipoglicemia ou hiperglicemia, dependendo do estado diettico.
- O acetaldedo e os radicais livres podem resultar em hepatites induzidas, gerando inflamao,
necrose e morte das clulas. Danos difusos aos hepatcitos resultam em cirrose (fibrose, cicatrizante;
alterao na arquitetura normal e fluxo sanguneo; perda da funo e insuficincia hepticas).
EFEITOS PELO AUMENTO DA PROPORO NADH/NAD+
- ingesto proporo da oxidao regulada pela quantidade de lcool consumida e pela taxa
de NADH reoxidada em cadeias transportadoras de eltrons. No entanto, o NADH no efetivo na inibio
de ADH ou ALDH, e no existe outra regulao retroativa por ATP, ADP, AMP. Logo, ocorre acmulo de NADH
no citosol e na mitocndria, aumentando a proporo NADH/NAD+ (acmulo pode ser ainda maior se
ocorrer danos na mitocndria por acetaldedo ou radicais).
- Modificaes no metabolismo de cidos graxos:
- NADH/NAD+ inibio da oxidao de cidos graxos (j que, por exemplo, a -oxidao
responsvel pela formao tanto de acetil-CoA quanto de equivalentes redutores, o NADH).
- Assim, esses cidos so reesterificados em TAGs pela combinao de com glicerol-fosfato
( NADH inibe oxidao do gliceraldedo-3-fosfato a 1,3-bifosfoglicerato. Assim, ele isomerizado a di-
hidroxiacetona-fosfato, que reduzida, com consumo de NADH, a glicerol-3-fosfato).
- Os TAGs so incorporados em VLDLs, que acumulam no fgado e passam ao sangue,
resultando em hiperlipidemia induzida.
- O consumo crnico acentua a esteatose heptica: o consumo crnico aumenta a rota MEOS
as acil-CoA transferases so enzimas microssomais, logo, tambm so induzidas pelo consumo de lcool
a reesterificao de c. graxos a TAGs acentuada.
- cidos graxos: a partir da gordura da dieta, sntese no fgado ou liberao dos estoques dos
adipcitos. A liplise nos adipcitos aumenta aps consumo de etanol devido liberao de epinefrina.
- Cetoacidose induzida: NADH no h necessidade de oxidar acetil-CoA no ciclo de Krebs (pelo
NADH e pelo oxaloacetato, que direcionado para sntese de malato, com consumo de NADH. Assim, no
h oxaloacetato para reagir e formar citrato) acetil-CoA passa para rota de sntese de corpos cetnicos.
- Apesar da alta produo, o metabolismo restrito pelo suplemento de acetato (que o
principal substrato) concentrao plasmtica do que sob condies normais de jejum.
- Acidose lctica, hiperuricemia e hipoglicemia:
- NADH equilbrio da lactato-desidrogenase deslocado para formao de lactato (j que
a converso de piruvato a lactato consome NADH) lactato acidose lctica.
- A elevao de lactato no sangue pode diminuir a excreo de cido rico: o lactato, quando
elevado, compete com o cido rico para ser eliminado na urina, gerando hiperuricemia. Assim, pacientes
com gotas devem evitar consumo excessivo de lcool. * degradao purinas tambm causa hiperuricemia.
- NADH pode causar hipoglicemia em indivduos em jejum que estiverem ingerindo bebida
alcolica: alanina e lactato so os principais precursores gliconeognicos NADH desloca a lactato-
desidrogenase para formar lactato (fazendo com que o piruvato formado a partir da alanina tambm seja
convertido em lactato) outros precursores importantes no entram na via (oxaloacetato, glicerol).
- Hiperglicemia transitria: o consumo de etanol associado a uma refeio pode aumentar a
glicose circulante, j que a alta de NADH inibe a oxidao do gliceraldedo-3-fosfato, impedindo a gliclise.

TOXICIDADE DO ACETALDEDO
- Produzido tanto por ADH quanto por MEOS, acumula-se no fgado e liberado para dentro da
corrente sangunea aps altas doses de lcool. Sendo altamente reativo, liga-se covalentemente a grupos
amino e sulfidrila, nucleotdeos e fosfolipdios, formando aductos.
- Hepatite induzida por acetaldedo/lcool: formao de aductos de acetaldedo e aminocidos
diminui a sntese de protenas plasmticas (calmodulina, ribonuclease, tubulina). Protenas no corao e em
outros tecidos tambm podem ser afetadas pelo acetaldedo no sangue.
- Formao de aductos de acetaldedo de tubulina: diminuio da secreo de protenas
sricas e VLDL do fgado. O fgado sintetiza diversas protenas (albumina srica, fatores de coagulao
sangunea, protenas transportadoras para vitaminas, esteroides e ferro), que se acumulam no fgado com
lipdios. Esse acmulo gera afluncia de gua para dentro dos hepatcitos, causando inchao e contribuindo
para hipertenso porta e modificao da arquitetura heptica.
- Acetaldedo e danos por radicais livres: acetaldedo liga-se diretamente glutationa e diminui sua
habilidade em proteger contra H2O2 e prevenir a peroxidao lipdica, alm de se ligar a enzimas de defesa
de radicais livres.
- Consumo crnico mitocndria fica danificada inibe a taxa de transporte de eltrons
desacopla a fosforilao oxidativa diminuio da oxidao de cido graxo acmulo de lipdios.
ETANOL E FORMAO DE RADICAIS LIVRES
- O aumento da produo de radicais livres pela CYP2E1 responsvel pelo aumento do estresse
oxidativo no fgado durante a intoxicao crnica por etanol (transferncia de eltrons desemparelhados por
FAD, FMN, etc, formando radicais).
- O radical hidroxietila (CH3CH2O) produzido durante o metabolismo e pode ser liberado como livre.
- A induo da CYP2E1 (e outras enzimas do citocromo P450) pode aumentar a formao de radicais
livres a partir do metabolismo de drogas e da ativao de toxina e carcinognicos.
** O aumento da concentrao do citocromo P450 (que tambm responsvel pela metabolizao
de diversos antibiticos) pelo consumo de lcool aumenta a resistncia de alcolatras em abstinncia a
alguns medicamentos.
** A ao do lcool sobre antibiticos pode se dar tambm na sua absoro: lcool estimula
diretamente as membranas do aparelho digestivo, produzindo mais HCl (que ioniza o medicamento e
dificulta sua absoro) e aumentando os movimentos de estmago e intestino (a passagem do medicamento
se torna mais rpida).
- Os fosfolipdios so alvo primrio de peroxidao pela liberao de radicais. A peroxidao de
lipdios no interior da membrana mitocondrial contribui para inibir o transporte de eltrons e desacoplar a
mitocndria, levando a inflamao e necrose celular.
CIRROSE HEPTICA E PERDA DA FUNO HEPTICA
- Relacionada com alcoolismo crnico (acima de 80g de etanol dirias).
- Inicia com o fgado aumentado, preenchido de gordura, cruzado com fibras de colgeno (fibroses) e
com ndulos/hepatcitos regenerados entre as fibras. Como a funo heptica est perdida, o fgado torna-
se enrugado.
- Durante o desenvolvimento da cirrose: funes metablicas prejudicadas (rotas de biossntese,
detoxicao), diminuio da sntese de protenas sanguneas (albumina, fatores de coagulao), acmulo de
nveis txicos de amnia no sangue (pela diminuio da capacidade de incorporar grupos amino em ureia),
acmulo de bilirrubina no sangue (pela diminuio da conjugao e excreo de pigmento amarelado da
bilirrubina).
- Apesar do lcool ser a causa mais comum de cirrose no mundo ocidental, sendo responsvel por 60
a 70% de todos os casos, apenas 10 a 15% evolui para cirrose.
PAPEL DA TIAMINA NO ALCOOLISMO
- Na sua forma ativa, essa vitamina essencial como coenzima para o funcionamento timo da
piruvato-desidrogenase (piruvato -> acetil-CoA), da transcetolase (transferncia de fragmento de 2 carbonos
de uma cetose para uma aldose) e da -cetoglutarato-desidrogenase (-cetoglutarato -> succinil-
CoA) em reaes do metabolismo dos carboidratos pelas vias da pentose fosfato, da gliclise, do ciclo do
cido ctrico, formando NADH e ATP.
- O consumo crnico de lcool est relacionado baixa absoro de tiamina pelas clulas intestinais,
bem como menor fosforilao da mesma, em sua forma ativa, e diminuio do estoque heptico de
tiamina. Esses fatores, associados menor ingesto de alimentos contendo tiamina, podem ser uma das
causas da baixa concentrao de tiamina nos dependentes de lcool.
- Sndrome de Wernicke-Korsakoff: potencialmente fatal associada deficiente de tiamina.
- Caracterizada pela trade de anormalidades descritas por Wernicke: oftalmoplegia (paralisia
do globo ocular), ataxia (perturbao da coordenao muscular, onde o movimento controlado apenas
parcialmente) e distrbios mentais e de conscincia.
- As anormalidades oculares consistem em nistagmo (rpido movimento), horizontal ou
vertical, paralisia ou paresia dos msculos retos externos...
- A ataxia de marcha e postural, sendo que nos estgios agudos da doena pode inviabilizar
a deambulao ou postura sem suporte.
- Os distrbios de conscincia e de estado mental ocorrem principalmente como um estado
confusional global, no qual o paciente est aptico, desatento e com mnima expresso verbal espontnea.
No caso do alcoolismo mostram sinais de abstinncia alcolica, com alucinaes, agitao, alterao da
percepo e hiperatividade autonmica. Com a pronta reposio de tiamina o paciente recobra rapidamente
o estado de alerta e a tenacidade.
- Vrios mecanismos tm sido implicados na patognese dessa sndrome, mas ainda no so
totalmente compreendidos. Uma das explicaes so as perdas neuronais e os mecanismos para essa morte
cerebral incluem deficincia energtica cerebral, acidose lctica focal e alterao da barreira
hematoenceflica.
- A acidose lctica focal pode ser um dos mecanismos que levam a uma deficincia de tiamina
cerebral (reduzindo a permeabilidade tiamina no crebro). A explicao mais plausvel para esse fenmeno
parece ser uma diminuio da oxidao do piruvato, resultante da diminuio da atividade das
desidrogenases dependentes de tiamina. Com o acmulo de lactato nos neurnios, h uma alterao de pH
(acidose), gerando morte celular. A intensa formao de radicais livres tambm est associada a quadros de
SWK.
- A sndrome pode ser exacerbada por uma mutao no gene da transcetolase que resulta em
uma enzima com baixa afinidade por TPP uma afinidade dez vezes menor que a normal. Esse defeito torna
os indivduos muito mais sensveis deficincia de tiamina: mesmo uma deficincia moderada de tiamina
faz o nvel de TPP cair abaixo daquele necessrio para saturar a enzima. O resultado uma reduo da
velocidade de toda via das pentoses-fosfato, agravando os sintomas.
- Hipoglicemia e reposio de glicose/tiamina:
- Em casos de reposio de glicose, deve receber 1 ampola de tiamina 100mg trinta minutos
antes, desde que os nveis glicmicos no estejam crticos e ameaadores vida do doente.
- As clulas nervosas utilizam a tiamina na metabolizao da glicose. A ausncia dessa, entre
usurios crnicos, pode desencadear a encefalopatia de Wernicke.
- Indivduos com histria nutricional adequada, tendo feito um uso abusivo e isolado de lcool,
no necessitam de administrao prvia de tiamina.

REPERCUSSES BIOPSICOSSOCIAIS
- Quando consideramos que o consumo e a dependncia de lcool aumentam o risco para problemas
sociais, de trabalho, familiares, fsicos, legais e com violncia, podemos afirmar que ele merece ateno e
configura-se como um problema de sade pblica.
- Os transtornos decorrentes do uso de lcool penalizam enormemente os membros da famlia,
contribuindo para altos nveis de conflito interpessoal, violncia domstica, inadequao parental, abuso e
negligncia infantil, separao e divrcio, dificuldades financeiras e legais e problemas clnicos relacionados
ao uso de lcool.
- Alm disso, as crianas criadas em famlias nas quais outros membros abusam ou so dependentes
de lcool e outras substncias tambm apresentam risco elevado para abuso fsico e sexual.
- Os nveis elevados de estresse social ou transcultural tambm exercem um papel no
desenvolvimento e perpetuao do transtorno decorrente do uso abusivo.
- compreensvel que para os familiares exista uma predisposio descrena do tratamento e da
manuteno da abstinncia, assim como para o paciente difcil entender que est doente e aps essa
compreenso manter-se sbrio. Historicamente explicar comunidade em geral que o alcoolismo uma
doena uma tarefa complexa.
ASSOCIAO DE ALCOLICOS ANNIMOS
- Alcolicos Annimos uma irmandade de homens e mulheres que compartilham suas experincias,
foras e esperanas, a fim de resolver seu problema comum e ajudar outros a se recuperarem do alcoolismo.
- O nico requisito para se tornar membro o desejo de parar de beber.
- Para ser membro de A.A. no h taxas ou mensalidades; eles so autossuficientes, graas s prprias
contribuies.
- A.A. no est ligada a nenhuma seita ou religio, nenhum partido poltico, nenhuma organizao ou
instituio; no deseja entrar em qualquer controvrsia; no apoia nem combate quaisquer causas.
- O propsito primordial manter a sobriedade e ajudar outros alcolicos a alcanarem essa meta.
- A.A. pode ser descrito como um mtodo para recuperao do alcoolismo, no qual os membros
ajudam-se mutuamente, compartilhando entre si uma enorme gama de experincias semelhantes em
sofrimento e recuperao do alcoolismo.
- O que a A.A. faz:
- Membros de A.A. dividem suas experincias com qualquer um que procure ajuda com
problemas de alcoolismo; eles do depoimento cara a cara em reunies ou apadrinhando o alcolico recm-
chegado em A.A.
- O programa de A.A. proposto em Doze Passos, que proporciona ao alcolico uma maneira
de desenvolver satisfatoriamente a vida sem o lcool.
- Este programa apresentado nas reunies de grupo de A.A.
- O que a A.A. no faz:
- Recrutar membros, ou tentar aliciar algum para juntar-se ao A.A.
- Manter registro de seus membros ou de suas histrias.
- Acompanhar ou tentar controlar seus membros.
- Fazer diagnsticos ou prognsticos clnicos ou psicolgicos.
- Providenciar hospitalizao, medicamentos ou tratamento psiquitrico.
- Fornecer alojamento, alimentao, roupas, emprego, dinheiro ou outros servios
semelhantes.
- Fornecer aconselhamento familiar ou profissional.
- Participar de pesquisas ou patrocin-las.
- Filiar-se a entidades sociais (embora muitos membros e servidores cooperem com elas).
- Oferecer servios religiosos.
- Participar de qualquer controvrsia sobre lcool ou outros assuntos.
- Aceitar dinheiro pelos seus servios ou contribuies de fontes no A.A.
- Fornecer cartas de recomendao a juntas de livramento condicional, advogados, oficiais de
justia, escolas, empresas, entidades sociais ou quaisquer outras organizaes ou instituies.

METABOLISMO DAS PURINAS


- Os nucleotdeos so compostos de uma base nitrogenada, um monossacardeo pentose e um, dois
ou trs grupos fosfato.
-DNA e RNA apresentam as mesmas bases pricas: Adenina e Guanina. Ambos contm a pirimidina
Citosina, mas diferem na segunda base pirimdica: DNA contm Timina e RNA contm Uracila.
- Timina e Uracila diferem-se pela presena de um grupo metil na timina e ausente na uracila.
- A adio de um acar pentose a uma base produz um nucleosdeo. Se o acar for a ribose, ser
produzido um ribonucleosdeo, se for a 2-desoxirribose ser produzido um desoxirribonucleosdeo.
- Nucleotdeos so steres mono, di ou trifosfatados dos nucleosdeos.
- Os grupos fosfato so responsveis pelas cargas negativas associadas aos nucleotdeos e tambm
so o motivo pelo qual o DNA e o RNA so chamados ''cidos nuclicos''.
DEGRADAO DE NUCLEOTDEOS PRICOS
- As ribonucleases e as desoxirribonucleases, secretadas pelo pncreas, hidrolisam o RNA e o DNA
oriundos da dieta, produzindo principalmente oligonucleotdeos.
- Os oligonucleotdeos so, a seguir, hidrolisados pelas fosfodiesterases pancreticas, produzindo
uma mistura de 3'- e 5'-mononucleotdeos.
- Nas clulas da mucosa intestinal, uma famlia de nucleotidases remove hidroliticamente os grupos
fosfato, liberando nucleosdeos que so posteriormente degradados a bases livres.
- As bases pricas da dieta no so usadas em quantidade significativa para a sntese dos cidos
nucleicos teciduais. Em vez disso, so geralmente convertidas em cido rico pelas clulas da mucosa
intestinal. A maior parte do cido rico entra no sangue, sendo, aps, excretado pela urina.

- Remoo de grupo amino do AMP para


produzir IMP pela AMP-desaminase, ou da
adenosina para produzir inosina (hipoxantina-
ribose) pela adenosina-desaminase.
- IMP e GMP so convertidos em suas formas
nucleosdicas (inosina e guanosina) pela 5-
nucleotidase.

- A purina-nucleosdeo fosforilase
(nucleosidase) converte inosina e guanosina em
suas respectivas bases pricas (hipoxantina e
guanina). ** uma mutase interconverte ribose-1-
fosfato e ribose-5-fosfato.
- A guanina desaminada, formando xantina.
- A hipoxantina sucessivamente oxidada a
xantina e a cido rico pela xantina-oxidase,
flavoenzima com 1 tomo de molibdnio e 4
centros de ferro-enxofre em seu grupo prosttico.
O oxignio molecular o aceptor de eltrons
nessa reao.
- O cido rico o produto final de excreo do
catabolismo das purinas. Um adulto humano
saudvel excreta cido rico em uma taxa de
cerca de 0,6 g/24 h; o produto excretado origina-
se, em parte, das purinas ingeridas e, em parte, da
renovao dos nucleotdeos pricos dos cidos
nucleicos.
-
GOTA
- Doena caracterizada por altos nveis de cido rico no sangue (hiperuricemia), como resultado da
superproduo ou de sua baixa excreo.
- Pode resultar na deposio de cristais de urato monossdico nas articulaes e em uma resposta
inflamatria aos cristais, variando de um quadro agudo artrite gotosa crnica.
- Massas nodulares de cristais so depositadas nos tecidos moles do corpo (gota tofcea crnica).
- A formao de pedras de cido rico nos rins (urolitase) tambm pode ocorrer.
* A hiperuricemia assintomtica e no leva gota necessariamente, mas a gota precedida pela
hiperuricemia.
- Diagnstico (artrocentese): coleta por aspirao e anlise do fluido sinovial de uma articulao
afetada (ou material de um tofo), para a confirmao da presena de cristais de urato monossdico.
- Baixa excreo de cido rico:
- Principal causa de hiperuricemia que leva gota.
- Primria: problemas excretrios hereditrios ainda no identificados.
- Secundria a processos patolgicos conhecidos: afetam a maneira como os rins processam
urato. Ex.: acidose lctica (lactato e urato competem pelo mesmo transportador renal).
- Fatores ambientais: uso de frmacos (Ex.: diurticos tiazdicos, uso crnico. O uso agudo
aumenta a excreo. Agem aumentando a reabsoro e reduzindo a secreo), exposio ao chumbo (gota
saturnina)
- Superproduo de cido rico:
- Causa menos comum.
- Identificadas vrias mutaes no gene da PRPP-sintetase, ligado ao cromossomo X,
resultando em: Vmx aumentada para a produo de PRPP, Km menor para a ribose-5-fosfato, ou uma
sensibilidade diminuda aos nucleotdeos pricos seus inibidores alostricos.
- Assim, a maior disponibilidade de PRPP aumenta a produo de purinas, resultando em nveis
elevados de cido rico plasmtico.
- Tratamento para gota:
- Ataques agudos: anti-inflamatrios frmacos esteroides (prednisona), no esteroides
(indometacina), colchinina (despolimeriza os microtbulos, diminuindo o movimento de neutrfilos dentro
da rea afetada).
- A longo prazo: diminuio dos nveis de cido rico abaixo do ponto de saturao,
prevenindo a deposio dos cristais de urato.
- Agentes uricosricos (que aumentam a excreo renal; probenecida, sulfimpirazona) so
usados em pacientes que excretam baixas quantidades de cido rico.
- Alopurinol: anlogo estrutural da hipoxantina, inibe a sntese de cido rico um
substrato da xantina-oxidase que o converte em oxipurinol (aloxantina). O oxipurinol inativa a forma
reduzida da enzima permanecendo fortemente ligado ao seu stio ativo. Com a inibio da enzima, os
produtos de agora (xantina e hipoxantina) so mais hidrossolveis que o cido rico e apresentam menor
probabilidade de formar depsitos de cristais.
- Febuxostat: inibidor no prico da xantina-oxidase.
**
PROBLEMA 6
CORPOS CETNICOS
- A mitocndria heptica tem a capacidade de converter acetil-CoA proveniente da -oxidao de
cidos graxos em corpos cetnicos. Os compostos classificados como corpos cetnicos so o acetoacetato, o
3-hidroxibutirato (tambm chamado de -hidroxibutirato) e a acetona (produto colateral no metabolizvel).
- O acetoacetato e o 3-hidroxibutirato so transportados pelo sangue aos tecidos perifricos, onde
podem ser novamente convertidos em acetil-CoA, que oxidado no ciclo de Krebs.
- Importncia:
- So solveis em meio aquoso, ou seja, no precisam ser incorporados ou transportados por
lipoprotenas ou albumina.
- Produzidos no fgado, quando h alta de acetil-CoA.
- So usados pelos tecidos extra-hepticos (msculos, encfalo, rins) em quantidade
proporcional a sua concentrao sangunea, podendo permitir economia de glicose (importante no jejum
prolongado).
- A produo e exportao dos corpos cetnicos do fgado para tecidos extra-hepticos permite a
oxidao contnua de cidos graxos no fgado quando acetil-CoA no est sendo oxidada no ciclo de Krebs.
CETOGNESE
- No jejum, o fgado inundado com cidos graxos
mobilizados do tecido adiposo. Como resultado, eleva-
se a produo de acetil-CoA heptica, principalmente
pela degradao de cidos graxos.
- Acetil-CoA inibe a piruvato-desidrogenase e ativa
a piruvato-carboxilase. O oxaloacetato produzido
usado pelo fgado na gliconeognese, mais do que no
ciclo de Krebs. Desse modo, a acetil-CoA canalizada
para a sntese de corpos cetnicos.
- Ainda, a oxidao de cidos graxos diminui a razo
NAD+/NADH, e o aumento de NADH favorece a
converso de OAA em malato. Isto dirige a acetil-CoA
para a cetognese.
- A primeira etapa na formao de acetoacetato,
que ocorre no fgado, a condensao enzimtica de
2 molculas de acetil-CoA, catalisada pela tiolase
(inverso da ltima etapa da -oxidao).
- A HMG-CoA-sintase mitocondrial combina uma
terceira molcula de acetil-CoA com acetoacetil-CoA
para produzir HMG-CoA. (HMG-CoA-sintase citoslica
responsvel pela sntese de colesterol).
- A reao da HMG-CoA-sintase a etapa limitante
na sntese de corpos cetnicos e esta enzima ocorre
em quantidades significantes somente no fgado.
- O HMG-CoA clivado para produzir acetoacetato
e acetil-CoA. O acetoacetato pode ser reduzido para
formar 3-hidroxibutirato, utilizando NADH como
doador de hidrognio. O acetoacetato pode tambm
sofrer descarboxilao espontnea no sangue,
formando acetona (no metabolizado biologicamente,
que pode ser liberado na respirao).
- Equilbrio acetoacetato/3-hidroxibutirato: razo
NAD+/NADH. -oxidao consumo de NAD+
sntese de 3-hidroxibutirato favorecida.
CETLISE
- A sntese muito mais significativa durante o
jejum, quando os corpos cetnicos so necessrios para
produzir energia nos tecidos perifricos.
- O 3-hidroxibutirato oxidado a acetoacetato pela
3-hidroxibutirato-desidrogenase, produzindo NADH.
- O acetoacetato ativado ao seu ster de
coenzima A pela transferncia da CoA do succinil-CoA,
intermedirio do ciclo de Krebs, catalisada pela
-cetoacil-CoA-transferase (ou tioforase).
- Essa reao reversvel, mas o produto,
acetoacetil-CoA, ativamente removido por sua
converso em duas molculas de acetil-CoA. Assim, os
tecidos extra-hepticos oxidam eficientemente o
acetoacetato e o 3-hidroxibutirato.
- Assim, os corpos cetnicos so usados como
combustvel em todos os tecidos, exceto o fgado, que
carece de tioforase. O fgado , consequentemente, um
produtor de corpos cetnicos para os outros tecidos, mas
no um consumidor.
- A produo e exportao dos corpos cetnicos
pelo fgado permite a oxidao contnua de cidos graxos
com mnima oxidao de acetil-CoA.
- Quando os intermedirios do ciclo de Krebs so
desviados para a sntese de glicose pela gliconeognese, por exemplo, a oxidao dos intermedirios do ciclo
desacelera bem como a oxidao de acetil-CoA.
- Alm disso, o fgado contm apenas uma quantidade limitada de coenzima A, e quando a maior parte
est comprometida com acetil-CoA, a -oxidao desacelera esperando por coenzima livre. A produo e a
exportao de corpos cetnicos liberam a coenzima A, permitindo a contnua oxidao dos cidos graxos.
PRODUO EXCESSIVA NO DIABETES
- Jejum e diabetes melito no tratado leva
superproduo de corpos cetnicos, com vrios
problemas mdicos associados. Durante o jejum,
a gliconeognese consome os intermedirios do
ciclo de Krebs, desviando acetil-CoA para a
produo de corpos cetnicos.
- No diabetes no tratado, quando o nvel de
insulina insuficiente (tipo 1), os tecidos extra-
hepticos no podem captar a glicose do sangue
de maneira eficiente, para combustvel ou para
conservao como gordura.
- Nessas condies, os nveis de malonil-CoA
(incio da sntese de cidos graxos) caem, a
inibio da carnitina-aciltransferase 1 aliviada, e
os cidos graxos entram na mitocndria para ser
degradado a acetil-CoA que no pode passar
pelo ciclo do cido ctrico, j que os intermedirios
do ciclo foram drenados para uso como substrato
na gliconeognese. O acmulo resultante de acetil-CoA acelera a formao de corpos cetnicos alm da
capacidade de oxidao dos tecidos extra-hepticos.
- acetoacetato e 3-hidroxibutirato pH sangue acidose (pode levar ao coma e morte).
- Diabticos com cetose grave: concentrao sangunea de 90mg/mL (normal < 0,03mg/mL) e excreo
urinria de 5000mg/24h (normal 125mg/24h). ** Dietas hipocalricas tambm podem causar cetoacidose.
INSULINA
- A insulina um hormnio polipeptdico produzido pelas clulas das ilhotas de Langerhans - grupos
de clulas endcrinas, incrustados na poro excrina do pncreas. Seus efeitos metablicos so anablicos,
favorecendo, por exemplo, a sntese de glicognio, de TAGs e de protenas.
SNTESE

- Envolve dois precursores inativos, a pr-pr-insulina e a pr-insulina, que so clivados


sequencialmente para formar o hormnio ativo mais o peptdeo C, essencial para a organizao correta da
molcula de insulina, alm de ser um bom indicador da produo e secreo dela.
- A insulina estocada em grnulos no citosol, que, com o estmulo apropriado, so liberados por
exocitose. degradada pela enzima insulinase, presente no fgado e, em menor quantidade, nos rins. Possui
uma meia-vida plasmtica de aproximadamente seis minutos, permitindo alteraes rpidas nos nveis
circulantes.
REGULAO DA SECREO
- Estmulo: As quantidades relativas de insulina e glucagon liberadas pelo pncreas so reguladas de
modo que a velocidade de produo heptica da glicose mantida igual velocidade de utilizao da glicose
pelos tecidos perifricos.
- Glicose: estmulo mais importante para secreo. Favorece a expresso do gene da insulina.
- glicose GLUT2 carrega a
glicose para clula , que sofre
fosforilao e entra na gliclise.
- ATP fechamento do canal
de K+ controlado por ATP.
- efluxo K+ despolarizao
da membrana.
- Despolarizao abertura do
canal de Ca2+ controlado por
voltagem.
- Ca2+ citoslico + Ca2+
liberado pelo retculo
endoplasmtico liberao de
insulina por exocitose.
** Sistema n. parassimptico
estimula liberao de insulina.
- Aminocidos: Ingesto de protenas transitrio de AAs no plasma induzem secreo imediata
de insulina.
- Hormnios gastrointestinais: colecistocinina e polipeptdio inibitrio gstrico aumentam a secreo
em resposta glicose oral (incretinas). Liberados pelo intestino delgado aps a ingesto dos alimentos,
causando aumento antecipatrio da insulina. Assim, uma mesma quantidade de glicose desencadeia um nvel
de insulina circulante maior por via oral, em comparao por via intravenosa.
- Inibio: Diminuio por escassez de combustveis da dieta e tambm durante perodos de estresse.
- Mediados principalmente pela adrenalina, que secretada pela medula adrenal em resposta
ao estresse, ao trauma ou ao exerccio intenso. Causa uma mobilizao rpida de combustveis produtores de
energia, incluindo a glicose heptica (produzida pela glicogenlise ou pela gliconeognese) e os cidos graxos
provenientes do tecido adiposo. Alm disso, impede a liberao normal de insulina mediada por glicose, sendo
um controlador da secreo pelas clulas .
- Retroalimentao: insulina glicose sangunea fluxo da reao da glicocinase
ATP ...... reduo ou interrupo da liberao da insulina. Essa regulao mantm a concentrao da
glicose sangunea praticamente constante apesar da grande variao na captao diettica.
EFEITOS METABLICOS

- Os seus efeitos promovem o armazenamento da glicose e so mais proeminentes em 3 tecidos:


fgado, msculo e tecido adiposo.
- Carboidratos: Fgado e msculo -> sntese de glicognio
- Msculo e tec. adiposo -> captao de glicose ( do n de transportadores GLUT4)
- Fgado -> glicogenlise e gliconeognese
- Lipdios: reduo na liberao de cidos graxos
- Desfosforilao da lipase sensvel a hormnio (HSL) inativao degradao de TAGs/
cidos graxos livres circulantes.
- sntese de TAGs: glicerol-3-fosfato; lipase lipoproteica no tecido adiposo, aumentando
os cidos graxos para esterificao. Fgado: glicose TAGs (acetil-CoA cidos) VLDL tec. adiposo
- Sntese proteica: estimula a entrada de aminocidos nas clulas e a sntese de protenas (ativao de
fatores necessrios para a traduo).
- Efeitos na membrana: No msculo e tecido adiposo (sensveis insulina), promove o recrutamento
de GLUT4 (oriundos de vesculas intracelulares).
- Regulao: Ocorre internalizao do complexo hormnio-receptor. Dentro da clula, a insulina
degradada por lisossomos. Os receptores podem ser degradados, mas a maioria ressequestrada em
vesculas por endocitose.
- Curso temporal das aes: Resposta mais imediata: transporte de glicose para dentro dos
adipcitos e dos micitos, aps a ligao da insulina aos seus receptores de membrana. Aumenta a quantidade
de enzimas (glicocinase, piruvato-cinase heptica, acetil-CoA-carboxilase, cido-graxo sintase).
** Insensveis insulina: hepatcitos, eritrcitos, clulas do sistema nervoso, da mucosa intestinal, dos
tbulos renais e da crnea.
** A insulina, liberada em resposta alta concentrao sangunea de glicose, estimula a captao do
acar pelos tecidos. Parte da glicose exportada para o crebro para suas necessidades energticas e parte
para os tecidos adiposo e muscular.
** No fgado, o excesso de glicose oxidado a acetil-CoA, que usada na sntese de cidos graxos que
so exportados como triacilgliceris, em VLDL, para os tecidos adiposo e muscular. O NADPH necessrio para
a sntese de lipdeos obtido pela oxidao da glicose na via das pentoses-fosfato.
** O excesso de aminocidos convertido em piruvato e acetil-CoA, tambm usados para a sntese de
lipdeos.
** As gorduras da dieta se deslocam na forma de quilomicra, via sistema linftico, do intestino para o
msculo e o tecido adiposo.
DIABETES MELITO
- Grupo heterogneo de sndromes multifatoriais e polignicas, caracterizadas por hiperglicemia em
jejum, causada por deficincia relativa ou absoluta de insulina.
- Principal causa de cegueira e amputao no adulto e uma importante causa de falha renal, dano
neural, ataques cardacos e acidentes vasculares cerebrais.
- Dois grupos: tipo 1 (dependente de insulina) e tipo 2 (independente de insulina).
- A incidncia e a prevalncia do tipo 2 est aumentando devido ao aumento na prevalncia de
obesidade e do estilo de vida sedentrio.
DIABETES TIPO 1
- Caracterizada por deficincia absoluta de insulina, causada por ataque autoimune s clulas do
pncreas. No diabetes tipo 1, as ilhotas de Langerhans tornam-se infiltradas com linfcitos T ativados, levando
insulite.
- Esse ataque autoimune leva depleo gradual da populao de clulas , e os sintomas aparecem
abruptamente quando h destruio de cerca de 80% das clulas.
- Assim, o pncreas no consegue responder adequadamente ingesto de glicose, sendo necessria
a insulinoterapia para restaurar o controle metablico e prevenir a cetoacidose grave.
- A destruio das clulas requer um estmulo ambiental (infeco viral, por exemplo) e um
determinante gentico, que faz com que as clulas sejam reconhecidas como corpos estranhos.
- Em suma, no diabetes tipo 1 praticamente no h clulas e os portadores no conseguem responder
aos nveis de combustveis circulantes, nem manter uma secreo mnima de insulina.
- Diagnstico: Ocorre tipicamente durante a infncia ou a puberdade, e os sintomas se desenvolvem
rapidamente.
- Geralmente podem ser reconhecidos pelo aparecimento abrupto de:
- Poliria: o excesso de glicose no reabsorvido pelos rins. Como a glicose
osmoticamente ativa, sua excreo acompanhada por um aumento da perda de gua. ** desidratao
- Polidipsia: devido ao aumento da perda de gua.
- Polifagia: a no entrada de glicose em msculo e tecido adiposo sinaliza um estado de
baixa energia no organismo, fazendo com que o portador coma mais.
- Esses sintomas so acompanhados por fadiga, perda de peso e fraqueza.
- Glicemia em jejum maior ou igual a 126mg/dL, comumente acompanhada por cetoacidose.
(Jejum = ausncia de ingesto calrica por pelo menos 8 horas).
ALTERAES METABLICAS NO TIPO 1
- A captao da glicose do sangue mediada pela famlia GLUT de transportadores de glicose. Os
transportadores nos hepatcitos (GLUT1, GLUT2) e nos neurnios cerebrais (GLUT3) esto sempre presentes
nas membranas plasmticas.
- Por outro lado, o principal transportador de glicose nas clulas do msculo esqueltico, msculo
cardaco e tecido adiposo (GLUT4) est armazenado em pequenas vesculas intracelulares e se desloca para a
membrana plasmtica APENAS em resposta a um sinal de insulina.
- Assim, a incapacidade de liberar insulina suficiente para desencadear a captao de glicose pelas
clulas dos msculos esqueltico e cardaco e do tecido adiposo gera a hiperglicemia aps uma refeio.
- Normalmente, a insulina
desencadeia a insero
de transportadores
GLUT4 na membrana
plasmtica pela fuso de
vesculas contendo
GLUT4 com a membrana,
permitindo a captao de
glicose do sangue.
Quando os nveis de
insulina diminuem no
sangue, GLUT4
ressequestrado em
vesculas por endocitose.
No diabetes melito tipo 1
(dependente de insulina),
a insero de GLUT4 nas
membranas, assim como
outros processos
normalmente
estimulados por insulina,
esto inibidos.
- Hiperglicemia: Causada pela no captao perifrica da glicose pelos tecidos dependentes de insulina
associada a uma produo maior de glicose no fgado (gliconeognese).
- Incapazes de captar glicose, o msculo e o tecido adiposo utilizam os cidos graxos armazenados nos
triacilgliceris como seu principal combustvel.
- A degradao proteica gera aminocidos glicognicos, que so desaminados no fgado e tm seu
esqueleto de carbono remanescente incorporado aos intermedirios do Ciclo de Krebs, que sero desviados
para a gliconeognese.
- A mobilizao aumenta de cidos graxos do tecido adiposo combinada com sua -oxidao acelerada
no fgado gera grandes quantidades de acetil-CoA, que no entra no ciclo de Krebs devido utilizao do
oxaloacetato para a gliconeognese. Assim, o acmulo de acetil-CoA desviado para a sntese de corpos
cetnicos.
- Os compostos produzidos pela cetognese (3-hidroxibutirato e acetoacetato) so exportados e
levados a outros tecidos para serem utilizados como combustvel.
- Esses compostos so especialmente crticos para o crebro, que utiliza os corpos cetnicos como
combustvel alternativo quando glicose est indisponvel. (Os cidos graxos no conseguem atravessar a
barreira hematoenceflica e, por isso, no servem de combustvel para os neurnios do crebro).
- Em pacientes com diabetes tipo 1 no tratados, a superproduo de acetoacetato e 3-hidroxibutirato
leva a seu acmulo no sangue, e a consequente reduo do pH sanguneo leva cetoacidose, potencialmente
letal.
- Em casos de cetoacidose, os pacientes podem apresentar hiperventilao como forma de contrapor
o baixo pH do sangue.
- A cetoacidose diabtica tratada pela reposio de fluidos e eletrlitos, juntamente com a
administrao de doses baixas de insulina de ao rpida e de curta durao, para corrigir gradualmente a
hiperglicemia sem causar hipoglicemia.
- Hipertriacilglicerolemia: A mobilizao macia de cidos graxos do tecido adiposo sobrecarrega a -
oxidao e tambm a cetognese. Assim, o excesso de cidos graxos convertido em TAGs, que so
empacotados e secretados em VLDLs. Alm disso, devido a uma diminuio da sntese de lipase lipoproteica
(por conta do baixo nvel de insulina), ocorre aumento tanto de VLDL quanto de quilomcrons.
- Glicao: o excesso de glicose txico. Assim, pode ocorrer glicao de protenas do organismo
(colgeno, albumina, apoB), que afeta suas respectivas funes.
- Os produtos finais da glicao avanada se ligam a seus receptores de membrana nas clulas
endoteliais, gerando estresse oxidativo, lesando o endotlio e estimulando via pr-inflamatria.
- O resultado uma inflamao crnica de baixo grau, que vai lesar ainda mais o endotlio
vascular. Assim, a formao dos produtos finais um fator que contribui para o desenvolvimento de
complicaes microvasculares do diabetes. Contribui, do mesmo modo, para a aterognese.
- A hiperglicemia altera o estado redox da clula, ao aumentar a razo NADH/NAD+ e ao diminuir a
razo NADPH/NADP+. Essa alterao dirige os substratos para a via do poliol, onde a glicose reduzida a
sorbitol pela aldose-redutase.
- Essa enzima possui alto Km, assim, essa via ativada durante a hiperglicemia, quando a
concentrao de glicose nos tecidos independentes de insulina (eritrcitos, nervo, cristalino) aumenta.
- De forma similar glicose, o sorbitol osmoticamente ativo. Sua acumulao no tecido ocular
contribui para o desenvolvimento de cataratas diabticas.
- No tecido nervoso, altas concentraes de sorbitol diminuem a captao celular de outro
lcool, o mioinositol, inibindo a ATPase Na+/K+ de membrana e afetando assim a funo nervosa.
- A acumulao de sorbitol, a hipxia e o fluxo sanguneo reduzido no tecido nervoso
contribuem para o desenvolvimento da neuropatia diabtica.
TRATAMENTO
- Diabticos do tipo 1 dependem de insulina exgena injetada subcutaneamente para controlar a
hiperglicemia e a cetoacidose. H dois procedimentos (tratamento padro e intensivo).
- Tratamento padro: consiste tipicamente em uma ou duas injees dirias de insulina humana
recombinante. Nvel de glicose esperado (225-275 mg/dL) com hemoglobina glicada de 8 a 9%.
- Tratamento intensivo: monitoramento mais frequente e de subsequentes injees de insulina -
tipicamente trs ou mais vezes ao dia. Nvel de glicose esperado (150mg/dL) com hemoglobina glicada de 7%.
Apresentam reduo de 50% ou mais nas complicaes microvasculares a longo prazo do diabetes -
retinopatia, nefropatia e neuropatia - quando comparados com pacientes recebendo o tratamento padro.
- Hipoglicemia: a dosagem adequada de insulina difcil de alcanar. A hipoglicemia causada pelo
excesso de insulina a complicao mais comum dessa terapia, ocorrendo em mais de 90% dos pacientes.
- A frequncia de episdios hipoglicmicos, coma e convulses especialmente elevada nos
casos de tratamento intensivo, destinados a atingir um controle rgido da glicose sangunea.
- Indivduos normais -> secreo compensatria de adrenalina e glucagon. No tipo 1, h
deficincia na secreo do glucagon. Logo, os pacientes dependem da adrenalina para prevenir hipoglicemia
grave.
- Avanar da doena -> neuropatia diabtica autonmica -> diminuio da secreo da
adrenalina em resposta hipoglicemia -> hipoglicemia despercebida
- Tambm causada por exerccio fsico intenso.
DIABETES TIPO 2
- Os pacientes com diabetes tipo 2 apresentam uma combinao de resistncia insulina com
disfuno das clulas , mas no necessitam de insulina para manter a vida, embora a insulina possa ser
necessria em algum momento para controlar a hiperglicemia.
- As alteraes metablicas observadas no diabetes tipo 2 so mais brandas do que as descritas para o
tipo 1, em parte, porque a secreo da insulina no diabetes tipo 2 embora inadequada - impede a cetognese
excessiva e restringe o desenvolvimento da cetoacidose diabtica.
- Existem fatores genticos que claramente predispem para o diabetes tipo 2. Embora 80% das
pessoas com diabetes tipo 2 sejam obesas, a maioria das pessoas obesas no desenvolve a doena.
RESISTNCIA INSULINA
- Estado no qual necessria mais insulina para realizar os mesmos efeitos biolgicos produzidos no
estado sadio normal por uma quantidade mais baixa do hormnio. Por exemplo, a resistncia insulina
caracterizada por produo no controlada de glicose heptica e por captao diminuda de glicose pelo
msculo e pelo tecido adiposo.
- Resistncia e obesidade: causa mais comum de resistncia insulina; contudo, a maioria das pessoas
obesas e resistentes insulina no se torna diabtica. Na ausncia de um defeito na funo das clulas ,
indivduos obesos no diabticos podem compensar a resistncia insulina com nveis elevados (secreo 2,
3 vezes maior) desse hormnio.
- Resistncia e diabetes tipo 2: O diabetes tipo 2 se desenvolve em indivduos resistentes insulina
que tambm apresentam diminuio na funo das clulas . Em geral, a resistncia insulina com o risco
subsequente de desenvolvimento do diabetes tipo 2 comumente observada em idosos e em indivduos
obesos, sedentrios, ou nos 3 a 5% de mulheres grvidas que desenvolvem diabetes gestacional.
- O estgio intermedirio que precede o diabetes tipo 2 , s vezes, a sndrome metablica,
caracterizada por obesidade, especialmente no abdome; hipertenso (presso sangunea alta); lipdeos
sanguneos anormais (altos TAG e LDL, baixa HDL); glicose sangunea levemente elevada; e uma capacidade
reduzida de remover a glicose sangunea em um teste de tolerncia glicose.
- As pessoas com sndrome metablica apresentam, com frequncia, alteraes nas protenas
sanguneas, associadas com coagulao anormal (alta concentrao de fibrinognio) ou inflamao (alta
concentrao de peptdeo C reativo, a qual aumenta com a resposta inflamatria).
- Causas da resistncia: hiptese da toxicidade lipdica.
-> A ao do PPAR-gama sobre os adipcitos normalmente mantm as clulas preparadas para sintetizar e
armazenar TAGs os adipcitos so sensveis insulina e produzem leptina, o que leva deposio intracelular
contnua de TAG.
-> O excesso de ingesto calrica em pessoas obesas faz os adipcitos ficarem repletos de TAG, tornando o
tecido adiposo incapaz de receber uma demanda aumentada para estocar TAG.
-> O tecido adiposo repleto de lipdeos libera fatores proteicos que atraem macrfagos que se infiltram no
tecido.
-> Os macrfagos desencadeiam a resposta inflamatria, que prejudica a deposio dos cidos graxos nos
adipcitos e favorece sua liberao para o sangue.
-> Esse excesso de cidos graxos entra nas clulas hepticas e musculares, onde convertido em TAG que se
acumulam como gotculas lipdicas.
-> Essa deposio ectpica (fora de lugar) de TAG leva insensibilidade insulina no fgado e no msculo, a
caracterstica do diabetes tipo 2. Alm disso, de acordo com essa hiptese, cidos graxos e TAG em excesso
so txicos para o fgado e o msculo.

- Clulas disfuncionais: No diabetes tipo 2, a capacidade das clulas do pncreas inicialmente


mantida, resultando em nveis de insulina que variam desde acima at abaixo do normal.
- Entretanto, com o decorrer do tempo, as clulas tornam-se progressivamente disfuncionais,
sendo incapazes de secretar insulina suficiente para corrigir a hiperglicemia preponderante.
- Assim, a progresso natural da doena resulta no declnio da capacidade de controlar a
hiperglicemia com secreo endgena de insulina. A deteriorao da funo das clulas pode ser acelerada
pelos efeitos txicos da hiperglicemia persistente e pelo aumento dos cidos graxos livres.
ALTERAES METABLICAS NO TIPO 2

- So o resultado da resistncia insulina, que se expressa principalmente no fgado, no msculo e no


tecido adiposo.
- Hiperglicemia: causada por aumento na produo de glicose heptica, combinado com diminuio
na sua utilizao perifrica. Em geral, a cetose mnima ou ausente em pacientes com diabetes tipo 2, pois a
presena de insulina - mesmo na presena de resistncia insulina - diminui a cetognese heptica.
- Dislipidemia: No fgado, os cidos graxos so convertidos em triacilgliceris, que so empacotados e
secretados como VLDL. Como a degradao das lipoprotenas, catalisada pela lipase lipoproteica no tecido
adiposo, baixa nos diabticos, os nveis plasmticos de quilomcrons e de VLDL se elevam, resultando em
hipertriacilglicerolemia.
TRATAMENTO

- Estudos mostram que trs fatores aumentam a sade das pessoas com diabetes tipo 2: restrio
alimentar, exerccio regular e frmacos que aumentam a produo de insulina ou a sensibilidade ao hormnio.
- A restrio alimentar (e a perda de peso que a acompanha) reduz a carga total de cidos graxos
controlveis. O exerccio ativa a AMPK, como o faz a adiponectina; a AMPK altera o metabolismo no sentido
da oxidao da gordura e ao mesmo tempo inibe sua sntese.
- As sulfonilureias agem sobre os canais de K+ controlados por ATP em clulas pancreticas e
estimulam a liberao de insulina.
- Biguanidas como a metformina (Glicofage) ativam a AMPK, mimetizando os efeitos da adiponectina.
- As tiazolidinedionas agem por meio de PPAR-gama, aumentando a concentrao de adiponectina no
plasma e estimulando a diferenciao dos adipcitos, de forma a aumentar a capacidade de armazenamento
de TAG.
- Os inibidores de protease IV de dipeptdeos (DPP IV) previnem a degradao proteoltica do GLP-1,
hormnio peptdico produzido no intestino e que estimula a secreo pancretica de insulina. A inibio da
peptidase prolonga a ao do GLP-1, aumentando efetivamente a secreo da insulina.
- A combinao entre perda de peso e exerccio a forma recomendada para prevenir o
desenvolvimento da sndrome metablica e do diabetes tipo 2.
ASPECTOS PSICOSSOCIAIS
-Doentes crnicos, de um modo geral, principalmente os portadores de diabetes, constituem-se num
desafio para profissionais de sade que com eles lidam, pois a manuteno contnua do tratamento prescrito
e a obedincia consciente s condutas orientadas so de difcil aceitao e, principalmente, de incorporao
de novos hbitos que podero cercear suas vidas com base no diagnstico da doena.
-A adeso do diabtico ao tratamento pode ser mais complexa quando comparada com outras doenas
crnicas, pois o uso da insulina requer mais habilidade do que o simples uso de medicao via oral e pela
variedade de complicaes decorrentes da doena, tais como as vasculopatias e neuropatias que podem
acarretar o p diabtico e as amputaes, alm das mesmas complicaes de outras doenas crnicas, como
acidente vascular cerebral, doenas isqumicas do corao e comprometimento renal.
-Soma-se a isso a obrigatoriedade de realizar testes de glicemia capilar ou srica, assim como a
realizao de outros exames para controle do colesterol e triglicrides, exames radiolgicos e cardiolgicos e
avaliao com outros especialistas como ginecologista, urologista e oftalmologista. E ainda o cuidado rigoroso
com os ps, no intuito de prevenir leses que possam evoluir para amputaes parciais ou totais de um ou
mais membros.
-O grande desafio dos profissionais que cuidam de indivduos portadores de doenas crnicas a
manuteno contnua e a fiel obedincia s condutas prescritas, sejam de que ordem for. Isto se denomina
adeso ao tratamento, que no muito diferente em pases desenvolvidos ou subdesenvolvidos e no parece
estar relacionada a fatores, tais como conhecimento do paciente sobre a patologia ou aquisio ou no de
medicaes gratuitamente.
PROBLEMA 7
VITAMINAS
- Compostos orgnicos no relacionados quimicamente, que no podem ser sintetizados por humanos
em quantidades adequadas e, portanto, devem ser supridos pela dieta.
- So classificadas de acordo com sua solubilidade e suas funes no metabolismo.
- Nove vitaminas hidrossolveis (cido flico, cobalamina, cido ascrbico, piridoxina, tiamina, niacina,
riboflavina, biotina e cido pantotnico e quatro vitaminas lipossolveis (A, D, E, K).
- So necessrias para a execuo de funes celulares especficas (ex.: precursoras de coenzimas para
as enzimas do metabolismo intermedirio).
- As vitaminas lipossolveis so liberadas, absorvidas e transportadas com a gordura da dieta. Elas no
so facilmente excretadas na urina, e quantidades significantes so armazenadas no fgado e no tecido
adiposo. O consumo excessivo pode acarretar acmulo de quantidades txicas desses compostos.
HIDROSSOLVEIS
- Muitas so precursoras de coenzimas para o metabolismo energtico.
- No so txicas (exceto piridoxina) e o organismo no possui capacidade para armazen-las (exceto
B12). Assim, devem ser regularmente fornecidas pela dieta.
- Qualquer excesso facilmente excretado na urina.
CIDO FLICO
- A di-hidrofolato-redutase converte folato em tetrahidrofolato, a forma biologicamente ativa.
- Funo: O tetrahidrofolato (folato reduzido) recebe fragmentos de um carbono de doadores (serina,
glicina, histidina) e os transfere para intermedirios na sntese de AAs, purinas e monofosfato de timidina
(TMP), pirimidina encontrada no DNA.
- O cido flico encontrado no fgado, levedura e em vegetais folhosos verdes (espinafre) e frutas,
incluindo citrinos. Suas fontes so tambm cereais e gros enriquecidos em cido flico.
- Anemias nutricionais:
- O sangue apresenta concentrao de hemoglobina abaixo do normal, o que resulta na reduo
da sua capacidade de transportar oxignio.
- Anemia nutricional (ingesto inadequada de um ou mais nutrientes essenciais) podem ser
classificadas de acordo com o tamanho dos eritrcitos ou com o volume corpuscular mdio observado (VCM):
- Microcticas (VCM < 80 m): Deficincia em ferro, cobre ou piridoxina. A causa pelo
ferro a forma mais comum de anemia nutricional.
- Normocticas (VCM = 80 100 m): Desnutrio calrico-proteica.
- Macrocticas (VCM > 80 m): Deficincia em B12 ou folato. a segunda principal
categoria de anemia nutricional.
- So chamadas de megaloblsticas pois a deficincia causa o acmulo de
precursores grandes e imaturos do eritrcito (megaloblastos) na medula ssea e sangue.
- A falha na sntese de metionina e cidos nucleicos nos estados de deficincia
de folato responsvel pelos sinais e sintomas da anemia megaloblstica.
- Os eritrcitos macrocticos tm membranas frgeis e tendncia a hemolisarem:
existe anemia macroctica em associao a uma medula ssea megaloblstica.
- Folato e anemia: nveis sricos inadequados podem ser causados por:
- Aumento da demanda: Gestao e lactao.
- Absoro deficiente: patologias associadas ao intestino delgado, alcoolismo ou frmacos
inibidores da di-hidrofolato-redutase, como o metotrexato (anlogo do folato, atua como inibidor
competitivo, ligao com a enzima com afinidade cerca de 100x maior que o folato. Usado como
quimioterpico). ** antagonistas: toxicidade seletiva contra clulas em crescimento rpido.
** Outro antagonista do folato pode atuar como antibitico: trimetoprima (liga-se di-
hidrofolato-redutase com eficincia cerca de 100.000 vezes maior do que se liga enzima de mamferos, sendo
usada no tratamento de certas infeces bacterianas urinrias e do ouvido mdio.
- Uma dieta sem folato pode causar deficincia em poucas semanas. O principal resultado a
anemia megaloblstica ( sntese de purinas e TMP incapacidade da clula, incluindo precursores de
eritrcitos, de produzir DNA problemas na diviso celular).
** As alteraes hematolgicas no podem ser distinguidas da deficincia de vitamina B12. As
alteraes neurolgicas tambm so similares. Assim, a causa deve ser investigada antes de comear a terapia.
- Folato e defeitos do tubo neural em fetos:
- A suplementao de cido flico antes da concepo e durante o 1 trimestre (0,4mg/dia)
diminui o risco de espinha bfida e anencefalia, que so os defeitos mais comuns do tubo neural.
- A suplementao nos perodos iniciais da gestao crucial, j que o desenvolvimento crtico
dependente de folato ocorre nas primeiras semanas do desenvolvimento fetal - perodo em que muitas
mulheres ainda no tm conscincia da sua gravidez.
- Existe uma associao entre suplementao com altas doses de cido flico (> 0,8 mg/dia) e o
aumento no risco de cncer. Portanto, a suplementao no recomendada para a maioria dos adultos de
meia idade e idosos.

COBALAMINA (B12)
- Necessria em duas reaes: remetilao da homocistena em metionina (metionina sintetase) e
isomerizao do metilmalonil-CoA, que produzido durante a degradao de alguns AAs (isoleucina, valina,
treonina e metionina) e cidos graxos com nmero mpar de carbonos (propionil-CoA carboxilao D-
metilmalonil-CoA epimerase L-metilmalonil-CoA mutase succinil-CoA).
- Deficincia gera acmulo de cidos graxos incomuns nas membranas celulares, incluindo as do
sistema nervoso, contribuindo para as manifestaes neurolgicas.
- Em deficincia de B12, o propionato e o metilmalonato so excretados na urina. Ausncia ou
deficincia da mutase (ou menor afinidade pela coenzima) ou incapacidade de converter B12 na coenzima
tambm resultam em acidose metablica.
- Estrutura: Tem uma estrutura complexa em anel similar porfirina do grupo heme, mas mais
hidrogenada. O ferro localizado no centro do anel do grupo heme substitudo por um on cobalto (Co3+),
mantido por 4 ligaes coordenadas com os nitrognios dos grupos pirrol.
- Em preparaes comerciais da vitamina, as demais ligaes coordenadas do cobalto so com
o nitrognio do 5,6-dimetilbenzimidazol e com o cianeto na forma de cianocobalamina.
- As formas de coenzima da cobalamina so 5'-desoxiadenosilcobalamina, em que o cianeto
substitudo pela 5'-desoxiadenosina (formando uma ligao no usual carbono-cobalto), e a metilcobalamina,
em que o cianeto substitudo por um grupo metila.

- Distribuio: Sintetizada apenas por microrganismos (ausente em todos os vegetais). Os animais


obtm a vitamina pr-formada pela sua flora bacteriana natural ou por alimentos derivados de animais.
- Est presente em quantidades apreciveis no fgado, no leite integral, em ovos, ostras,
camares frescos e nas carnes de porco e de galinha.
- Hiptese da captura do folato: os efeitos da deficincia so mais pronunciados em clulas que se
dividem rapidamente (tecido eritropoitico da medula ssea e as clulas da mucosa intestinal).

- A vitamina B12 e o folato esto envolvidos na converso da homocistena em metionina. A


ausncia de vitamina B12 inibe a reao e leva ao acmulo de N5-metiltetraidrofolato (N5MeTHF).
- Os nveis das outras formas diminuem. Assim, levanta-se a hiptese de que a deficincia de
cobalamina conduz deficincia das formas de tetra-hidrofolato necessrias para a sntese de purinas e de
TMP, resultando nos sintomas de anemia megaloblstica.
- Indicaes clnicas: Como exceo, quantidades significativas (4 a 5 mg) de B12 so armazenadas no
organismo. Assim, podem ser necessrios vrios anos para o desenvolvimento de sintomas clnicos de
deficincia de B12 nos casos de gastrectomia total ou parcial, 10-12 anos em dietas veganas, 1-4 anos em
falhas do fator intrnseco, entretanto, pode ser rpido nos casos de disfuno do leo.
- Absoro: A cobalamina liberada dos alimentos por uma protease gstrica e pelo HCl. Est ligada
ao fator intrnseco secretado pelas clulas parietais no estmago, e absorvida no leo distal por endocitose
mediada por receptores. A vitamina B12 excretada na bile, e existe acentuada circulao entero-heptica.

- TC II:
protena de
transporte >
transcobalami-
na

- Anemia perniciosa: A deficincia de B12 est mais relacionada com falhas na absoro da vitamina
pelo intestino. A m absoro de cobalamina em idosos frequentemente devida reduzida secreo de cido
gstrico e menor absoro da vitamina B12 da dieta, resultando em anemia perniciosa. Na maioria dos casos,
essa doena resulta da destruio autoimune das clulas gstricas parietais, que so responsveis pela sntese
do fator intrnseco.
- A deficincia de vitamina B12 caracterizada por anemia, fadiga, constipao, perda de peso,
diarreia e sintomas neurolgicos, como dormncia e sensao de formigueiro, perda de equilbrio, confuso,
perturbaes de humor e demncia. Os efeitos sobre o SNC so irreversveis.
- Tratada pela administrao oral de altas doses de B12 ou injeo intramuscular (IM) de
cianocobalamina. A terapia deve continuar por toda a vida em pacientes com anemia perniciosa.
** cido flico reverte as alteraes hematolgicas causadas pela falta de B12, mascarando a deficincia.
Assim, terapia de anemia megaloblstica feita associando as duas vitaminas at a determinao da causa.
CIDO ASCRBICO (VITAMINA C)
- A forma ativa da vitamina C o cido ascrbico. A principal funo do ascorbato a de agente redutor
em diversas reaes diferentes.
- Atua como coenzima nas reaes de hidroxilao (ex.: hidroxilao dos resduos prolil- e lisil- do
colgeno, sendo assim necessria para a manuteno do tecido conjuntivo normal e para recompor tecidos
danificados).
- Melhora a absoro de ferro no heme e participa do metabolismo mineral sseo. Sua principal
funo manter cofatores metlicos em seus menores estados de valncia, como, por exemplo, Fe2+ e Cu2+.
- Participa da sntese de adrenalina, da esteroidognese (sntese de hormnios esteroides), da
degradao da tirosina, da formao de cidos biliares, assim como da sntese de l-carnitina e de
neurotransmissores.

- A vitamina C absorvida no intestino por transporte mediado por um carreador e dependente de


sdio (ativo). reabsorvida nos tbulos proximais renais. Progressivamente, mais vitamina C excretada na
urina medida que sua ingesto aumenta.
- A vitamina C lbil: facilmente destruda por oxignio, ons metlicos, pH aumentado, calor e luz.
Os citrinos, frutos de bagas, tomates e pimentes so fontes ricas em vitamina C.
- Deficincia: A deficincia de vitamina C causa sntese defeituosa de colgeno. O escorbuto
caracterizado por fragilidade capilar, causando hemorragias subcutneas e outras, fraqueza muscular,
gengivas moles, inchadas e sangrentas, queda de dentes, dificuldade na cicatrizao de ferimentos e anemia.
- Ocorre fadiga, mal-estar e depresso. A incapacidade de manter a matriz ssea, em associao
desmineralizao, resulta em osteoporose.
- A vitamina C necessria para a funo leucocitria normal, e os nveis de vitamina C dos
leuccitos caem abruptamente durante o estresse causado por trauma ou infeco.
- O consumo de dietas ricas nesses compostos est associado diminuio na incidncia de algumas
doenas crnicas, como doena cardaca coronariana e alguns tipos de cncer. No entanto, testes clnicos
envolvendo a suplementao com antioxidantes isolados tm falhado na determinao de qualquer efeito
benfico convincente.
** Linus Pauling defendia o consumo de altas doses (ele consumia 18g, recomendao diria
de 60mg) na preveno de resfriados e cncer. Morreu de cncer de prstata e sua mulher, de cncer de
estmago. Ele alegava que o consumo de vitamina C atrasou a doena em aproximadamente 20 anos, j os
mdicos achavam que tanto seu cncer quanto o de sua mulher foram causados pelo consumo excessivo.
PIRIDOXINA (VITAMINA B6)
- Difere no grupo funcional ligado ao anel (piridoxina, piridoxal, piridoxamina, todos derivados da
piridina). A piridoxina ocorre principalmente nas plantas, ao passo que o piridoxal e a piridoxamina so
encontrados em alimentos de origem animal.
- Todos os trs compostos podem servir como precursores da coenzima biologicamente ativa, o
piridoxal-fosfato (PLP).
- Funciona como uma coenzima para um grande nmero de enzimas catalisam reaes envolvendo
aminocidos transaminao e desaminao.
- O PLP funciona como carreador intermedirio de grupos amino,
no stio ativo das aminotransferases. Ele sofre transformaes
reversveis entre sua forma aldedica, o piridoxal-fosfato, que pode
aceitar um grupo amino, e sua forma aminada, a piridoxamina-
fosfato, que pode doar seu grupo amino para um -cetocido.
- Geralmente, o piridoxal-fosfato encontra-se ligado
covalentemente ao stio ativo da enzima por meio de uma ligao
aldimina (base de Schiff) com o grupo amino de um resduo
especfico de lisina.
- absorvida no jejuno. Devido ao seu papel no metabolismo de
AAs, as necessidades de vitamina B6 aumentam com o aumento da
ingesto de protenas. Est presente em uma grande variedade de
alimentos, como peixe, carne bovina, fgado, aves, bem como em
batatas e frutas (mas no citrinos).
- Indicaes clnicas: A isoniazida, um frmaco frequentemente
usado para o tratamento da tuberculose, pode induzir deficincia de
vitamina B6 pela formao de um derivado inativo com o piridoxal-
fosfato. A suplementao de B6 na dieta , assim, um adjuvante para
o tratamento com isoniazida.
- A plula contraceptiva, por aumentar a sntese de enzimas
que requerem a vitamina, pode precipitar a deficincia.
- Observam-se nveis baixos no alcoolismo, obesidade e
em estados de m absoro, assim como na doena renal terminal
e em condies autoimunes.
- Deficincia: Em sua forma mais branda, causa irritabilidade, nervosismo e depresso, progredindo
para neuropatia perifrica, convulses e coma na deficincia grave.
- A deficincia grave est associada tambm anemia sideroblstica (anemia caracterizada pela
presena de eritrcitos nucleados com grnulos de ferro). Ocorrem tambm dermatite, queilose e glossite.
- A avaliao dos nveis de piridoxina baseada na medio dos nveis da enzima aspartato
aminotransferase (AST, TGO) eritrocitria
- Toxicidade: nica vitamina hidrossolvel com toxicidade significante. Ingesto 100x maior que a
recomendao diria pode causar sintomas neurolgicos.
TIAMINA (VITAMINA B1)
- O pirofosfato de tiamina a forma biologicamente ativa dessa vitamina, formada pela transferncia
do grupo pirofosfato do trifosfato de adenosina (ATP) para a tiamina.
- Na sua forma ativa, essa vitamina essencial como coenzima para o funcionamento timo da
piruvato-desidrogenase (piruvato -> acetil-CoA), da transcetolase (transferncia de fragmento de 2 carbonos
de uma cetose para uma aldose) e da -cetoglutarato-desidrogenase (-cetoglutarato -> succinil-CoA) em
reaes do metabolismo dos carboidratos pelas vias da pentose fosfato, da gliclise, do ciclo do cido ctrico,
formando NADH e ATP.

- A descarboxilao de piruvato e -cetoglutarato so especialmente importantes em tecidos do


sistema nervoso. Deficincia de tiamina resulta em reduo da produo de ATP, diminuindo a funo celular.
- Deficincia: Diagnosticada pelo aumento na atividade da transcetolase em eritrcitos, observado
aps a adio de pirofosfato de tiamina.
- Beribri:
- Os sinais de beribri em bebs incluem taquicardia, vmito, convulses e, se no for tratada,
morte. A sndrome da deficincia pode ter rpido incio em lactentes cujas mes so deficientes em tiamina.
- O beribri em adultos caracterizado por pele seca, irritabilidade, pensamento desordenado
e paralisia progressiva.
- Sndrome de Wernicke-Korsakoff: Observada em associao ao alcoolismo crnico, devida
insuficincia diettica ou diminuio na absoro intestinal da vitamina.
- Os sintomas iniciais da deficincia de tiamina so perda de apetite, obstipao e nuseas. Eles
podem progredir para depresso, neuropatia perifrica e tremores. A deteriorao adicional resulta em
confuso mental (perda da memria recente), ataxia e perda da coordenao ocular. Esta combinao,
frequentemente observada em pacientes alcolatras, a psicose de Wernicke-Korsakoff.
- As consequncias neurolgicas da sndrome de Wernicke so tratadas com suplementao de
tiamina.
NIACINA (VITAMINA B3)
- Niacina um nome genrico para cido nicotnico ou nicotinamida, e ambos so nutrientes essenciais.
- As formas biologicamente ativas da coenzima so nicotinamida-adenina-dinucleotdeo (NAD+) e seu
derivado fosforilado, nicotinamida-adenina-dinucleotdeo-fosfato (NADP+).
- A niacina pode ser sintetizada a partir do triptofano e, portanto, no uma vitamina no verdadeiro
sentido da palavra. A converso , contudo, muito ineficiente e no consegue suprir quantidades suficientes
de niacina.
- O NAD+ e o NADP+ servem como coenzimas em reaes de oxidao-reduo em que a coenzima
sofre reduo do anel piridina, pela incorporao de um on hidreto (tomo de hidrognio mais um eltron),
formando assim as formas reduzidas, NADH e NADPH.
- Assim, so de suma importncia como transportadores de eltrons para a cadeia transportadora
(transferncia do complexo I para ubiquitina e depois aos outros complexos) e fosforilao oxidativa
(responsveis pelo gradiente de prtons para o espao intermembrana, necessrio para a sntese de ATP).
- Distribuio: encontrada em gros enriquecidos e no refinados, em cereais, no leite e em carnes
magras, especialmente fgado.
- Deficincia: produz inicialmente glossite superficial, mas pode progredir para pelagra, que
caracterizada por dermatite (leses de pele semelhantes a queimaduras solares nas reas corporais expostas
ao sol e presso) assim como diarreia e demncia. A pelagra fatal, se no tratada.
- Tratamento de hiperlipidemia: Altas doses (1,5g/dia, 100x que a quantidade diria) inibem
fortemente a liplise. Assim, h diminuio heptica de TAGs, necessrios para a produo de VLDLs. Logo,
tanto VLDL quanto LDL so diminudos no plasma. ** Tambm aumenta os nveis de HDL.
RIBOFLAVINA (VITAMINA B2)
- As duas formas biologicamente ativas so flavina mononucleotdeo (FMN) e flavina adenina
dinucleotdeo (FAD), formadas pela transferncia da poro monofosfato de adenosina do ATP para o FMN.
- A FMN e a FAD so capazes de aceitar reversivelmente dois tomos de hidrognio, formando FMNH2
ou FADH2. A FMN e a FAD encontram-se fortemente ligadas algumas vezes covalentemente - a flavoenzimas
que catalisam a oxidao ou a reduo de um substrato.
- Tambm importante para a transferncia de eltrons para a cadeia transportadora (a partir do
complexo II). ** Complexo II no gera gradiente de prtons.
- facilmente degradada pela luz UV do sol.
- A falta de riboflavina na dieta causa uma sndrome de deficincia com inflamao dos cantos da boca
(estomatite angular), da lngua (glossite) e dermatite de descamao.
- Devido sua sensibilidade luz, a deficincia de riboflavina pode ocorrer em recm-nascidos com
ictercia, os quais so tratados com fototerapia.
- Sabe-se que o hipotireoidismo tambm afeta a converso de riboflavina em FMN e FAD.
- Para determinar os nveis de riboflavina, medida a atividade da glutationa redutase eritrocitria.
BIOTINA (VITAMINA B7)
- A biotina uma coenzima nas reaes de carboxilao, em que serve como carreadora do dixido de
carbono ativado.
- A biotina liga-se covalentemente ao grupo amino de resduos de lisina nas enzimas dependentes de
biotina.
- No stio cataltico 1, o on bicarbonato convertido a CO2 com gasto de ATP. Em seguida, o CO2 reage
com a biotina, formando carboxibiotinil-enzima. O brao longo composto pela biotina e a cadeia lateral da Lys
transporta o CO2 da carboxibiotinil-enzima para o stio cataltico 2 na superfcie da enzima, onde o CO2
liberado, regenerando o complexo biotinil-enzima. A clivagem de um fosfato de alta energia do ATP impulsiona
a formao do intermedirio enzima-biotina-CO2.

- A biotina normalmente sintetizada pela flora intestinal, e isso satisfaz grande parte das necessidades
do organismo. Alm disso, est amplamente distribuda nos alimentos.
- Os sintomas da deficincia de biotina incluem depresso, alucinaes, dor muscular e dermatite.
Crianas com mltiplas deficincias das enzimas descarboxilases tambm desenvolvem imunodeficincia.
- O consumo de claras de ovos crus pode causar deficincia de biotina, uma vez que a protena da clara
do ovo, a avidina, se liga fortemente com a biotina, impedindo sua absoro pelo intestino.

CIDO PANTOTNICO (VITAMINA B5)


- Componente da coenzima A (CoA) que atua na transferncia de grupos acila.
- A CoA contm um grupo tiol que transporta os compostos acila, como steres de tiol ativados.
- Tambm um componente de um domnio protena carreadora de acila (PCA) da sintetase dos cidos
graxos.
- Os ovos, o fgado e as leveduras so as fontes mais importantes
de cido pantotnico, embora esta vitamina seja amplamente
distribuda. No h evidncia de deficincia no homem, exceto em
dietas experimentais.
SUPLEMENTAO VITAMNICA
- Gestantes: As gestantes tm benefcio comprovado na suplementao de cido flico e ferro, que
diminuem o risco de m formao fetal e de anemia, respectivamente.
- Pessoas com mais de 60 anos: No pela idade em si, mas pela prevalncia maior de doenas e de uso
de medicaes que afetam a absoro vitamnica e menor exposio ao sol, pessoas nessa faixa etria podem
se beneficiar de uma investigao quanto a carncia de vitamina B12 e vitamina D.
- Pacientes com problemas gastrointestinais e praticantes de dietas restritivas: Pessoas que sofrem de
doenas intestinais, que passaram por cirurgias para tratar a obesidade ou que sofrem de problemas
nutricionais devido a hbitos de vida, como o etilismo ou dietas muito restritivas merecem uma ateno
especial e podem ter carncia de diversos minerais e vitaminas.
- Quem tem problemas no fgado: Pessoas que sofrem de hepatite gordurosa (a inflamao causada
pela gordura no fgado) podem se beneficiar da suplementao de vitamina E.
- Vegetarianos: pela carncia de vitamina B12 em suas dietas e pelo risco de apresentarem deficincia
de riboflavina, clcio, ferro e dos aminocidos essenciais lisina e metionina no organismo.

ROTULAGEM DE SUPLEMENTOS VITAMNICOS/MINERAIS


2. DESCRIO **Portaria n 32, de 13 de janeiro de 1998
2.1. Definio
Suplementos Vitamnicos e ou de Minerais para fins deste regulamento, doravante denominados
simplesmente de "suplementos", so alimentos que servem para complementar com estes nutrientes a dieta
diria de uma pessoa saudvel, em casos onde sua ingesto, a partir da alimentao, seja insuficiente ou
quando a dieta requerer suplementao. Devem conter um mnimo de 25% e no mximo at 100% da Ingesto
Diria Recomendada (IDR) de vitaminas e ou minerais, na poro diria indicada pelo fabricante, no podendo
substituir os alimentos, nem serem considerados como dieta exclusiva.
2.2. Classificao
Classificam-se como Suplementos:
- Vitaminas isoladas ou associadas entre si;
- Minerais isolados ou associados entre si;
- Associaes de vitaminas com minerais;
4. COMPOSIO E REQUISITOS
4.1. Composio
4.1.1. Os Suplementos Vitamnicos e ou de Minerais devem conter no mnimo 25% e no mximo 100% das IDR
para cada nutriente na poro diria indicada pelo fabricante. Para garantir a dosagem especificada na
rotulagem, permitida a sobredosagem de vitaminas e ou minerais , desde que justificada tecnologicamente.
4.1.2. A formulao de Suplementos para estados fisiolgicos especiais (gestantes e lactantes) deve se basear
nas IDR para cada caso.
4.1.3. As dosagens de vitaminas e minerais nos Suplementos devem ser calculadas com base nas IDR
estabelecida pela legislao especfica.
4.1.4.Nas formulaes de Suplementos, o fabricante deve evitar incompatibilidades tecnolgicas e ou
associaes de vitaminas e ou minerais em nveis que possam interferir negativamente na biodisponibilidade
desses nutrientes.
7. HIGIENE
Os Suplementos Vitamnicos e ou de Minerais devem ser preparados, manipulados, processados,
acondicionados e conservados conforme as Boas Prticas de Fabricao (BPF), e atender aos padres
microbiolgicos, microscpicos e fsico-qumicos estabelecidos por legislao especfica.
10. ROTULAGEM
10.1. proibida toda e qualquer expresso que se refira ao uso do Suplemento para prevenir, aliviar, tratar
uma enfermidade ou alterao do estado fisiolgico.
10.2. Os rtulos dos Suplementos devem observar a legislao para alimentos, no que couber, alm dos tens
abaixo discriminados:
10.2.2. A advertncia em destaque e em negrito: "Consumir este produto conforme a Recomendao de
Ingesto Diria constante da embalagem"
10.2.3. So permitidas somente informaes sobre as funes cientificamente comprovadas das vitaminas e
minerais, descrevendo o papel fisiolgico desses nutrientes no desenvolvimento e ou em funes do
organismo.
10.2.4. A recomendao do modo de ingesto do produto (quantidade, freqncia, condies especiais) e
modo de preparo, quando for o caso.
10.2.7. A orientao em destaque e em negrito: "Gestantes, nutrizes e crianas at 3 (trs) anos, somente
devem consumir este produto sob orientao de nutricionista ou mdico".
11. REGISTRO
11.1. Os Suplementos esto sujeitos aos mesmos procedimentos administrativos exigidos para o registro de
alimentos em geral.
11.2. No caso de Suplementos que no possuam padres ou aqueles que necessitem de atualizao, o
interessado deve apresentar ao rgo competente do Ministrio da Sade a proposta de Padro de Identidade
e Qualidade, para fins de registro.

You might also like